Sunteți pe pagina 1din 106

CAPITOLUL II

TEOREME FUNDAMENTALE DIN GEOMETRIA


TRIUNGHIULUI

II.1. Teorema bisectoarei interioare


„Teorema este mai presus de constatare şi mai presus de greşeală.”-Gh. Ţiţeica89

Teorema bisectoarei
BD AB
Fie triunghiul ABC şi AD , D ∈ ( BC ) bisectoarea unghiului BAC . Atunci, = .
DC AC
Demonstraţie. E

B D C

Fig. 233

Fie CE AD , E ∈ AB (Fig. 233 ). Atunci ACE ≡ DAC ( unghiuri alterne interne) şi


BAD ≡ CEA . Cum BAD ≡ DAC , rezultă ACE ≡ AEC , adică triunghiul ACE este isoscel,
BD AB AB
deci AC ≡ AE . Din teorema lui Thales rezultă: = = .
DC AE AC

Reciproca teoremei bisectoarei interioare


DB AB
În triunghiul ABC, fie D ∈ ( BC ) astfel încât = , atunci (AD este bisectoarea
DC AC
interioară a unghiului BAC.
Demonstraţie. Fie CE AD, E ∈ AB. Din teorema lui Thales în triunghiul BCE rezultă
BD AB DB AB
= , iar cu relaţia din ipoteză = obţinem AE = AC , adică triunghiul AEC
DC AE DC AC
este isoscel, deci AEC ≡ ACE (1). Cum AD CE rezultă DAC ≡ ACE (2)
(unghiuri alterne interne) şi BAD ≡ AEC (3) (unghiuri corespondente). Din relaţiile (1),
(2) şi (3) rezultă BAD ≡ DAC , adică AD este bisectoarea unghiului BAC.

89
Gheorghe Ţiţeica (1873-1939) – matematician român, profesor la Universitatea din Bucureşti, membru al
Academiei Române, contribuţii importante în geometrie
239
1) Segmentele determinate pe latura BC de bisectoarea AD au lungimea egală cu
ac ab
, respectiv .
b+c b+c
BD c BD CD a
Demonstraţie. Din teorema bisectoarei avem = , sau = = , de unde
DC b c b b+c
ac ab
BD = , şi CD = .
b+c b+c

2) În triunghiul ABC, fie D piciorul bisectoarei interioare a unghiului A, D ∈ ( BC ) .


2bc A
Atunci la = cos , unde cu la am notat lungimea segmentului AD.
b+c 2
Demonstraţie. Soluţia1. Din A[ A B D ] + A[ A D C ] = A[ A B C ] A
1 A 1 A 1
rezultă c ⋅ la ⋅ sin + b ⋅ la ⋅ sin = bc sin A , adică
2 2 2 2 2
2bc A
la = cos .
b+c 2
Soluţia 2. Din teorema bisectoarei avem :
B C
BD c BD c ac D
= ⇒ = ⇔ BD = şi
DC b BD + DC b + c b+c
Fig. 234
ab
DC = . Teorema sinusurilor aplicată în ABD ne
b+c
ac
sin B
la
de unde la = + c
BD b a b 2bc A
dă: = . Dar = , de unde: la = ⋅ cos .
sin B sin A sin
A sin A sin B b + c 2
2 2
2ac B
Analog, se obţin lungimile celorlalte bisectoare interioare: lb = ⋅ cos şi
a+c 2
2ab C
lc = ⋅ cos .
a+b 2

II.2. Teorema bisectoarei exterioare


„Toate invenţiile unui om sunt adevărate, poţi fi sigur de asta. Poezia
este atât ştiinţă cât şi geometrie.” – Gustave Flaubert (1821-1880)
Teorema bisectoarei exterioare
Fie triunghiul ABC şi AB ≠ AC. Dacă ( AE este bisectoarea exterioară a unghiului A,
EB AB
E ∈ BC , atunci = . T
EC AC
A
Demonstraţie. Fie b > c, deci
B ∈ ( EC ). Paralela prin B la AE
B1
intersectează latura AB în B1 (Fig. 235). Din
EB AB1 E
teorema lui Thales rezultă = (1). B
EC AC C
240 Fig. 235
Dar TAE ≡ AB1 B (unghiuri corespondente) şi EAB ≡ ABB1 (unghiuri alterne
interne), deci AB1 B ≡ ABB1 , adică triunghiul ABB1 este isoscel, de unde AB = AB1 (2).
EB AB
Din relaţiile (1) şi (2) rezultă = .
EC AC

Observaţii:
1) Condiţia AB ≠ AC din teoremă este esenţială pentru că dacă AB = AC , atunci
bisectoarea exterioară a unghiului A este paralelă cu BC, deci nu ar mai exista punctul E.
EB c c EB EB
2) Din teorema bisectoarei = (presupunând b > c ) rezultă = = ,
EC b b − c EC − EB a
ac ab
adică EB = şi analog EC = .
b−c b−c

Reciproca teoremei bisectoarei exterioare


EB AB
Fie triunghiul ABC şi E ∈ BC \ [ BC ] astfel încât = , atunci ( AE este
EC AC
bisectoarea exterioară a unghiului A.
Demonstraţie. Evident AB ≠ AC , deoarece astfel ar rezulta EB = EC ceea ce este imposibil
datorită faptului că E ∈ BC \ [ BC ] . Fie BB1 AE , B1 ∈ AC. Din teorema lui Thales în
EB AB1
triunghiul EAC rezultă = , care cu relaţia din ipoteză dă AB = AB1 , adică
EC AC
triunghiul ABB1 este isoscel, de unde obţinem că AB1 B ≡ ABB1 . Din AE BB1 rezultă
TAE ≡ AB1 B (unghiuri corespondente) şi EAB ≡ ABB1 (unghiuri alterne interne) şi
de aici obţinem că TAE ≡ EAB, adică (AE este bisectoarea exterioară a unghiului A.

1) Segmentele determinate pe dreapta BC de bisectoarea exterioară a unghiului A au


ac ab
lungimile egale cu , respectiv .
b−c b−c
EB c EB EC a
Demonstraţie. Din teprema bisectoarei exterioare avem: = , sau = =
EC b c b b−c
(unde am considerat b > c ).
B'
2) Fie A' , B ' , C ' picioarele bisectoarelor exterioare ale
unghiurilor triunghiului isoscel ABC. Punctele A
A' , B ' , C ' sunt coliniare.
Demonstraţie: A'
B C
Din teorema bisectoarei obţinem:
A 'B A B B ' C BC C ' A CA
= , = şi = . Avem:
A 'C A C B ' A BA C ' B CB
A' B B ' C C ' A AB BC CA
⋅ ⋅ = ⋅ ⋅ = 1 şi din reciproca
A' C B' A C ' B AC BA CB
teoremei lui Menelaus rezultă că punctele A' , B ' , C ' sunt
coliniare. C'

241
Fig. 236
Observaţie: Teorema de mai sus aparţine geometrului grec Pappus 90.

3) În triunghiul ABC, fie D ' piciorul bisectoarei exterioare a unghiului A, D ' ∈ (CB .
2bc A
Atunci la' = sin , unde cu la' am notat lungimea segmentului AD ' .
b−c 2
Demonstraţie. Deoarece:
A [ A D ' C ] − A[ A D ' B ] = A [ A B C ] rezultă
 A A A
b ⋅ l 'sin  90° +  − c ⋅ l 'cos = bc sin A
 2 2
2bc A
adică la ' = sin .
b−c 2

D' C
B D

Fig. 237

II.3. Teorema lui Pitagora91


„După ce a descoperit celebra sa teoremă, Pitagora a sacrificat o sută de boi. De atunci, de fiecare dată, când se
descoperă vreun adevăr nou, vitele cornute mari au palpitaţii.” - Ludwig Björne
C
Într-un triunghi dreptunghic pătratul lungimii ipotenuzei este egal cu
suma pătratelor lungimilor catetelor.
Demonstraţia 1. În triunghiul dreptunghic ABC ( m( BAC ) = 90° ) fie
înălţimea AD , D ∈ ( BC ) (Fig. 238). Din asemănarea triunghiurilor
AB BD D
ABD şi CBA rezultă = şi de aici AB 2 = BC ⋅ BD (1), iar din
BC AB
AC DC
asemănarea triunghiurilor ADC şi BAC rezultă = , de unde B
BD AC A
AC 2 = BC ⋅ DC (2). Din relaţiile (1) şi (2) rezultă : Fig. 238

AB 2 + AC 2 = BC ( BD + DC ) = BC ⋅ BC = BC 2 . R Q P
b c
Demonstraţia 2. Pe ipotenuza BC se construieşte c
pătratul CBNQ (Fig. 239). În prelungirea catetelor c
a a
AB şi AC se construieşte pătratul AMPR având C
latura de lungime b+c. Atunci,
bc N
A[ AMPR ] = A[ BCQN ] + 4 A[ ABC ] sau (b + c)2 = a2 + 4⋅ a a
2 b b
de unde rezultă a 2 = b 2 + c 2 .

A c B b M
90
Pappus (290 – 350) – matematician şi filosof grec; a pus bazele geometriei proiectiveFig. 239
91
Pitagora (Pythagoras) (c. 560 – c. 500 î. Hr.) – matematician, om politic şi filosof grec
242
II.4. Teorema lui Pitagora generalizată
„Când apa frânge o vargă, raţiunea o îndreaptă
Raţiunea mi-e stăpână înţeleaptă
Şi astfel, ochi-mi, ajutaţi de gând,
Nici nu mă înşeală, deşi mă mint oricând.”
La Fontaine92

Fie triunghiul ABC şi D proiecţia punctului A pe dreapta BC. Dacă


m( ACB) < 90°, atunci AB 2 = CA2 + CB 2 − 2CB ⋅ CD. Dacă m( ACB) > 90°, atunci
AB 2 = CA2 + CB 2 + 2CB ⋅ CD.
Demonstraţie.
A A

B C D
B D C
Fig. 241
Fig. 240

Din triunghiurile dreptunghice ABD şi ACD (Fig. 240) rezultă


AB = BD + AD , AC = AD + DC .
2 2 2 2 2 2
Dacă m( ABC ) < 90°, atunci
D ∈ ( BC ) şi BD = BC − CD. Dacă m( ABC ) > 90°, atunci B ∈ (CD ) şi BD = DC − BC ,
deci AB 2 = AD 2 + ( BC − CD )2 = ( AD 2 + CD 2 ) + BC 2 − 2 BC ⋅ CD , de unde rezultă
AB 2 = CA2 + CB 2 − 2CB ⋅ CD. Dacă m( ACB) > 90°, atunci BD = BC + CD (Fig. 241).
Avem: AB 2 = AD 2 + ( BC + CD) 2 = ( AD 2 + DC 2 ) + BC 2 + 2 BC ⋅ CD = CA2 + CB 2 + 2CB ⋅ CD.

1) Consecinţă: Teorema cosinusului


În orice triunghi ABC, având laturile de lungimi a, b, c, au loc relaţiile:
i) a 2 = b 2 + c 2 − 2bc cos A,
ii) b 2 = a 2 + c 2 − 2ac cos B,
iii) c 2 = a 2 + b 2 − 2ab cos C.
Demonstraţie.
A i) Fie D proiecţia lui A pe BC. Dacă m( ACB) < 90°, atunci
din teorema lui Pitagora generalizată avem:
c = a + b − 2a ⋅ CD .
2 2 2
Cum CD = b cos C rezultă
c 2 = a 2 + b 2 − 2ab cos C. Dacă m( ACB) = 90° atunci
c = a +b ,
2 2 2
adică teorema lui Pitagora. Dacă
B D C m( ACB) > 90°, atunci c = a + b − 2a ⋅ CD,
2 2 2

Fig. 242
92
La Fontaine (1621-1695) – poet, dramaturg francez
243
CD = CA cos(180° − C ) = −b cos C adică c 2 = a 2 + b 2 − 2ab cos C.
ii) şi iii) se demonstrează analog cu i).

Observaţii:
b2 + c2 − a2 a 2 + c2 − b2 b2 + a 2 − c2
1) În orice triunghi ABC, cos A = , cos B = , cos C = .
2bc 2ac 2ba
2) i) Dacă m( BAC ) < 90° , atunci cos A > 0 ⇔ a 2 < b 2 + c 2 .
ii) Dacă m( BAC ) = 90° , atunci cos A = 0 ⇔ a 2 = b 2 + c 2 .
iii) Dacă m( BAC ) > 90° , atunci cos A < 0 ⇔ a 2 > b 2 + c 2 .

2) Teorema lui Pappus. Formula medianei


Dacă M este mijlocul laturii BC a triunghiului ABC atunci:
AB 2 + AC 2 = 2( AM 2 + BM 2 ). A
Demonstraţie. Teorema cosinusului aplicată în
triunghiurile ABM şi AMC dă:
AB = BM + AM − 2 BM ⋅ AM ⋅ cos AMB (1)
2 2 2

AC 2 = MC 2 + AM 2 − 2 AM ⋅ MC ⋅ cos(π − AMB) =
MC 2 +AM 2 +2AM ⋅ MC ⋅ cos AMB (2) . Din relaţiile
(1) şi (2) prin sumare obţinem: C
AB + AC = 2( AM + BM ) , unde am ţinut cont că
2 2 2 2
B M
BM = MC.
Fig. 243
Observaţii:
1) Expresia AB 2 + AC 2 = 2( AM 2 + BM 2 ) se numeşte relaţia lui Pappus.
2) Dacă a,b,c sunt lungimile laturilor triunghiului ABC iar ma lungimea medianei AM,
2
2(b + c ) − a
2 2
relaţia lui Pappus devine: ma2 = ( Formula medianei ).
4
3) Prin permutări circulare ale relaţiei precedente se obţin următoarele egalităţi:
2(c 2 + a 2 ) − b 2 2(b 2 + a 2 ) − c 2
mb2 = , mc2 = .
4 4
4) Teorema lui Pappus ne oferă un mod de a determina lungimile medianelor în funcţie de
lungimile laturilor triunghiului.

3) Consecinţă: Dacă a, b, c sunt lungimile laturilor unui triunghi ABC şi ma , mb , mc


sunt lungimile medianelor triunghiului ABC, atunci: i)
4(ma + mb + mc ) = 3(a + b + c ) ,
2 2 2 2 2 2
ii)
16(ma4 + mb4 + mc4 ) = 9(a 4 + b 4 + c 4 ) ( Relaţia lui Cesaro)
Demonstraţia se realizează înlocuind formula medianei în relaţiile date.

244
II.5. Teorema lui Stewart93
„Geometria este cea mai bună şi mai simplă dintre toate logicile, cea
mai potrivită să dea inflexibilitate judecăţii şi raţiunii.” – Denis Diderot94

Fie triunghiul ABC şi M un punct pe latura BC. Atunci:


AB 2 ⋅ MC + AC 2 ⋅ BM − AM 2 ⋅ BC = BC ⋅ BM ⋅ MC.
Demonstraţie.
Aplicând teorema cosinusului în triunghiurile
A ABM şi AMC obţinem:
AB 2 = AM 2 + BM 2 − 2 AM ⋅ BM ⋅ cos AMB
AC 2 = AM 2 + MC 2 − 2 AM ⋅ MC ⋅ cos AMC.
Cum cos( AMC) = cos(180° − AMB) = − cos AMB,
B C rezultă:
M AB2 ⋅ MC = AM 2 ⋅ MC + BM 2 ⋅ MC − 2AM ⋅ BM ⋅ CM ⋅ cos AMB
Fig. 244 AC2 ⋅ MB = AM 2 ⋅ MB + CM 2 ⋅ MB + 2AM ⋅ BM ⋅ CM ⋅ cos AMB.
Sumând egalităţile precedente obţinem:
AB2 ⋅ MC + AC2 ⋅ BM = AM2 (MC + MB) + BM ⋅ MC(MB + MC)
adică AB 2 ⋅ MC + AC 2 ⋅ BM = AM 2 ⋅ BC + BM ⋅ MC ⋅ BC .

Consecinţe:
1) Teorema medianei
2(b 2 + c 2 ) − a 2
Fie M mijlocul laturii BC a triunghiului ABC. Atunci, ma2 = (unde ma
4
reprezintă lungimea medianei AM).
a
Demonstraţie. Avem BM = MC = . Din relaţia lui Stewart aplicată în triunghiul ABC şi
2
2(b 2
+ c 2
) − a2
punctului M rezultă ma2 = .
4

2) Lungimea bisectoarei interioare


Fie triunghiul ABC, ( AD bisectoarea interioară a unghiului BAC , unde D ∈ BC ).
4bc
Atunci AD 2 = p( p − a) , unde p este semiperimetrul triunghiului ABC.
(b + c) 2
c BD
Demonstraţie. Din teorema bisectoarei rezultă = , de A
b DC
c + b BD + DC ab ac
unde = DC = (1) şi BD = (2)
b DC b+c b+c
(Fig. 245). Teorema lui Stewart în ABC pentru M ≡ D dă:
AD 2 ⋅ a = c 2 ⋅ DC + b 2 ⋅ BD − a 2 ⋅ DB ⋅ DC (3). Din relaţiile (1), B
D C
Fig. 245

93
Matthew Stewart (1714-1785) – geometru scoţian, profesor la Universitatea din Edinburgh
94
Denis Diderot (1713-1784) – scriitor şi filosof francez
245
4bc
(2) şi (3) rezultă la2 = p( p − a) (unde prin la am notat lungimea bisectoarei AD).
(b + c ) 2

3) Lungimea bisetoarei exterioare


Fie triunghiul ABC, (AE bisectoarea exterioară a unghiului A, E ∈ BC ). Atunci,
4bc( p − b)( p − c)
AE 2 = .
(b − c ) 2
Demonstraţie.Fie b > c, deci B ∈ ( EC ) (Fig. 246). Din teorema lui Stewart aplicată în
triunghiul AEC rezultă: AE ⋅ BC + AC ⋅ EB − AB2 ⋅ EC = AB⋅ EB⋅ BC (∗).
2 2
Din teorema
bisectoarei exterioare avem:
A
c EB
= de unde rezultă:
b EC
c EB EB
= = şi
b − c EC − EB a
ac ab E
EB = ; analog EC = . B C
b−c b−c
Relaţia (∗) devine Fig. 246
ac ab ac
AE 2 ⋅ a + b 2 ⋅ − c2 ⋅ = c⋅ ⋅a
b−c b−c b−c
4bc( p − b)( p − c) a+b+c
de unde rezultă: AE 2 = , unde p = .
(b − c)2 2

II.6. Teorema sinusurilor


„Fiecare problemă pe care am rezolvat-o a devenit o regulă care pe
urmă mi-a servit la rezolvarea altor probleme.” – René Descartes95

În orice triunghi ABC, raza R a cercului circumscris verifică egalitatea:


a b c
= = = 2 R.
sin A sin B sin C A
Demonstraţie. Vom demonstra teorema pentru cele D
trei cazuri date de natura triunghiului ABC.
i) Triunghiul ABC este ascuţitunghic (Fig. 247).
Fie diametrul BD. Atunci BCD este dreptunghic .
1
Avem: m( BAC ) = m( BDC ) = m( BXC ) de unde C
2
BC a
sin(BAC) = sin(BDC) = = . Analog, avem : B
BD 2 R •

b c X
sin( ABC ) = şi sin( ACB) = .
2R 2R Fig. 247
ii) Triunghiul ABC este dreptunghic. Fie

95
René Descartes (1596-1650) – matematician şi filosof francez, contribuţii în geometrie
246
b c
m( BAC ) = 90° . Avem sin( BAC ) = 1, sin( ABC ) = şi sin( BCA) = . Cum a = 2 R
a a
concluzia este evidentă.
iii) Triunghiul ABC este obtuzunghic (Fig. 248).
A
Fie sin( BAC ) > 90°. În triunghiul BCD
a C
(m( BCD ) = 90°) , avem: . sin( BDC ) =
2R
Deoarece patrulaterul ABCD este inscriptibil
B D
rezultă m( BAC ) + m( BDC ) = 180°, deci O
a
sin(π / 2 − BAC) = sin(BAC) = . Pentru unghiurile
2R
ascuţite ABC şi BCA se repetă demonstraţia de la
subpunctul i). Fig. 248

II.7. Teorema lui Ceva96


„Geometria este ştiinţa care restaurează situaţia dinainte de creaţia lumii
şi încearcă să umple "golul", renunţând la oficiile materiei.” - L. Blaga97
Teorema lui Ceva
Fie triunghiul ABC şi punctele D ∈ BC , E ∈ CA , F ∈ AB . Dacă dreptele AD, BE şi CF
AF BD CE
sunt concurente, atunci ⋅ ⋅ = 1.
FB DC EA
Demonstraţie: Fie {K } = AD ∩ BE ∩ CF .
Prin A ducem o paralelă la BC, iar G şi F A
sunt punctele de intersecţie dintre dreptele BE
respectiv CF cu această paralelă. Din H G
AF AH F
AHF ∼ BCF rezultă = (1),
FB BC E
CE BC K
BCE ∼ AEG rezultă = (2),
EA AG
AG AK B D C
AGK ∼ BDK rezultă = (3),
BD DK Fig. 249
AH AK
CDK ∼ AHK rezultă = (4). Din
DC DK
AG AH AG BD
relaţiile (3) şi (4) obţinem = de unde = (5). Din relaţiile (1) , (2) şi (5)
BD DC AH DC
AF BD CE AH AG BC
rezultă ⋅ ⋅ = ⋅ ⋅ =1
FB DC EA BC AH AG

96
Giovanni Ceva (1647-1734) – matematician italian, profesor la Universitatea din Mantua, contribuţii în
geometrie
97
Lucian Blaga (1895-1961) - filozof, umanist, jurnalist, poet, dramaturg, traducător, profesor universitar şi
diplomat român, membru titular al Academiei Române

247
Reciproca teoremei lui Ceva
AF BD CE
Fie triunghiul ABC şi punctele D ∈ BC , E ∈ CA , F ∈ AB . Dacă ⋅ ⋅ = 1,
FB DC EA
atunci dreptele AD, BE şi CF sunt concurente.
Demonstraţie. Fie {K } = BE ∩ CF şi {D '} = AK ∩ BC . Conform primei părţi rezultă
AF BD ' CE BD ' BD
⋅ ⋅ = 1 care împreună cu relaţia din ipoteză dă : = de unde
FB D ' C EA D ' C DC
BD '+ D ' C BD + DC BC BC
= deci = şi de aici rezultă că D ' C = DC , adică D ≡ D '.
D 'C DC D ' C DC

Observaţii:
1) Reciproca teoremei lui Ceva este adevărată şi în cazul în care unul din punctele D,E, sau
F aparţine unei laturi – de exemplu D ∈ BC - şi celelalte două puncte E ∈ CA , F ∈ AB
verifică condiţia: “dreapta BE nu este paralelă cu dreapta CF.
2) Dacă BE CF reciproca teoremei lui Ceva nu mai este
adevărată, aşa cum o arată următorul exemplu: „Fie D E F
mijlocul segmentului BC, F simetricul lui B faţă de A şi E A
simetricul lui C faţă de A. Atunci,
AF BD CE 1
⋅ ⋅ = ⋅1 ⋅ 2 = 1 , dar dreptele AD, BE, CF nu
FB DC EA 2 B D C
sunt concurente (deoarece AD BE CF , AD fiind linie
Fig. 250
mijlocie în triunghiurile BEC şi BFC.”

II.8. Teorema lui Menelaus98


„Un punct pierdut e lumea în haosul imens.
Toată ştiinţa noastră: cuvinte fără sens.
Om, pasăre şi floare sunt umbre în abis.
Zadarnic este gândul, iar existenţa - vis.”
Omar Khayyam99
Teorema lui Menelaus
Fie triunghiul ABC şi punctele A ' ∈ BC , B ' ∈ CA , C ' ∈ AB . Punctele A ', B ', C ' sunt
coliniare dacă şi numai dacă
A' B B 'C C ' A A
⋅ ⋅ =1.
A 'C B ' A C ' B
Demonstraţie. Presupunem că C1
punctele A ', B ', C ' sunt coliniare. B '
Conform axiomei lui Pasch, cel puţin P
A1
unul din punctele A ', B ', C ' se află pe
C'
prelungirea laturilor triunghiului
ABC . Fără a restrânge generalitatea B1
putem presupune că
A' B C
Fig. 251
98
Menelaus (70-130) – mathematician grec, contribuţii importante în geometrie
99
Omar Khayyam (1048-1122) – matematician, poet, filosof, astronom persan, contribuţii în algebră şi geometrie
248
B ' ∈ ( AC ) , C ' ∈ ( AB) şi A ' ∈ [CB \ [CB ] (Fig. 251).
Soluţia 1. Fie A1 , B1 , C1 proiecţiile punctelor A, B, C pe dreapta A ' B '. .Din asemănările
triunghiurilor: A ' BB1 şi A ' CC1 ; B ' CC1 şi B ' AA1 ; C ' AA1 şi C ' BB1 rezultă egalităţile
A ' B BB1 B ' C CC1 C ' A AA1 A' B B 'C C ' A
= , = , = care prin înmulţire dau: ⋅ ⋅ =1.
A ' C CC1 B ' A AA1 C ' B BB1 A 'C B ' A C ' B
A[ AC ' B '] A[ BC ' A '] A[ CA ' B ']
Soluţia 2. Egalitatea evidentă ⋅ ⋅ =1 este echivalentă cu:
A[ BC ' A '] A[ CA ' B '] A[ AC ' B ']
C ' A⋅C ' B '⋅sin AC ' B ' A ' C '⋅ A ' B⋅sin C ' A ' B B ' A '⋅B ' C⋅sin A ' B ' C
⋅ ⋅ = 1, adică
C ' B⋅C ' A '⋅sin A ' C ' B A ' B '⋅ A 'C ⋅sin CA ' B ' B ' A⋅B ' C '⋅sin(π − A ' B ' C )
A' B B 'C C ' A
⋅ ⋅ = 1.
A 'C B ' A C ' B
Soluţia 3. Fie BP A ' B ' ( P ∈ AC ) . Din asemănarea triunghiurilor BPC cu A ' B ' C ,
B ' P A' B B'A C'A
respectiv a triunghiurilor AC ' B ' cu ABP rezultă: = şi = care prin
B 'C A 'C B'P C 'B
înmulţire dau concluzia.
A' B B 'C C ' A
Reciproc, presupunem că ⋅ ⋅ = 1 (1) şi demonstrăm că punctele A ', B ', C '
A 'C B ' A C ' B
sunt coliniare. Fie A ' ∈ [CB \ [CB ] , C ' ∈ ( AB) şi {B "} = A ' C '∩ AC . Atunci, conform
A ' B B "C C ' A B ' C B "C
primei părţi rezultă: ⋅ ⋅ = 1 care cu relaţia (1) dă = şi de aici
A 'C B " A C ' B B ' A B" A
B ' C B "C
= ,adică B ' C = B " C şi cum există doar un punct interior laturii AC pentru care
AC AC
B ' C = B " C , rezultă B ' ≡ B " , deci punctele A ', B ', C ' sunt coliniare.
Soluţia 4. Considerăm cazul când două puncte sunt pe laturi şi unul pe prelungirea unei
A' B B 'C C'A A 'C 1 A 'C 1
laturi Notăm: α = ,β = ,γ = . Din = rezultă = , deci
A'C B'A C 'B A' B α BC 1 + α
uuuur 1 uuur B 'C uuuur β uuur
CA ' = ⋅ CB . Din =β rezultă CB ' = CA , iar
1+α B' A 1+ β
uuuur uuur uuuur uuur AC ' uuur uuur γ uuur uuuuur
CC ' = CA + AC ' = CA + ⋅ AB = CA + ⋅ AB . Putem exprima acum vectorii B ' A '
AB γ −1
uuuuur uuuuur uuuur uuuur 1 uuu
r β r uuuuur uuuur uuuur uuur γ uuur β uuur
uuu
şi B ' C ' : B ' A' = CA' − CB ' = CB − CA , B ' C ' = CC ' − CB ' = CA + AB − CA =
1+ α 1+ β γ −1 1+ β
1 uuur γ uuur uuur −1− βγ uuur γ uuur uuuuur uuuuur
CA + ( AC + CB) = CA + CB . Din condiţia ca vectorii B ' A ' şi B ' C '
1+ β γ −1 (1+ β )(γ −1) γ −1
γ −1 β (1 + β )(γ − 1)
să fie coliniari, rezultă = , de unde obţinem αβγ = 1 sau
(1 + α )γ (1 + β )(1 + βγ )
A' B B 'C C ' A
⋅ ⋅ = 1 .Analog se tratează cazul când punctele A ', B ', C ' sunt pe prelungirile
A 'C B ' A C ' B
laturilor. Pentru demonstraţia afirmaţiei reciproce, fie αβγ = 1 şi notând

249
A' B B 'C C'A 1 uuuuur uuuur uuuur 1 uuur β uuur
α= ,β = ,γ = = , avem B ' A ' = CA ' − CB ' = CB − CA (1),
A 'C C 'B C ' B αβ 1+α 1+ β
uuuuur uuuur uuuur 1 + βγ uuur γ uuur 1
B ' C ' = CC ' − CB ' = − CA + CB şi înlocuind γ = obţinem
(1 + β )(γ − 1) γ −1 αβ
uuuuur α + 1  1 uuur β uuur  uuuuur α uuuuur
B 'C ' =  CB − CA  (2). Din (1) şi (2) avem B ' C ' = B ' A ' , deci
1 − αβ  1 + β 1+ β  1 − αβ
A ', B ', C ' sunt puncte coliniare.

Teorema lui Menelaus pentru patrulatere

Dacă X, Y, Z, W sunt puncte coliniare pe laturile AB, BC, CD, respectiv DA ale
AX BY CZ DW
patrulaterului ABCD, atunci ⋅ ⋅ ⋅ = 1.
XB YC ZD WA
Demonstraţie.
D

X W
A
T

B Y C

Fig. 252 Z

Fie {T } = BD ∩ XY . Din teorema lui Menelaus aplicată în triunghiurile ABD şi BCD


XA WD TB TD ZC YB
rezultă: ⋅ ⋅ = 1 şi ⋅ ⋅ = 1, relaţii care prin înmulţire dau concluzia.
XB WA TD TB ZD YC

II.9. Teorema transversalei


„Matematica nu se face în stare de urgenţă.” – Ion Cucurezeanu100
Teorema transversalei
Dacă într-un triunghi ABC se duce o ceviană AD, iar o secantă oarecare intersectează
AM AC PN DB
dreptele AB, AC şi AD în punctele M, N, respectiv P, atunci ⋅ ⋅ ⋅ = 1.
AB AN PM DC
Demonstraţie. Fie B ', C ', M ' şi N ' proiecţiile punctelor B, C, M, respectiv N pe AD. Din
asemănarea triunghiurilor AMM ' şi ABB ' , ACC ' şi ANN ' , PNN ' şi PMM ' , BB ' D şi

100
Ion Cucurezeanu – matematician român, profesor la Universitatea din Constanţa,contribuţii în studiul ecuaţiilor
diofantice
250
AM MM ' AC CC ' PN NN ' DB BB '
CC ' D rezultă: = , = , = şi = . Înmulţind
AB BB ' AN NN ' PM MM ' DC CC '
AM AC PN DB
membru cu membru relaţiile precedente rezultă: ⋅ ⋅ ⋅ = 1.
AB AN PM DC

1) Dacă într-un triunghi ABC se duce A


o ceviană AD, D ∈ ( BC ) şi o secantă
intersectează pe AB, AC şi AD în M'
punctele M, N, respectiv P, atunci:
MB NC PD P
⋅ DC + ⋅ BD = ⋅ BC. M N
MA NA PA
Demonstraţie. Din aplicaţia precedentă B' N'
în triunghiurile ABD şi ADC cu D
cevianele AC respectiv AB şi secanta
NP AP AB DC B C
MN rezultă: = ⋅ ⋅ şi C'
NM AM AM BC
Fig. 253
MP AP AC BD
= ⋅ ⋅ . Sumând relaţiile
MN AD AN BC
AP  AB DC AC BD  AD AB DC AC BD
precedente obţinem 1 =  ⋅ + ⋅  , de unde = ⋅ + ⋅ .
AD  AM BC AN BC  AP AM BC AN BC
Cum AB = AM + MB, AC = AN + NC , BC = BD + DC şi AD = AP + PD rezultă
MB NC PD
⋅ DC + ⋅ BD = ⋅ BC.
MA NA PA

2) Fie triunghiul ABC şi punctele D ∈ ( BC ), E ∈ ( AB ), F ∈ ( AC ), M ∈ ( AD ). Dacă


EB FC MD
DC ⋅ + BD ⋅ = BC ⋅ , atunci M ∈ EF .
EA FA MA
Demonstraţie. Fie {M '} = AD ∩ EF . Din teorema transversalei rezultă
EB FC M 'D MD M ' D
DC ⋅ + BD ⋅ = BC ⋅ care împreună cu relaţia din ipoteză dă: = sau
EA FA M 'A MA M ' A
MD + MA M ' D + M ' A AD AD
= , adică = . Din relaţia precedentă avem : MA = M ' A ,
MA M 'A MA M ' A
deci M ≡ M ' .

Observaţie: Teorema lui Menelaus este o consecinţă a teoremei transversalei.

251
II.10. Teorema lui Leibniz101
„Sub aspect elementar, numeroase teoreme interesante sunt create mereu fie de către amatori devotaţi, fie de către
marii matematicieni, care ori de câte ori au înţelegerea să revină la problemele elementare, le-au privit sub aspecte
noi, dând demonstraţii mai simple sau încadrări mai naturale.” – N. Mihăileanu102

Fie G centrul greutate al triunghiului ABC. Pentru orice punct M


de
din planul triunghiului ABC este adevărată relaţia:
AB 2
+ BC 2
+ CA 2
MA2 + MB 2 + MC 2 = + 3MG 2 (∗).
3
Demonstraţie.
Fie A ' mijlocul laturii BC. Relaţia lui Stewart
A aplicată în triunghiul AMA ' dă:
MA2 ⋅ A ' G + MA '2 ⋅ AG − AA '⋅ AG ⋅ GA ' = MG2 ⋅ AA '
AA ' 2
Egalităţile A ' G = , AG = A A ',
M 3 3
G 2(M B 2 + M C 2 ) − B C 2
M A '2 = ,
4
B 2( AB2 + AC2 ) − BC2
A' C AA'2 = înlocuite în relaţia
4
precedentă dau concluzia.
Fig. 254

Consecinţe:
AB 2 + BC 2 + CA2
1) Dacă M ≡ G, atunci GA2 + GB 2 + GC 2 = şi relaţia din teorema lui
3
Leibniz devine MA2 + MB 2 + MC 2 = GA2 + GB 2 + GC 2 + 3MG 2 .
AB 2 + AC 2 + BC 2
2) Din relaţia lui Leibniz rezultă că MA2 + MB 2 + MC 2 ≥ cu egalitate
3
dacă punctul M coicide cu G.
3) Dacă M coincide cu O – centrul cercului circumscris triunghiului ABC - atunci relaţia
a2 + b2 + c 2 a2 + b2 + c2
(∗) devine: 3OA2 = + 3OG 2 , adică OG 2 = R 2 − .
3 9
4) În orice triunghi ABC este adevărată relaţia: 9 R 2 ≥ a 2 + b 2 + c 2 .
a2 + b2 + c2
Demonstraţie: Cum OG 2 ≥ 0 avem: R 2 ≥ , adică 9R 2 ≥ a 2 + b 2 + c 2 .
9
5) Fie H şi O ortocentrul respectiv centrul cercului circumscris triunghiului ABC. Atunci:
4(a 2 + b 2 + c 2 )
i) OH 2 = 9 R 2 − (a 2 + b 2 + c 2 ), ii) GH 2 = 4 R 2 − unde R este lungimea razei
9
cercului circumscris triunghiului ABC şi a, b, c lungimile laturilor triunghiului ABC.

101
Gottfried von Leibniz (1646-1716) – matematician şi filosof german, contribuţii importante în analiza
matematică
102
Nicolae Mihăileanu (1912-1998) – matematician român
252
Demonstraţie: i) Din relaţia cunoscută OH = 3OG rezultă OH 2 = 9OG 2 care împreună cu
1
relaţia de la observaţia 3) ne dă OH 2 = 9 R 2 − (a 2 + b 2 + c 2 ) . ii) Cum OG = HG rezultă
2
1 4( a 2
+ b 2
+ c 2
)
OG 2 = HG 2 , de unde HG 2 = 4 R 2 − .
4 9

II.11. Teorema lui Toricelli - Fermat


„În matematică nu există ignorabimus, nu vom şti...., trebuie să ştim şi vom şti!” - David Hilbert103

Să se găsească punctul P din planul unui triunghi ABC pentru care suma PA + PB + PC
este minimă.
Demonstraţie. Soluţia 1. Prin rotaţia de
centru B şi unghi de 60° a triunghiului ABP A
se obţine triunghiul C ' BP ' . Atunci,
PB = P ' P şi PA = C ' P ' , de unde C'
P'
PA + PB + PC = P ' P + PC ≥ C ' C . Suma
este minimă atunci când punctul P ∈ C ' C ,
P
adică m( BPC ') = 60° . Analog, prin 60°
rotaţia de centru A şi unghi de 60° a
B C
triunghiului ABP se obţine:
m( APC ') = 60° , deci m( APB ) = 120° . Fig. 255
Analog, se arată că punctul P aparţine
dreptelor BB ', AA ' ( B ' şi A ' se obţine ca mai sus), deci punctul P căutat se află la
intersecţia dreptelor AA ', BB ', CC ' .
Soluţia 2: Fie P punctul pentru care suma PA + PB + PC este minimă şi d a dreapta ce
conţine punctele P şi A. Arătăm că dacă, de exemplu, punctul P se plimbă pe dreapta d a
punctul căutat P rămâne acelaşi. Fie că A1 ∈ AP şi presupunem că P1 este punctul pentru
care suma P1 A1 + P1 B + PC
1 este minimă. Astfel, pentru triunghiul ABC avem:
PA + PB + PC < P1 A + P1 B + PC 1 şi pentru : P1 A1 + P1 B + PC
1 < PA1 + PB + PC relaţii care
sumate dau PA + P1 A1 < PA1 + P1 A , sau PA1 + A1 A + P1 A1 < PA1 + P1 A , de unde
rezultă A1 A + P1 A1 < P1 A , absurd. Deci, dacă A ∈ d a , atunci poziţia punctului P pentru care
se realizează minimul nu se schimbă. Analog, se demonstrează proprietatea de mai sus şi
pentru punctele B şi C . Astfel, putem alege punctele B ∈ db şi C ∈ d c astfel încât
triunghiul ABC să fie echilateral, acest lucru poate fi realizat. De exemplu, alegem A ∈ d a
astfel încât AB = BC . Evident, dacă triunghiul ABC este isoscel, punctul P aparţine axei de
simetrie a triunghiului ABC. Plimbăm acum punctul B ∈ db ( iar P ∈ db ) astfel încât
triunghiul ABC devine echilateral şi atunci m( APB ) = m( APC ) = m( BPC ) = 120° .

103
David Hilbert (1962-1943) – matematician german, profesor la Universitatea din Göttingen, contribuţii
remarcabile în geometrie şi analiza matematică
253
Soluţia 3: Fie P un punct situat în interiorul
triunghiului ABC astfel încât m( APB) = m( APC )
Presupunem că lungimea segmentului [ PA] este A
constantă. Fie cercul cu centrul în A şi rază PA şi
tangenta d în P la cerc. Fie P1 ∈ d , P1 ≠ P, R
d
AP1 ∩ C(A,PA) = {R}. Cum APB ≡ APC rezultă: P1 P
PB + PC < P1 B + PC
1 < RB + RC şi de aici
PA + PB + PC < PA + RB + RC = RA + RB + RC . B C
Repetând raţionamentul pentru PB sau PC
constante rezultă că minimul se obţine pentru Fig. 256
APB ≡ APC ≡ BPC (= 120°) .

Observaţii:
1) Punctul P se numeşte punctul lui Fermat104 sau punctul izogon al triunghiului ABC.
2) Demonstraţia de mai sus nu mai este valabilă dacă un unghi al triunghiului ABC are
măsura mai mare de 120° (vezi „Triunghiurile lui Napoleon. Punctele lui Fermat”).

Generalizarea teoremei lui Toricelli - Fermat


Fie ABC şi DEF două triunghiuri de laturi a, b, c respectiv d, e, f. În exteriorul
triunghiului ABC se construiesc triunghiurile A ' B C , A B ' C , A B C ' asemenea
cu DEF, m( A) + m( B ) < 180°, m( B ) + m( E ) < 180°, m(C ) + m( F ) < 180° . Atunci:
a) d ⋅ AA ' = e ⋅ BB ' = f ⋅ CC ' ;
b) cercurile circumscrise triunghiurilor A ' BC , AB ' C şi ABC ' au un punct comun T;
c) dreptele AA ', BB ' şi CC ' sunt concurente în punctul T;
d) d ⋅ TA '+ e ⋅ TB '+ f ⋅ TC ' = 2(d ⋅ TA + e ⋅ TB + f ⋅ TC ) ;
e) suma d ⋅ MA + e ⋅ MB + f ⋅ MC este minimă când M coincide cu T;
f) 2(d ⋅ TA + e ⋅ TB + f ⋅ TC)2 = a2 (−d 2 + e2 + f 2 ) + b2 (d 2 − e2 + f 2 ) + c2 (d 2 + e2 − f 2 ) + 16 ⋅ S ⋅ S ',
unde S şi S ' sunt ariile triunghiurilor ABC respectiv DEF ;
g) Dacă OA , OB , OC sunt centrele cercurilor circumscrise triunghiurilor A ' BC , AB ' C
respectiv ABC ', triunghiurile OAOB OC şi DEF sunt asemenea.
CA ' BC
Demonstraţie. a) Din asemănarea triunghiurilor A ' BC şi AB ' C rezultă = şi
AC B ' C
cum A ' CA ≡ BCB ' rezultă că triunghiurile A ' CA şi BCB ' de unde
AA ' AC DF e
= = = , adică d ⋅ AA ' = e ⋅ BB '. Analog se arată că e ⋅ BB ' = f ⋅ CC ' de
BB ' B ' C EF d
unde rezultă d ⋅ AA ' = e ⋅ BB ' = f ⋅ CC ' .
b) Fie T al doilea punct de intersecţie dintre cercurile circumscrise triunghiurilor BCA ' şi
AB ' C. Atunci, m( BTC ) = 180° − m( BA ' C ) = 180° − m( D) şi
m( ATC) =180°− m( CB' A) =180°− m( E). Pentru că m( E ) + m( B) < 180° rezultă că T
aparţine arcelor cercurilor considerate aflate în interiorul triunghiului ABC. Atunci:
m( ATB) = 360° − m( BTC) − m( ATC) = 360° − (180° − m( D )) − (180° − m( E )) =

104
Pierre de Fermat (1601-1665) – matematician francez, contribuţii în teoria probabilităţilor şi teoria numerelor
254
180° − m( F ) = 180° − m( AC ' B ) , adică patrulaterul TAC ' B este inscriptibil, deci T
aparţine şi cercului circumscris triunghiului ABC '.
c) Deoarece patrulaterul BTCA ' este inscriptibil rezultă
BTA ' ≡ BCA ' ≡ DFE ≡ AC ' B şi cum m( AC ' B) + m( ATB) = 180° rezultă
m( BTA ') + m( BTA) = 180°, adică punctele A, T şi A ' sunt coliniare. Analog se arată că
punctele B, T , B ' şi respectiv C , T , C ' sunt
A
coliniare, deci {T } = AA '∩ BB '∩ CC '.
d) Din teorema lui Ptolemeu pentru patrulaterul
inscriptibil TBA ' C rezultă OC
TA '⋅ BC = TB ⋅ A ' C + TC ⋅ A ' B (1). Din P OB
C' B'
asemănarea triunghiurilor A ' BC şi DEF avem: T
A ' B A ' C BC
= = = k (2). Din relaţiile (1) şi B Q R C
DE DF EF
(2) rezultă TA '⋅ k ⋅ d = TB ⋅ e ⋅ f + TC ⋅ f ⋅ k , adică
TA '⋅ d = TB ⋅ e + TC ⋅ f . Analog se arată că:
OA
TB '⋅ e = TA ⋅ d + TC ⋅ f şi TC '⋅ f = TA ⋅ d + TB ⋅ e.
Sumând ultimele trei egalităţi membru cu
membru rezultă:
TA'⋅ d +TB'⋅ e +TC'⋅ f = 2(TA⋅ d +TB⋅ e +TC⋅ f ). A'
e) Fie M un punct arbitrar situat în planul
triunghiului ABC. Atunci, Fig. 257
d ⋅ AA ' ≤ d ( AM + MA ') = d ⋅ AM + e ⋅ BM + f ⋅ CM
cu egalitate atunci când M ∈ BTC I AA ', adică când M coincide cu T.
f) Din subpunctul precedent d ⋅ AA ' = d ⋅ AM + e ⋅ BM + f ⋅ CM . Determinăm pe AA ' din
triunghiul BAA ' aplicând teorema cosinusului: AA'2 = BA2 + BA'2 − 2BA⋅ BA'cos( B + E) ,
adică ( d ⋅ AA ') 2 = ( dc ) 2 + ( d ⋅ BA ') 2 − 2( d ⋅ BA ) ⋅ ( d ⋅ BA ') ⋅ [cos B cos E − sin B sin E ]
 a2 + c2 − b2 d 2 + f 2 − e2 
sau ( d ⋅ AA ') 2 = d 2 c 2 + a 2 f 2 − 2 d ⋅ c ⋅ a ⋅ f  ⋅ − sin B ⋅ sin E  ,
 2 ac 2 df 
( a 2 + c 2 − b 2 )( d 2 + f 2 − e 2 )
de unde ( d ⋅ AA ') 2 = d 2 c 2 + a 2 f 2 − − 2( ac sin B ) ⋅ ( df sin E ) şi
2
deci: 2(d ⋅ TA + e ⋅ TB + f ⋅ TC)2 = a2 (−d 2 + e2 + f 2 ) + b2 (d 2 − e2 + f 2 ) + c2 (d 2 + e2 − f 2 ) + 16 ⋅ S ⋅ S '.
g) Fie {P} = AT ∩ OB OC ,{Q} = BT ∩ OAOC ,{R} = CT ∩ OAOB . Deoarece OAOB ⊥ CT şi
OAOC ⊥ BT rezultă că patrulaterul OA RTQ este inscriptibil, deci
m( QOA R) = 180° − m ( QTR ) = 180° − m( BTC ) = 180° − [180° − m( BA ' C )] = m( BA ' C ) = m( D).
Analog se arată că m( OAOB OC ) = m( E ) şi m( OAOC OB ) = m( F ), adică triunghiurile
OAOB OC şi DEF sunt asemenea.

255
Observaţii:
1) Dacă ρ este raza cercului circumscris triunghiului DEF atunci OB C = b ⋅ ρ şi
ρ ⋅ AA' ρ ⋅ d ⋅ AA' ρ ⋅ d ⋅ AA'⋅ f d ⋅ AA' e ⋅ BB '
OAOB = = = = ⋅ f. Analog OBOC = ⋅d şi
e d ⋅e 4ρ S ' 4S ' 4S '
f ⋅ CC ' OO OO O O d ⋅ AA '
OC OA = ⋅ e, deci A B = B C = C A = ( d ⋅ AA ' = e ⋅ BB ' = f ⋅ CC ' ).
4S ' f d e 4S '
2) Dacă triunghiul DEF este echilateral se obţine teorema lui Toricelli.

II.12. Teorema lui Feuerbach105


„Ca să te îndoieşti de linia dreaptă
trebuie să ştii mai întâi din câte puncte
e făcută.” – Nichita Stănescu106
Teorema lui Feuerbach
Într-un triunghi, cercul lui Euler este tangent cercului înscris şi cercurilor exînscrise
corespunzătoare.
Demonstraţie. Soluţia 1. Fie A ' intersecţia A
bisectoarei interioare a unghiului BAC cu latura Cb
ϕ
BC, H a H b H c triunghiul ortic al triunghiului ABC, Cc I
M a , M b , M c mijloacele laturilor BC , AB respectiv
A ' Ma B
AC ale triunghiului ABC , Ca Cb Cc triunghiul de C H a C a ϕa D a Db
contact, iar Da , Db , Dc proiecţiile punctului I a –
centrul cercului A - exînscris - pe dreptele BC, CA D'a
respectiv AB (Fig. 258). Vom demonstra mai întâi
că: M a Ca2 = M a H a ⋅ M a A ' . Din teorema bisectoarei Dc Ia
BA ' c a−c
rezultă = , de unde BA ' = . Avem:
A 'C b b+c
a ac a (b − c)
Ma A' = − = (1). Din triunghiul
2 b + c 2(b + c)
dreptunghic AH a B şi AH a C rezultă
Fig. 258
AH a
2
= AB 2
− BH 2
= AC 2
− CH a
2

b 2 − c 2 = ( H a C + H a B )( H a C − H a B ) , b 2 − c 2 = a(a − 2 H a B) , de unde
a +c −b
2 2 2
b −c2 2
HaB = şi de aici M a H a = M a B − BH a = (2). Deoarece
2a 2a
a+b+c
BCa = CDa = p − b unde p = , (vezi „Cercul înscris într-un triunghi” ) rezultă
2
a b−c
M a Ca = M a B − BCa = − ( p − b) = (3). Din relaţiile (1), (2) şi (3)
2 2

105
Karl Feuerbach (1800-1834) – matematician german, contribuţii importante în geometrie
106
Nichita Stănescu (1933 – 1983) – eseist, poet român, ales postum membru al Academiei Române
256
rezultă M a Ca2 = M a H a ⋅ M a A ' (4). Egalitatea (4) arată că punctul H a (care aparţine
cercului lui Euler al triunghiului ABC ) se transformă prin inversiunea de centru M a şi
raport M a Ca2 în punctul A ' . Prin aceasta inversiune, cercul lui Euler (fără punctul M a ) se
transformă într-o dreaptă d antiparalelă cu BC în raport cu A ce trece prin A ' (vezi „Cercul
lui Euler”). Dreapta d este a doua tangentă comună interioară a cercului înscris şi A –
exînscris. Prin această inversiune cercul înscris se transformă în el însuşi deoarece modulul
inversiunii este egal cu puterea polului inversiunii faţă de cercul considerat.
Deoarece M a Ca = M a Da rezultă că şi cercul A – exînscris se transformă în el
însuşi. Dreapta d fiind tangentă cercului înscris şi A –exînscris (invariante în inversiunea
considerată) rezultă că şi cercul lui Euler ar fi tangent acestor cercuri în punctele ϕ şi ϕ a
(punctele de intersecţie dintre dreapta d şi cercurile inverse şi A – exînscris). Analog se
arată că cercul lui Euler este tangent cercurilor exînscrise corespunzând vârfurilor B şi C.
Soluţia 2. Fie C ( I , r ) cercul înscris în triunghiul ABC (Fig. 259). Utilizăm teorema lui
c b a
Casey, considerând cercurile ( M a , M b , M c ,C) obţinem: tM a M b = , t AM c = , tM b M c = ,
2 2 2
a b−c b a−c c b−a
t M a = − ( p − b) = , tM b = − ( p − c) = , tM c = − ( p − a ) = (unde
2 2 2 2 2 2
prin distanţa tangenţială tij dintre cercurile Ci şi C j înţelegem lungimea tangentei comune
exterioare duse la cele două cercuri, cele două cercuri aflându-se de aceeaşi parte a
tangentei). Pentru ca cercul

ϕ
Hc
Mb
Mc
Fig. 259

Hb

B Ma Ha C

înscris C şi cercul medial să fie tangente trebuie să demonstrăm că pentru o combinaţie a


semnelor + şi – rezultă ± c(b − a) ± a(b − c) ± b(a − c) = 0 , ceea ce este evident. Din
teorema lui Casey rezultă că există un cerc care trece prin M a , M b , M c şi C. Cum cercul
circumscris triunghiului median este cercul lui Euler urmează ca cercul celor nouă puncte şi
C sunt tangente.
257
Soluţia 3. Fie M c mijlocul laturii AB, H c piciorul înălţimii din H, O9 centrul cercului lui
Euler al triunghiului ABC, DE diametrul perpendicular pe AB, F şi K mijloacele
DE
segmentelor HD respectiv HE (Fig. 260). Deoarece KF = = R şi cum O9 ∈ KF
2

E
A
K
X
L
KX ϕ
ϕ

O O9 I O9 I
H
Y Hc
B C
Mc S F M Y
F

D
Fig. 260

rezultă că KF este diametru în cercul lui Euler al triunghiului ABC, deci m( KM c F ) = 90°.
Fie XY ⊥ AB, XY diametru în cercul înscris în triunghiul ABC (Y ∈ AB) şi
XL ⊥ MK ( L ∈ KF ). Atunci, LM = XY = 2r unde {M } = KF ∩ AB. Drepta FM c este
dreapta lui Simson a punctului D şi este perpendiculară pe dreapta CD în punctul S (vezi
„Dreapta lui Simson”). Avem H c CB ≡ SM c M ≡ DIY ≡ M c DS (= α ). Fie
{T } = FY ∩ KX . Arătăm că m( KTY ) = 90°, deci cercurile de diametre KF şi XY – adică
cercul lui Euler şi cercul înscris în triunghiul ABC – sunt tangente în ϕ . Din
M cY = DI sin α , YH c = IC sin α rezultă M cY ⋅ YH c = DI ⋅ IC ⋅ sin 2 α = 2 Rr sin 2 α ; dar
R sin α = FK sin α = M c F sin α = MF , de unde M c Y ⋅ YH c = 2r ⋅ MF = LM ⋅ MF . Din
2 2

puterea unui punct M faţă de cercul lui Euler rezultă


MM c ⋅ MHc = MM c = MF ⋅ MK = MF ( KL + LM ) = MF ⋅ KL + MF ⋅ LM = MF ⋅ KL + M cY ⋅ YHc (1).
2

Dar M H c
2
− MY 2
= (M H c − M Y )( M H c + M Y ) = YH c ⋅YM c, deci
MH = MM = MY + YH c ⋅ YM c (2).
2
c
2
c
2
Din relaţiile (1) şi (2) rezultă
LX MF
MY 2 = MF ⋅ KL = LX 2 , adică = relaţie care arată că K ϕ ⊥ FY , deci cercul lui
KL MY
Euler şi cercul înscris în triunghiul ABC sunt tangente în punctul ϕ .
OI 2 + IH 2 OH 2
Soluţia 4. Teorema medianei aplicată în triunghiul OIH ne dă: IO92 = −
2 4
R 2 − 2 Rr + 2r 2 − 2rh R R 2 − 4rh R
sau IO92 = − , unde rh este raza cercului înscris în
2 4
triunghiul ortic al triunghiului ABC (vezi „Cercul înscris” şi „Cercul circumscris”), şi de
258
R 2 − 4 Rr + 4r 2 ( R − 2r ) 2 R R
aici rezultă că IO92 = = , deci O9 I = − r . Cum este raza
4 4 2 2
cercului Euler rezultă cercul lui Euler şi cercul înscris sunt tangente interior.

Observaţii:
1) Punctele ϕ , ϕa , ϕb , ϕc de tangenţă dintre cercul lui Euler şi cu cercurile tritangente se
numesc punctele lui Feuerbach ale triunghiului ABC.
2) Într-un triunghi ABC se duce cea de-a doua tangentă interioară a cercului înscris cu
fiecare cerc exînscris (primele tangente fiind laturile triunghiului). Dreptele ce unesc
punctele de contact ale acestor trei tangente cu mijloacele laturilor corespunzătoare trec
prin punctele lui Feuerbach.

1) Dreptele care unesc punctelele lui Feuerbach ale cercurilor exînscrise cu punctul lui
Feuerbach al cercului înscris trec prin piciorul bisectoarei situate pe laturile respective.
Demonstraţie. Piciorul bisectoarei interioare a unghiului BAC - punctul A ' - este centrul de
omotetie inversă dintre cercurile înscris şi A - exînscris; punctul lui Feuerbach ϕ este
centrul de omotetie directă dintre cercurile lui Euler şi cercul înscris, iar ϕa cetrul de
omotetie inversă între cercul lui Euler şi cercul A - exînscris, deci punctele A ' , ϕ şi ϕa
sunt coliniare.

Triunghiul lui Feuerbach ϕaϕbϕc este triunghiul a cărui vârfuri sunt punctele de tangenţă
dintre cercul celor nouă puncte cu cercurile exînscrise unui triunghi ABC.

2) Cercul ce trece prin picioarele bisectoarelor interioare ale unui triunghi conţine
punctul lui Feuerbach al triunghiului.
Demonstraţie. Vom arăta că triunghiul determinat de picioarele bisectoarelor este asemenea
şi omologic cu triunghiul lui Feuerbach.Vom utiliza în demonstraţia teoremei două leme:
Lema 1. Cercul C (O, R ) este tangent exterior cercurilor C1 (O1 , r1 ) şi C2 (O2 , r2 ) în punctele
A , respectiv B. Dacă A1 şi B1 sunt punctele de tangenţă ale tangentei exterioare comune
R
cercurilor C1 şi respectiv C2 , atunci AB = ⋅ A1 B1 .
( R + r1 )( R + r2 )
Demonstraţie. Teorema cosinusului aplicată în triunghiurile AOB şi O1OB (Fig. 261)
2 R 2 − AB 2 AB 2
ne dă: cos AOB = 2
= 1− ,
2R 2R 2
1 2 = (R + r1) + (R + r2 ) − 2(R + r1)(R + r2 ) ⋅ cos(OOO
2 2 2
OO 2)
1 O
Din relaţiile precedente rezultă: R B O2
2
 AB  r2
O1O22 = (r1 − r2 ) 2 + ( R + r1 )( R + r2 ) ⋅   . Din R
 R 
A
trapezul A1 B1O2 O1 avem: B1
O1 r1
OO22 = (r1 − r2 ) 2 + A1 B12 , de unde rezultă
concluzia.
A1
Fig. 261

259
Lema 2. Fie a, b, c lungimile laturilor triunghiului
A
ABC şi C (O, R ) cercul circumscris triunghiului
ABC. Dacă ( I a , ra ) este A -cercul exînscris, iar B1
şi C1 picioarele bisectoarelor interioare ale B1
C1 O
abc R ( R + 2ra ) I C
unghiurilor B şi C, atunci B1C1 = .
(a + b)(a + c) ⋅ R B
Demonstraţie. Fie I a B2 ⊥ AC , B2 ∈ AC şi C2 B2
I a C2 ⊥ AB , C2 ∈ AC , OQ ⊥ I a B2 , Q ∈ I a B2 ,
P
a+b+c Q
OP ⊥ I a C2 , P ∈ I a C2 , AB2 = AC2 = =p Ia
2
c a+b
(Fig. 262). Atunci, OP = p − = şi
2 2
b a+c Fig. 262
OQ = p − = . Din teorema bisectoarei
2 2
bc cb AB1 a + b OP
rezultă: AB1 = , AC1 = , de unde: = = . Cum POQ = C2 AB2
a+c a+b AC1 a + c OQ
B1C1 AB1 2bc
rezultă că triunghiurile AB1C1 şi OPQ sunt asemenea şi = = (∗) .
PQ OP (a + c)(a + b)
Ţinând cont că punctele O, P, Q, I a sunt pe cercul de diametru OI a , din teorema sinusurilor
a
rezultă PQ = OI a ⋅ sin POQ = OI a ⋅ sin A = OI a ⋅ care împreună cu (∗) dă:
2R
abc
B1C1 = OI a ⋅ . Utilizând relaţia lui Euler OI a2 = R ( R + 2ra ) rezultă.
R(a + c )(a + b)
abc R ( R + 2ra )
B1C1 = . ϕ
R(a + c)(a + b)
C1 B1
Demonstraţia teoremei. Fie ϕ punctul lui
Feuerbach al triunghiului ABC şi O9 centrul ϕc ϕb
cercului lui Euler. Fie ϕ a , ϕb , ϕc punctele de
tangenţă al cercului lui Euler al triunghiului
ABC cu cercurile sale exînscrise şi X , Y
punctele de tangenţă ale cercurilor A - A1
exînscris şi B – exînscris cu latura AB. Avem:
a +b + c a +b +c
ZY = AY + BX − AB = + −c = a +b
2 2 ϕa
Din lema 1, rezultă :
R Fig. 264
( a + b) ⋅
2 ( a + b) R
ϕ aϕb = =
R  R  ( R + 2ra )( R + 2rb )
 + ra  + rb 
 2  2 

260
abc R ( R + 2rc
Din lema 2 rezultă A1 B1 = (Fig. 264).
(c + a)(c + b) R

Ic X

A
ϕc ϕ
Ib
ϕb
I
B
ϕa C

Y Ia

Fig. 263

A1 B1 abc R ( R + 2ra )( R + 2rb )( R + 2rc )


Atunci, = . Din simetria relaţiei precedente rezultă
ϕ aϕb (a + b)(c + a )(b + c) R 2
A1 B1 B1C1 C1 A1
că: = = ,adică triunghiurile A1 B1C1 şi ϕ aϕbϕc sunt asemenea (1). Arătăm
ϕ aϕb ϕbϕc ϕ cϕ a
O9ϕ R IB1 r Iϕ 2r
că punctele ϕ , B1 şi ϕb sunt coliniare. Din faptul că = , = , b b = b
ϕ I 2r B1 I b rb ϕb O9 R
O9ϕ IB1 I bϕb
rezultă : ⋅ ⋅ = 1 şi din reciproca teoremei lui Menelaus rezultă că punctele
ϕ I B1 I b ϕb O9
ϕ , B1 şi ϕb sunt coliniare. Analog se arată că punctele ϕ , C1 şi ϕc şi ϕ , A1 şi ϕ a sunt
coliniare, ceea ce arată că triunghiurile A1 B1C1 şi ϕ aϕbϕc sunt omologice (2). Din relaţiile
(1) şi (2) rezultă m(C1ϕ B1 ) + m(C1 A1 B1 ) = m(ϕcϕϕb ) + m(ϕcϕ aϕb ) = 180° adică ϕ aparţine
cercului circumscris triunghiului A1 B1C1 .

261
3) În triunghiul ABC fie Ca , Cb , Cc punctele de contact ale cercului înscris cu laturile
BC , AC, respectiv AB, X şi Y punctele de intersecţie dintre paralela dusă prin A la BC
cu dreptele Ca Cb , respectiv Ca Cc . Dreapta lui Euler a triunghiului Ca XY trece prin
punctul lui Feuerbach al triunghiului ABC.
Demonstraţie. Vom arata mai întâi că punctele A, Cb , Cc , X ', Y ', I (Fig. 265) sunt conciclice
(unde X ' şi Y ' sunt mijloacele segmentelor Ca X , respectiv CaY ), ele aparţinând cercului
celor nouă puncte al triunghiului Ca XY . Avem, AXCb ≡ CCaCb ≡ CCbCa ≡ ACb X de unde
AX ≡ ACb şi analog AY ≡ ACc .
Y A
Cum ACb ≡ ACc rezultă X
AY ≡ AX , deci A este mijlocul
segmentului XY . Cercul celor nouă H'
puncte al triunghiului Ca XY Cb
conţine punctele A, X ', Y ' (fiind Y '
Cc
mijloacele laturilor X'
triunghiului Ca XY ). Din I
AY = AX = ACb rezultă că Cb
este piciorul înălţimii din X pe
YCa ; analog Cc este piciorul B Ca
C
înălţimii din X pe YCa , deci Fig. 265
punctele Cb şi Cc aparţin cercului
celor nouă puncte al triunghiului Cc XY . Fie H ' ortocentrul triunghiului Ca XY . Atunci,
H ' Cb Ca ≡ H ' Cc Ca , deci punctul H ' aparţine cercului înscris în triunghiul ABC şi mai
mult este diametru în acest cerc,
Y A adică I este mijlocul
X segmentului H ' C , ceea ce arată
O' a

H ' O9' că I aparţine cercului lui Euler al


triunghiului Ca XY .
Cb
D M Demonstraţia teoremei. Fie
Cc O, H şi O9 centrul cercului
I O
H O9 circumscris, ortocentrul şi
I'
centrul cercului lui Euler al
triunghiului ABC ( O9 este
B mijlocul segmentului HO ) şi d
Ca Da C dreapta lui Euler a triunghiului
Ca XY . Fie O9 O9' IH a , O9' ∈ d şi
J d I AH = {H '} , iar O ' punctul
de intersecţie dintre paralela prin
Fig. 266 O la IH ' cu dreapta d. Cum
AD IH ' şi M este mijlocul
lui AI rezultă că AD = IH ' = r
(raza cercului înscris în ABC ).
Ia Dacă M a este mijlocul laturii
BC , atunci AH = 2OM a . Fie
262
I a centrul cercului exînscris corespunzător laturii BC şi ra – raza sa. Cum punctele A , I
şi I a sunt coliniare (vezi „Cercuri exînscrise”), atunci J, punctul de intersecţie dintre AIa cu
cercul circumscris triunghiului ABC , este mijlocul arcului BC (Fig. 266). Fie I '
simetricul lui I faţǎ de O . Deoarece I ' I a trece prin punctul Da de tangenţǎ a cercului
exînscris corespunzǎtor laturii BC . Din asemǎnarea triunghiurilor MAD şi MJO ' ,
JO ' MO '
respectiv MIH ' cu MJO ' avem = , de unde rezultă că
AD MD
MO ' MO ' MJ 2MJ AI r
JO ' = IH '⋅ =r⋅ = r⋅ =r⋅ = r ⋅ a = r ⋅ a = ra În trapezul HOO ' D avem:
MD MH ' MI 2MI AI r
2 O 9 O 9' = O O ' + H D = ( J O ' − R ) + ( H A − D A ) = ra − R + 2 O M a − r =
ra − ( R + r ) + Ca I + Da I ' , de unde 2O9 O9' = ra − ( R + r ) + r + (2 R − ra ) = R , rezultă că
O9 O9' = R / 2 , adică O9' aparţine cercului lui Euler al triunghiului ABC . Cum O9 O9' IH a ,
dreptele O9' H a şi O9 I se intersecteazǎ în centrul de asemǎnare al cercurilor înscris,
respectiv al lui Euler al triunghiului ABC . Dar cele două cercuri sunt tangente interior în
punctul lui Feuerbach care este centrul de asemǎnare pentru cele două cercuri. Astfel,
dreapta lui Euler a triunghiului Ca XY conţine punctul lui Feuerbach al triunghiului ABC.

Observaţii :
1) Vom nota triunghiul Ca XY cu Ta . Analog cu Ta se construiesc triunghiurile Tb şi Tc .
Punctul lui Feuerbach este punctul de intersecţie dintre dreptele lui Euler corespunzătoare
triunghiurilor Ta , Tb şi Tc .
P
2) Punctul H ' este punctul antipodal al punctului
Ca al triunghiului ABC.
3) Centrul cercului lui Euler al triunghiului Ca XY
este punctul M, mijlocul segmentului IA . Q
4) Dreapta MH ' este dreapta lui Euler a
triunghiului Ca XY .

4) Punctul lui Feuerbach ϕa de pe cercul A - A


exînscris se află pe bisectoarea unghiului A a
triunghiului ABC dacă m( A) = 60° sau
m( B ) = m(C ) . A"
B Ma C
Demonstraţie. Fie H a piciorul înalţimii din A, I a Ha T
A1 ϕa Da Db
centrul cercului A - exînscris şi Da , Db , Dc punctele
de tangenţă ale acestuia cu dreptele BC, CA
respectiv AB , A ' punctul diametral opus lui Da în
cercul A - exînscris { A1} = AA '∩ BC , M a mijlocul Dc Ia
laturii BC, A" mijlocul segmentului
AI a , {Q} = A ' A "∩ AH a , T intersecţia dintre BC şi
A'
tangenta în ϕa la cercul A - exînscris,
{P} = I aT ∩ AH a , unde {ϕa } = A ' A "∩ C ( I a , ra ) Fig. 267
263
(Fig. 267). Deoarece T ϕ a = TDa rezultă TI a ⊥ Daϕ a şi cum Daϕ a ⊥ ϕ a A '
rezultă A ' Q I a P deci patrulaterul A ' QPI a este paralelogram, deci
QP ≡ A ' I a ≡ I a A1 .Cum I a Da QP rezultă că I a Da PQ este paralelogram Da P QI a (1).
Cum A ' I a AQ şi A " este mijlocul segmentului A ' I a rezultă că patrulaterul QAA ' I a
este paralelogram,deci I a Q AA ' (2). Din relaţiile (1) şi (2) rezultă Da P AA ' . Deoarece
punctele Da şi A1 sunt izotomice rezultă că I a Q trece prin punctul M a . Din asemănarea
TM TI a TDa
triunghiurilor TM a I a şi TPDa , respectiv TDa I a şi TH a P rezultă = = şi de
TDa TP TH a
aici TDa2 = TH a ⋅ TM a = T ϕa2 , adică T este pe axa radicală a cercului lui Euler a triunghiului
ABC şi a cercului A -exînscris, această axă este tangenta în ϕa la cercul A-exînscris , ceea
ce arată că punctul ϕa de intersecţie al cercului A-exînscris cu dreapta A ' A " este punctul
lui Feuerbach de pe cercul A - exînscris. Punctul lui Feuerbach se află pe bisectoarea A ' I a
AI a
dacă şi numai dacă ϕa coincide cu A " ceea ce este echivalent cu AA " = = ra . Din
2
ra ra A 1
triunghiul ADc I a rezultă AI a = , de unde ra = , adică sin = , deci
A A 2 2
sin 2sin
2 2
m( A) = 60° . Dacă dreptele A ' A" şi AH a coincid – adică triunghiul ABC este isoscel,
atunci punctele Da , M a şi A " coincid cu ϕa şi reciproc.

5) Punctul ϕ al lui Feuerbach este ortopolul dreptei OI în raport cu triunghiul ABC.


Demonstraţie. Cercul înscris în triunghiul ABC conţine ortopolul dreptei OI (vezi
„Ortopolul unei drepte). Deoarece ortopolul unui diametru al cercului circumscris unui
triunghi ABC aparţine cercului lui Euler al triunghiului ABC rezultă că ortopolul dreptei OI
în raport cu triunghiul ABC aparţine atât cercului înscris cât şi cercului lui Euler al
triunghiului ABC, deci ortopolul dreptei OI este punctul lui Feuerbach ( ϕ ) triunghiului
ABC.

6) Distanţele de la punctul lui Feuerbach corespunzător unui triunghi ABC la picioarele


înălţimilor triunghiului sunt egale, respectiv, cu perpendicularele coborâte din vârfurile
triunghiului pe dreapta OI.
Demonstraţia rezultă din faptul că punctul ϕ al lui Feuerbach este ortopolul dreptei OI în
raport cu triunghiul ABC, iar distanţa dintre ortopolul unui diametru al cercului circumscris
şi piciorul unei înălţimi este egală cu distanţa între vârful din care pleacă această înălţime şi
vârful considerat (vezi „Ortopolul unei drepte”).

7) Distanţele de la punctul lui Feuerbach corespunzător unui triunghi ABC la vârfurile


triunghiului sunt egale, respectiv, cu distanţele de la picioarele înălţimilor la proiecţiile
vârfurilor pe dreapta OI.
Demonstraţia rezultă din faptul că punctul ϕ al lui Feuerbach este ortopolul dreptei OI în
raport cu triunghiul ABC, iar distanţa între un vârf al triunghiului ABC şi ortopolul unui
diametru al cercului circumscris este egală cu distanţa între proiecţiile aceluiaşi vârf pe
latura opusă şi pe diametru (vezi „Ortopolul unei drepte”).

264
8) Punctul lui Feuerbach ϕ al triunghiului ABC este punctul anti - Steiner al dreptei IO
în raport cu triunghiul median al triunghiului ABC.
Demonstraţie. Vezi „Punctul anti-Steiner”.

9) Dreapta lui Simson a punctului lui Feuerbach ϕ al triunghiului ABC în raport cu


triunghiul median al acestuia este paralelă cu dreapta OI.
Demonstraţie. Deoarece punctul lui Feuerbach al triunghiului ABC este ortopolul dreptei
OI, dreapta lui Simson a punctului ϕ în raport cu triunghiul median al triunghiului ABC se
află la egală distanţă de punctul ϕ şi dreapta OI, deci dreapta lui Simson a punctului ϕ este
paralelă cu OI.

10) Fie ϕ punctul lui Feuerbach al triunghiului ABC şi M a , M b , M c mijloacele laturilor


BC, AC respectiv AB. Una din distanţele ϕ M a , ϕ M b , ϕ M c este egală cu suma celorlalte
două.
Demonstraţie. Fără a restrânge generalitatea presupunem A
că b > c > a. Fie P punctul de intersecţie dintre ϕ M a şi
cercul înscris în triunghiul ABC şi Ca punctul de contact ϕ
al cercului înscris cu latura BC. Atunci,
I
(b − c ) 2
M ϕ R
M a P ⋅ M aϕ = M a Ca2 = , iar a = , deoarece
4 Pϕ 2r P
ϕ este centrul de asemănare dintre cercul medial şi B Ca Ma C
cercul înscris în triunghiul ABC. Astfel,
M aϕ R (b − c) R Fig. 268
= , de unde M aϕ = , distanţa
M a P R − 2r 2 R − 2r
(c − a ) R
M aϕ este proporţională cu diferenţa b − c . Analog, se arată că M bϕ = şi
2 R − 2r
(b − a) R
McF = . Evident, M aϕ + M bϕ = M cϕ .
2 R − 2r

11) Dreapta lui Simson a punctului lui Feuerbach al triunghiului ABC în raport cu
triunghiul ortic H a H b H c al triunghiului ABC este paralelă cu OI.
Demonstraţie. Deoarece triunghiurile H a H b H c şi M a M b M c sunt triunghiuri S în cercul
median (vezi „Triunghiuri ortopolare”) cum dreapta lui Simson a unui punct în raport cu
triunghiurile S din aceeaşi familie păstrează aceeaşi direcţie rezultă concluzia.

265
12) Dreapta lui Simson a punctului lui Feuerbach în raport cu triunghiul de contact
Ca Cb Cc al triunghiului ABC este paralelă cu dreapta OI.
Demonstraţie. Fie Pa Pb Pc triunghiul determinat
A
de mijloacele arcelor H a M a , H b M b , H c M c ale
cercului medial. Triunghiurile Pa Pb Pc şi
Hb
MaMbMc sunt triunghiuri S deoarece Pb
Mc
1 Mb
m ( Pa M a ) = m( B) − m( C) ,
2 Pc
1 Hc
m ( Pb M b ) = m ( C ) − m ( A) ,
2
1
c c) =
m(PM m( A) − m( B) , deci suma algebrică a B Ha Ma C
2 Pa
măsurilor lor este egală cu zero. Deoarece Fig. 269
triunghiul de contact Ca Cb Cc este omotetic cu
triunghiul Pa Pb Pc (centrul de omotetie fiind
punctul lui Feuerbach ) rezultă că dreapta lui Simson a punctului ϕ în raport cu triunghiul
Ca Cb Cc este paralelă cu OI.

13) Dreapta lui Simson a punctului lui Feuerbach al triunghiului ABC în raport cu
triunghiul de contact Ca Cb Cc coincide cu dreapta lui Simson punctul lui Feuerbach al
triunghiului ABC în raport cu triunghiul median M a M b M c .
Demonstraţie. Deoarece dreapta lui Euler a triunghiului Ca Cb Cc este OI, ortocentrul
triunghiului Ca Cb Cc aparţine dreptei OI şi cum O este ortocentrul triunghiului median
M a M b M c , rezultă că dreapta lui Simson comună este paralelă cu OI şi trece la o distanţă
egală de punctul lui Feuerbach şi de dreapta OI.

14) Fie P mijlocul segmetului HI şi O centrul cercului lui Euler al triunghiului ABC.
Punctul lui Feuerbach ( ϕ ) şi centrul cercului înscris ( I ) al triunghiului ABC sunt
puncte inverse în cercul de centru O9 şi raza PO9 .
H
Demonstraţie. Deoarece punctele O9 , I şi ϕ sunt coliniare
rezultă: OI = 2 PO9 , OI 2 = 4 PO92 (1), iar din teorema lui P O9
R
Euler avem : OI 2 = R 2 − 2 Rr (2). Deoarece O9 I = − r şi
2 ϕ I O
R
O9ϕ = (3) rezultă: 4O9 P 2 = 4O9ϕ ⋅ O9 I , adică Fig. 270
2
O9 P 2 = O9ϕ ⋅ O9 I , de unde rezultă concluzia.

15) Fie A1 proiecţia vârfului A al triunghiului ABC pe dreapta OI şi ϕ punctul lui


Feuerbach corespunzător triunghiului ABC. Punctele ϕ şi A1 sunt simetrice faţă de
latura M b M c a triunghiului median.
Demonstraţia rezultă din teorema 11- „Ortopolul unei drepte”.

266
16) Fie A1 proiecţia vârfului A al triunghiului ABC pe dreapta OI, ϕ punctul lui
Feuerbach corespunzător triunghiului ABC şi H a piciorul perpendicularei duse din A
pe BC. Dreapta H a φ şi perpendiculara AA1 coborâtă din A pe OI sunt simetrice în
raport cu înălţimea AH a şi se intersectează pe latura M b M c a triunghiului median.
Demonstraţia rezultă din simetria punctelor A1 şi φ în raport cu latura M b M c .

17) Fie M a M b M c triunghiul median, H a H b H c triunghiul ortic, Ca Cb Cc triunghiul de


contact al unui triunghi ABC şi ϕ punctul lui Feuerbach corespunzător. Dreapta ϕ Ca
este bisectoarea unghiului H aϕ M a .

D Ma Ca Ha C
B
E
Fig. 271

Demonstraţie. Fie D punctul în care tangenta în ϕ la cercul lui Euler intersectează latura
BC şi E punctul de intersecţie dintre dreapta ϕ Ca cu cercul lui Euler. Atunci, Dϕ ≡ DCa ,
1
deci Dϕ Ca ≡ DCaϕ . Dar m( DCaϕ ) = [m(ϕ H a ) + m( EM a )] şi
2
1 1
m(DϕCa ) = m(ϕCa ) = [m(ϕHa ) + m(HaCa )] , de unde rezultă că m( EH a ) = m( H a Ca )] , deci
2 2
H aϕ Ca ≡ Caϕ M a , adică ϕ Ca este bisectoarea unghiului H aϕ M a .

267
18) Fie Ca Cb Cc triunghiul de contact al unui triunghi ABC , I centrul cercului înscris în
triunghiul ABC şi Ca' , Cb' , Cc' punctele diametral opuse punctelor Ca , Cb , Cc în cercul
înscris, iar A ', B ', C ' mijloacele segmentelor AI , BI , respectiv CI . Dreptele
A ' Ca' , B ' Cb' , C ' Cc' sunt concurente în punctul lui Feuerbach (ϕ )
Demonstraţie. Fie A " punctul de intersecţie dintre paralela dusă din Ca la AI şi înălţimea
 1 
AH a . Atunci, A " AI ≡ H a A " Ca  = [m( B) − m( C )]  (vezi „Drepte izogonale”). Cum
 2 

A ' C'a
ϕ
Cb
Cc A"

B Ha Ca Ma C
Fig. 272

1
m ( H a ϕ Ca ) =[ m( B) − m( C )] rezultă H a AI ≡ H aϕ Ca ≡ H a A " Ca , deci
2
patrulaterul ϕ H a Ca A " este inscriptibil ,de unde A "ϕ Ca ≡ A " H a Ca (= 90°) şi cum
Ca Ca' este diametru în cercul lui Euler rezultă că punctele ϕ , A ", Ca' sunt coliniare. Cum
A ", A ' şi Ca' sunt colinire rezultă că punctele ϕ , A ', Ca' sunt coliniare.

19) Consecinţă: Fie Ca' , Cb' , Cc' punctele diametral opuse vârfurilor Ca , Cb , respectiv Cc
ale triunghiului de contact al triunghiului ABC, în cercul înscris triunghiului ABC.
Cercurile de diametru Ca Ca' , Cb Cb' , Cc Cc' se intersecteaza în puntul lui Feuerbach al
triunghiului ABC.
Demonstraţia rezultă din teorema precedentă.

268
II.13. Teorema lui Desargues107
„Matematica a apărut şi se dezvoltă, printr-un continuu proces de modelare la nivelul Gândirii a fenomenelor lumii
fizice,Matematica servind, pe această cale, înţelegerii acestor fenomene.” - Aristotel108

Teorema lui Desargues


Punctele de intersecţie ale dreptelor omologe, a două triunghiuri omologe coplanare,
sunt coliniare.
Demonstraţie.
M

N
A'
A

O B
B'
C
C'

L
Fig. 273

Fie ABC şi A ' B ' C ' două triunghiuri coplanare astfel încât AA '∩ BB '∩ CC ' = {O} .
Teorema lui Menelaus aplicată triunghiurilor OBC , OCA, OAB şi transversalelor B ' C ' ,
LC B ' B C ' O MA C ' C A ' O
C ' A' respectiv A' B ' dă: ⋅ ⋅ =1, ⋅ ⋅ =1,
LB B ' O C ' C MC C ' O A ' A
NB A ' A B ' O
⋅ ⋅ = 1 , relaţii care prin înmulţire membru cu membru dau:
NA A ' O B ' B
LC NB MA
⋅ ⋅ = 1 şi conform teoremei lui Menelaus aplicată triunghiului ABC şi
LB NA MC
punctelor L, M , N rezultă că punctele L, M , N sunt coliniare.

Observaţie: Dreapta ce conţine punctele L, M , N se numeşte axa de omologie.


Triunghiurile ABC şi A ' B ' C ' se numesc omologice.

107
Gérard Desargues (1591-1661) – matematician francez , fondatorul geometriei proiective
108
Aristotel (384-322 î.e.n.) – filosof grec

269
Reciproca teoremei lui Desargues
Fie triunghiurile ABC şi A ' B ' C ' cu proprietatea că există punctele L, M , N astfel
încât {L} = BC ∩ B ' C ' , {M } = AC ∩ A ' C ' şi {N } = AB ∩ A ' B ' , iar dreptele AA ' şi BB '
nu sunt paralele. Doar punctele L, M şi N sunt coliniare, atunci dreptele AA ' , BB ' şi
CC ' sunt concurente.
Demonstraţie. Fie {O} = AA '∩ BB ' . Dar {N } = AB ∩ A ' B '∩ MN şi conform teoremei lui
Desargues, dreptele suport ale laturilor triunghiului LB ' B şi MAA ' se intersectează doar
câte două în trei puncte coliniare O , C şi C ' : {O} = AA '∩ BB ' , {C} = LB ∩ MA .,
{C '} = MA '∩ LB ' , deci O ∈ CC ' , adică dreptele AA ', BB ', CC ' sunt concurente în
punctul O .

Fie triunghiurile omologice ABC şi


A ' B ' C ', P centrul lor de omologie, M un Y
punct în planul triunghiului ABC,
M 1 M 2 M 3 triunghiul cevian al lui M în
raport cu triunghiul ABC şi Q
{M 1} = PM 1 ∩ B ' C ',{M 2 } = PM 2 ∩ C ' A ',
Z
{M 3 } = PM 3 ∩ A ' B '. Dreptele PM, A ' M 1' , A'
B ' M 2' şi C ' M 3 ' sunt concurente. A
M '3
Demonstraţie. Fie {X } = BC ∩ B ' C ', M2 M '2
B M3
{Y } = CA ∩ C ' A ', {Z } = AB ∩ A ' B '. Din P
M1 M M' B'
teorema lui Desargues rezultă că punctele X, '
Y, Z sunt coliniare. Din reciproca teoremei C M 1

lui Desargues rezultă că triunghiurile AA ' Y C'


şi M 1 M 1 ' X sunt omologice, PCC ' fiind
axa lor de omologie, deci dreptele X
AM 1 , A ' M 1 ' şi YX sunt concurente într-un
Fig. 274
punct Q. Analog, triunghiurile CC ' X şi
M 3 M 3 ' Z sunt omologice, PB ' B fiind axa
lor de omologie, deci dreptele CM 3 , C ' M 3 '
şi XZ sunt concurente într-un punct R . Fie
{M '} = A ' Q ∩ C ' R. Triunghiurile CC ' R şi R
AA ' Q sunt omologice, centrul de omologie
fiind punctul Y iar axa de omologie este PMM '. Atunci dreptele C ' M 3 ', A ' M 1 ' şi PM sunt
concurente şi analog A ' M 1 ', B ' M 2 ' şi PM sunt concurente de unde rezultă concluzia.

270
II.14. Teorema lui Döttl
„Lucrul cel mai uimitor este că raţionamentele matematice cele mai abstracte
sfârşesc prin a lărgi cunoaşterea noastră despre lume.” - Albert Einstein109

Fie triunghiul ABC şi A ' ∈ ( BC ), B ' ∈ (CA), C ' ∈ ( AB) astfel încât dreptele AA ', BB ', CC '
sunt concurente. Dacă A " ∈ ( B ' C '), B " ∈ (C ' A '), C " ∈ ( A ' B ') astfel încât dreptele
A ' A ", B ' B ", C ' C " sunt concurente, atunci dreptele AA ", BB ", CC " sunt concurente.
Demonstraţie. Fie { X } = AA "∩ BC ,{Y } = BB "∩ AC ,{Z } = CC "∩ AB (Fig. 275). Din
teorema lui Menelaus (pentru patrulatere) aplicată
patrulaterului BCB ' C ' şi punctelor coliniare A
BX CA B ' A " C ' A
X , A , A " , A avem: ⋅ ⋅ ⋅ = 1, Z A"
XC AB ' A " C ' AB
B'
BX AB ' AB A " C ' C'
de unde = ⋅ ⋅ (1). Analog,
XC AC ' AC A " B ' C" Y
CY BC ' BC B " A ' AZ CA ' CA C " B ' B"
= ⋅ ⋅ (2) şi = ⋅ ⋅
YA BA ' BA B " C ' ZB CB ' CB C " A '
B
(3). Deoarece dreptele AA ', BB ', CC ' respectiv A' X C
A ' A ", B ' B ", C ' C " sunt concurente, din teorema lui
A' B B 'C C ' A Fig. 275
Ceva rezultă: ⋅ ⋅ =1 şi
A 'C B ' A C ' B
A" B ' B " C ' C " A '
⋅ ⋅ = 1 (4). Înmulţind relaţiile (1), (2) şi (3) membru cu membru şi ţinând
A"C ' B " A ' C " B '
BX YC ZA
cont de relaţiile (4) rezultă ⋅ ⋅ = 1, iar din reciproca teoremei lui Ceva rezultă că
XC YA ZB
dreptele AA ", BB ", CC " sunt concurente.

II.15. Teorema lui Van - Aubel


„Învăţând matematica, înveţi sa gandeşti.” – Grigore Moisil110
Teorema lui Van-Aubel
Dacă AD, BE şi CF sunt trei ceviene concurente într-un punct P interior triunghiului
AP AF AE
ABC, atunci = + .
PD FB EC
AP A[ A P B ] A[ A P C ] A[ A P B ] + A[ A P C ]
Demonstraţie. Avem: = = = de unde rezultă că:
PD A[ B P D ] A[ P C D ] A[ B P D ] + A[ P C D ]

109
Albert Einstein (1879-1955) – fizician german, profesor universitar la Berlin şi Princeton, laureat al Premiului
Nobel
110
Grigore Moisil (1906-1973) – matematician român, profesor la Universitatea din Iaşi, membru al Academiei
Române
271
AP A[ APB] + A[ APC] A[ APB] A[ APC] AF A[ ACF] A[ APF] A[ ACF] − A[ APF] A[ APC]
= = + (1). Dar = = = = (2)
PD A[ BPC] A[ BPC] A[ BPC] FB A[ FCB] A[ FPB] A[ FCB] − A[ FPB] A[ BPC]
AE A[ APB ]
şi analog = (3). Din relaţiile (1), (2) şi (3) rezultă concluzia.
EC A[ BPC ]

1) Dacă D, E, F sunt mijloacele laturilor triunghiului ABC, atunci P este centrul de


greutate al triughiului ABC şi relaţia lui Van - Aubel devine AG = 2GD.

2) Dacă P este I, centrul cercului înscris triunghiului A


AF b AE c
ABC, atunci = , = , relaţia lui Van Aubel
FB a EC a
E
AI b + c
devenind = > 1, relaţie ce arată că I este mai P
ID a F
„aproape” de piciorul bisectoarei D, decât de vârful A.
B D C
3) Dacă P este H, ortocentrul triunghiului ABC, atunci
AF tgB AE tgC Fig. 276
= , = , relaţia lui Van Aubel devenind
FB tgA EC tgA
AH cos A
= .
HD cos B ⋅ cos C

4) Dacă P este punctul lui Lemoine K al triunghiului ABC, atunci


AF b 2 AE c 2 AK b 2 + c 2
= 2, = 2 , relaţia lui Van - Aubel devenind = .
FB a EC a KD a2

AF p − a AE p−a
5) Dacă P este punctul lui Gergonne Γ, atunci = , = , de unde
FB p − b EC p−c
AΓ a ( p − a)
= .
ΓD ( p − b)( p − c)

AF b 2 AE b 2
6) Dacă P este primul punct Brocard Ω , atunci = , = , de unde
FB c 2 EC a 2
AΩ a 2 b 2 + b 2 c 2
= .
ΩD a2c2

AF c 2 AE c 2
7) Dacă P este al doilea punct al lui Brocard Ω ' , atunci = , = şi relaţia lui
FB a 2 EC b 2
AΩ ' b 2 c 2 + a 2 c 2
Van Aubel devine = .
Ω'D a 2b 2

272
8) În triunghiul ABC fie cevienele AD, BE şi CF concurente într-un punct P astfel încât
k k k
BD  AB  AF  AC  AE  AB  AP AC k + AB k
=  , =  şi =  , k ∈ . Atunci: = .
DC  AC  FB  BC  EC  BC  PD BC k
Demonstraţie: Notăm cu a, b, c lungimile laturilor BC, CA respectiv AB. Din teorema lui
AP AF AE AC k + AB k
Van – Aubel rezultă că = + = .
PD FB EC BC k

Observaţii:
1) Dacă pe latura BC a triunghiului ABC se consideră un punct D astfel încât
k
BD  AB 
=  ,k ∈ atunci dreapta AD se numeşte ceviană de rang k.
DC  AC 
0
BD  AB 
2) i) Mediana AD este o ceviană de rang 0, (k = 0), deoarece =  = 1.
DC  AC 
1
BD  AB 
ii) Bisectoarea AD este o ceviană de rang 1, (k = 1), deoarece = .
DC  AC 
2
BD  AB 
iii) Simediana AD este o ceviană de rang 2, (k = 2) , deoarece =  .
DC  AC 
−1
BD  AB 
iv) Antibisectoarea AD este o ceviană de rang (−1), (k = −1), deoarece =  .
DC  AC 

9) Dacă P este punctul de concurenţă a trei ceviene de rang k şi M este un punct din
uuur uuur uuuur
uuur a k ⋅ MA + b k ⋅ MB + c k ⋅ MC
planul triunghiului ABC, atunci: MP = , (unde a, b, c sunt
ak + bk + c k
lungimile laturilor BC, CA respectiv AB).
Demonstraţie. Fie cevienele AD, BE şi CF ceviene de rang k . Din triunghiul MAD:
uuur uuur
uuur MA + λ MB AP
MP = , unde λ = . Conform teoremei
1+ λ PD A
uuur uuuu
r
AP b k + c k uuur a k ⋅ MA + (b k + c k ) MD
(1), = , deci MP = .
PD a k
ak + bk + ck E
k k M
BD  AB   c  P
În triunghiul MBC, = =
   , deci F
DC  AC   b 
uuur  c k uuuur B D C
uuuur MB +   MC uuuur uuuur
 b b k MD + c k MC
MD = = de unde Fig. 277
c
k
bk + ck
1+  
b
uuur k uuur k uuuur
uuur a ⋅ MA + b ⋅ MB + c ⋅ MC
k
MP = .
a k + bk + ck

273
Cazuri particulare:
uuur uuur uuuur
uuuur MA + MB + MC
1) Pentru k = 0, P ≡ G relaţia din teoremă devine MG = .
3
uuur uuur uuuur
uuur aMA + bMB + cMC
2) Pentru k = 1, P ≡ I , relaţia din teoremă devine: MI = .
a+b+c
3) Pentru k = 2, P ≡ K (punctul lui Lemoine), relaţia din teoremă devine:
uuur uuur uuuur
uuuur a 2 MA + b 2 MB + c 2 MC
MK = .
a2 + b2 + c2
4) Pentru k = −1, P ≡ Z (punctul de recurenţă al antibisectoarelor), relaţia din teoremă
uuur uuur uuuur
uuur bcMA + acMB + abMC
devine: MZ = .
ab + bc + ac

10) Fie P punctul de concurenţă a trei ceviene de rang k, M un punct din planul unui
a k MA2 + b k MB 2 + c k MC 2 a k b k c k (a 2 − k + b 2 − k + c 2 − k )
triunghi ABC. Atunci: MP 2 = − .
ak + bk + ck (a k + b k + c k ) 2
uuur uuur
Demonstraţie. Utilizând teorema precedentă avem: MP ⋅ MP = MP 2 =
uuur uuur uuur uuuur uuur uuuur 1
(a2k MA2 + b2k MB2 + c2k MC 2 + 2ak bk MA ⋅ MB + 2bk ck MB ⋅ MC + 2ak ck MA ⋅ MC) ⋅ k k k 2
(a + b + c )
uuur uuur MA2 + MB2 − AB2 1
Dar MA ⋅ MB = MA ⋅ MB ⋅ cos AMB = MA ⋅ MB ⋅ = (MA2 + MB2 − AB2 ) şi
2MA ⋅ MB 2
analoagele. Înlocuind în relaţia precedentă va da relaţia cerută.

Cazuri particulare:
1) Pentru k = 0, P ≡ G , relaţia din teoremă
MA2 + MB 2 + MC 2 a 2 + b 2 + c 2
devine: MG 2 = − (relaţia lui Leibniz).
3 9
aMA2 + bMB 2 + cMC 2 − abc
2) Pentru k = 1, P ≡ I , relaţia devine MI 2 = .
a+b+c
a 2 MA2 + b 2 MB 2 + c 2 MC 2 3a 2 b 2 c 2
3) Pentru k = 2, P ≡ K , relaţia devine MK 2 = − 2 .
a +b +c
2 2 2
(a + b 2 + c 2 ) 2
4) Pentru k = −1, P ≡ Z , relaţia devine
bcMA + acMB + abMC
2 2
abc(a + b + c )
2 3 3 3
MZ 2 = − .
ab + bc + ac (ab + bc + ac)2

11) În triunghiul ABC fie cevienele de ordin k AD, BF şi CE (k ∈ ) concurente într-un


punct P. Dacă M ∈ ( AB) şi N ∈ ( AC ) dreapta MN trece prin P dacă şi numai dacă:
MB k NC
bk ⋅ +c ⋅ = a k , (unde a, b, c sunt lungimile laturilor BC, CA, respectiv AB).
MA NA
Demonstraţie. Utilizând teorema transversalei în triunghiul ABC cu ceviana AD şi secanta
MB DC NC BD PD
MN rezultă ⋅ + ⋅ = (1). Din relaţia lui Van-Aubel avem:
MA BC NA BC PA

274
AE AF AP b k + c k AP PD ak DC b k
+ = adică = , deci = k . Din = rezultă
EB FC PD a k
PD AP b + c k
BD c k
DC bk BD ck MB k NC
= k şi = (2). Din relaţiile (1) şi (2) rezultă b k ⋅ +c ⋅ = ak .
BC b + c k
BC b + c
k k
MA NA
MB k NC
Reciproc, fie bk ⋅ +c ⋅ = ak . Fie {R} = MN ∩ AD. Atunci,
MA NA
k k
MB DC NC BD RD  b  MB  c  NC
⋅ + ⋅ = . Din ipoteză avem   ⋅ +  ⋅ = 1, adică
MA BC NA BC RA  a  MA  a  NA
AE MB AF NC
⋅ + ⋅ = 1. Din teorema lui Menelaus aplicată în triunghiul ABD şi
EB MA FC NA
AE AP DC AF AP BD
transversala EC rezultă = ⋅ şi analog = ⋅ . Atunci,
EB PD BC FC PD BC
MB DC NC BD PD PD RD
⋅ + ⋅ = , deci = , de unde rezultă că P ≡ R.
MA BC NA BC PA PA RA

Cazuri particulare:
MB NC
1) Dacă P ≡ G, atunci k = 0. Dreapta MN trece prin G dacă şi numai dacă + = 1.
MA NA
2) Dacă P ≡ I , atunci k = 1 . Dreapta MN trece prin I dacă şi numai dacă
MB NC
b⋅ + c⋅ = a.
MA NA
3) Dacă P ≡ K , atunci k = 2 . Dreapta MN trece prin punctul lui Lemoine al triunghiului
MB 2 NC
ABC dacă şi numai dacă b 2 ⋅ +c ⋅ = a2 .
MA NA
4) Dacă P ≡ Z (punctul de concurenţă al antibisectoarelor), atunci k = −1 . Dreapta MN
1 MB 1 NC 1
trece prin Z dacă şi numai dacă ⋅ + ⋅ = .
b MA c NA a

12) Orice ceviană de ordinul k este locul geometric al punctelor pentru care distanţele la
două laturi ale triunghiului sunt proporţionale cu acele laturi la puterea (k-1).
Demonstraţie. Fie AD o ceviană de ordinul k, D ∈ ( BC ), M ∈ ( AD) , iar M 1 , M 2 , M 3
proiecţiile lui M pe AC, AB, respectiv BC. Notăm cu x, y, z lungimile segmentelor
MM 1 , MM 2 şi MM 3 , iar cu ϕ1 şi ϕ 2 măsurile unghiurilor
k
BD  c  A
BAD , respectiv CAD. Avem: =  ,
DC  b 
M3 ϕ1 ϕ2 M2
A[ BAD ] BD AD ⋅ c ⋅ sin ϕ1 c sin ϕ1
= = = ⋅ , de unde
A[ DAC ] DC AD ⋅ b ⋅ sin ϕ 2 b sin ϕ 2 z y
k −1
M
sin ϕ1  c  x
=  (1) (Fig. 278). Din triunghiurile
sin ϕ 2  b  B M1 D C
z
dreptunghice AM 2 M şi AM 3 M rezultă sin ϕ1 = şi Fig. 278
AM

275
y sin ϕ1 z z y
sin ϕ 2 = , de unde = (2). Din relaţiile (1) şi (2) rezultă k −1 = k −1 . Analog,
AM sin ϕ 2 y c b
z y z y z
se arată că k −1
= k −1
, de unde k −1
= k −1
= k −1
.
a b a b c

13) Fie AD, BE, CF ceviene de ordinul k în triunghiul ABC, {M } = AD ∩ BE ∩ CF şi x,


y, z proiecţiile lui M pe laturile BC, CA, respectiv AB. Atunci,
z y z 2 A[ ABC ]
k −1
= k −1 = k −1 = k .
a b c a + bk + ck
z y z ax by cz ax + by + cz 2 A[ ABC ]
Demonstraţie: Din k −1 = k −1 = k −1 rezultă k = k = k = k = k .
a b c a b c a +b +c
k k
a + bk + ck

II.16. Teorema lui Descartes 111

„Esenţa Matematicii constă în libertatea sa”- Georg Cantor112

1) Fie triunghiul ABC şi I centrul cercului înscris în acest triunghi. Există doar trei
cercuri care să aibă centrele în vârfurile triunghiului, sunt tangente exterioare două câte
două şi I are puteri egale faţă de cele trei cercuri.
Demonstraţie. Fie A ', B ', C ' punctele de tangenţă dintre cele trei cercuri (Fig. 279).
Evident, punctele de tangenţă sunt situate pe laturile triunghiului. Notând
AB ' = AC ' = r1 , BA ' = BC ' = r2 şi CA ' = CB ' = r3 obţinem
r2 + r3 = a, r3 + r1 = b, r1 + r2 = c , de unde r1 = p − a, r2 = p − b, r3 = p − c ( p = r1 + r2 + r3
fiind semiperimetrul triunghiului ABC) relaţii care arată că cercurile sunt unic determinate.
Punctele A ', B ', C ' coincid cu punctele de tangenţă ale cercului înscris cu laturile
triunghiului (vezi „Cercul înscris”), deci I are puteri egale faţă de cele trei cercuri.

2) Consecinţă: Dacă r este raza cercului înscris în triunghiul ABC, atunci


r1r2 r3
r2 = .
r1 + r2 + r3
Demonstraţie. Din relaţiile A[2ABC ] = p ( p − a)( p − b)( p − c) = (r1 + r2 + r3 )r1r2 r3 şi A[ ABC ] = rp
rezultă concluzia.

Fie patru cercuri C1 , C2 , C3 , C4 tangente două câte două în şase puncte distincte. Fiecărui
1
cerc îi corespunde numărul ε i = , i = 1, 4 , unde Ri reprezintă raza cercului Ci , i = 1, 4.
Ri
Dacă toate cercurile sunt tangente exterior, atunci numerele ε i se consideră cu semnul „+”,
(Fig. 279), iar dacă trei dintre aceste cercuri sunt tangente interior celui de-al patrulea cerc,

111
René Descartes (1596-1650) – matematician şi filosof francez, contribuţii în geometrie
112
Georg Cantor (1845-1918) – matematician german, creator al teoriei mulţimilor

276
atunci numărul ε i corespunzător cercului ce are raza de lungime maximă este egală
1
cu ε i = − (Fig. 280) .
Ri

C'
A p−a B B
p−c
p−b p−b
C A p
B'
A'
p−c
Fig. 279 Fig. 280
C

Teorema lui Descartes


2(ε12 + ε 22 + ε 32 + ε 42 ) = (ε1 + ε 2 + ε 3 + ε 4 )2 .
Demonstraţie. Cercurilor C1 , C2 , C3 , C4 le corespund cercurile C1' , C2' , C3' , C3' , tangente două
1
câte două în şase puncte (Fig. 281) şi notăm cu ηi numerele , i = 1, 4 , unde Ri' sunt
Ri'
razele cercurilor Ci' , i = 1, 4 . Cercul C1' - de exemplu – are trei puncte de tangenţă în comun
cu cercurile C2 , C3 , C4 ; analog se definesc cercurile C2' , C3' , C3' . Dacă A, B, C sunt centrele
cercurilor C1 , C2 , respectiv C3 , atunci C4' este cercul înscris sau un cerc exînscris al
triunghiului ABC . Corespunzător primului caz (Fig.279) avem:
1 1 1 1
ε1 = ,ε2 = ,ε3 = ,η 4 = − , iar corespunzător celui de-al doilea caz
p−a p −b p−c r
1 1 1 1 1 1 1
avem: ε1 = − , ε 2 = , ε3 = ,η4 = − .Atunci: ε2ε3 + ε1ε2 + ε1ε3 =  + +  ⋅ ε1ε2ε3 =
p p −c p −b ra  ε1 ε2 ε3 
( p − a ) + ( p − b) + ( p − c ) p 1
= = 2 = η 42 (în primul caz); sau
( p − a )( p − b)( p − c ) ( p − a)( p − b)( p − c) r

277
C1

C3
'
C4'
C3
C2'
C4
4
'
C1

C2

Fig. 281

p −b−c p−a
ε 2ε 3 + ε1ε 2 + ε1ε 3 = = = η 42 , deci ε 2ε 3 + ε1ε 2 + ε1ε 3 = η 42 .
− p ( p − c)( p − b) p ( p − b( p − c)
Analog se arată că η 2η3 + η3η1 + η1η 2 = ε 42 , iar permutând indicii se obţine altă relaţie
4 4
derivată. Atunci, (ε1 + ε 2 + ε 3 + ε 4 ) 2 = ε12 + ε 22 + ε 32 + ε 42 + 2 ∑ ε ε = ∑ ε + ∑η
1≤ i < j ≤ 4
i j
i =1
i
2

i =1
i
2
=

(η1 + η 2 + η3 + η 4 )2 (datorită simetriei relaţiilor precedente), de unde


ε1 + ε 2 + ε3 + ε 4 = η1 +η2 +η3 +η4 > 0 , (ε1 + ε 2 + ε3 − ε 4 )(ε1 + ε 2 + ε3 + ε 4 ) = (ε1 + ε 2 + ε3 ) − ε 42 = 2

ε 12 + ε 22 + ε 32 − ε 42 − 2η 42 = (η 2η 3 + η 3η 4 + η 4η 2 ) + (η1η 3 + η 3η 4 + η 4η1 ) + (η1η 2 + η 2η 4 + η1η 4 ) −


(ηη
1 2 + η2η3 +η3η1 ) + 2η4 = 2(ηη
2
1 4 +η2η4 +η3η4 ) + 2η4 = 2η4 (η1 +η2 + η3 +η4 ) = 2η4 (ε1 + ε 2 + ε 3 + ε 4 )
2

de unde ε1 + ε 2 + ε 3 − ε 4 = 2η 4 . Analog se arată că


−ε1 + ε 2 + ε 3 + ε 4 = 2η1 , ε1 − ε 2 + ε 3 + ε 4 = −2η 2 , ε1 + ε 2 − ε 3 + ε 4 = −2η3 . Ridicând relaţiile
precedente la pătrat şi sumându-le rezultă: ε 12 + ε 22 + ε 32 + ε 42 = η12 + η 22 + η32 + η42 , de unde
4 4
2(ε12 + ε 22 + ε 32 + ε 42 ) = ∑ ε i2 + ∑ηi2 = (ε1 + ε 2 + ε 3 + ε 4 ) 2 .
i =1 i =1

Soluţia 2. Fie A,B,C,D centrele cercurilor C1 , C2 , C3 , respectiv C4 şi R1 , R2 , R3 , R4 razele


acestor cercuri. Semiperimetrul triunghiului BCD este egal cu R2 + R3 + R4 . Din teorema
cosinusului şi formula unghiului pe jumătate rezultă:
BDC 1 + cos BDC R ( R + R + R )
cos 2 = = 4 2 3 4
şi
2 2 ( R2 + R4 )( R3 + R4 )
1 − cos BDC R2 R3
sin 2 = = . Utilizând egalitatea
2 ( R2 + R4 )( R3 + R4 )
sin2 x − sin2 y − sin2 z + 2sin x sin y sin z = 0 pentru

278
1 1 1
x= m( BDC ), y = m( ADC ), z = m( ADB) relaţia precedentă devine:
2 2 2
R2 R3 R1 R3 R1 R2 2 R R R R ( R + R3 + R4 )
− − + 1 2 3 4 2 =0
( R2 + R4 )( R3 + R4 ) ( R1 + R4 )( R3 + R4 ) ( R1 + R4 )( R2 + R4 ) ( R1 + R4 )( R2 + R4 )( R3 + R4 )
R1 + R4 R2 + R4 R3 + R4 R ( R + R3 + R4 )
sau − − +2 4 2 = 0, egalitate echivalentă cu
R1 R2 R3 R2 R3
ε1 − ε 2 − ε 3 − ε 4 + 2 ε 2ε 3 + ε 3ε 4 + ε 4ε 2 = 0 . Atunci (ε1 + ε 2 + ε3 + ε 4 )2 = (ε1 − ε 2 − ε 3 − ε 4 )2 +
4(ε1ε2 + ε1ε3 + ε1ε4 ) + 4(ε2ε3 + ε3ε4 + ε4ε2 ) + 4(ε1ε2 +ε1ε3 + ε1ε4 ) = 2(ε1 + ε2 + ε3 +ε4 )2 − 2(ε12 + ε22 + ε32 + ε42 )
de unde rezultă 2(ε12 + ε 2 2 + ε 3 2 + ε 4 2 ) = (ε1 + ε 2 + ε 3 + ε 4 ) 2 .

Observaţie: Cantităţile ε i şi ηi , i = 1, 4 se numesc numerele lui Beecroft..

Consecinţe:
i) ε1 + η1 = ε 2 + η 2 = ε 3 + η3 = ε 4 + η 4 .
ii) ε1 ⋅η1 + ε 2 ⋅η2 + ε 3 ⋅η3 + ε 4 ⋅η 4 = 0 .
Demonstraţie.
1
i) Avem −ε1 + ε 2 + ε 3 + ε 4 = 2η1 , deci (ε1 + ε 2 + ε 3 + ε 4 ) = η1 + ε1 = ε 2 + η 2 =
2
ε 3 + η3 = ε 4 + η 4 .
2
4 4
1 4 
4 4
ii) Avem: ∑ ε iηi = ∑ ε i (ε i + ηi ) −∑ ε =  ∑ ε i  + ∑ ε i2 = 0 .
i
2

i =1 i =1 i =1 2  i =1  i =1

II.17. Teorema lui Pompeiu113


„Dimitrie Pompeiu ştia să privească lucrurile vechi cu ochi noi.” – Paul Montel114

Fie triunghiul echilateral ABC şi M un punct în


planul sau ce nu aprţine cercului circumscris M'
triunghiului. Distanţele MA, MB, MC reprezintă
lungimile laturilor unui triunghi.
Demonstraţie. Soluţia 1. Fie M ' punctul obţinut A
60°
din M prin rotaţia de centru A şi unghi de 60º.
Atunci MM ' ≡ MA (deoarece triunghiul AMM’
este echilateral). Din congruenţa triunghiurilor
BAM şi CAM ' ( AM ≡ AM ' , BA ≡ CA , M

m(BAM ) = m(CAM ') = 60° + m(CAM ) ), rezultă


MB ≡ CM ', deci lungimile laturilor triunghiului
B C
113
Fig. 282
Dimitrie Pompeiu (1873-1954) –matematician român, profesor la Universitatea din Iaşi, membru al Academiei
Române, contribuţii importante în analiza matematică
114
Paul Montel (1876-1975) – matematician francez, membru al Academiei Franceze, contribuţii în analiza
matematică
279
MM ' C sunt egale cu cele ale segmentelor MA, MB, MC (Fig.282).
Observaţie: Dacă punctul M se află pe centrul cercului circumscris triunghiului echilateral
ABC, atunci conform teoremei lui Schooten segmentul cu cea mai mare lungime dintre
segmentele MA, MB, şi MC au lungimea egală cu suma lungimilor celorlalte două.
Soluţia 2. Notăm cu litere mici afixele punctelor corespunzătoare. Plecând de la relaţia
evidentă: ( m − a ) ( b − c ) + ( m − b ) ( c − a ) + ( m − c ) ( a − b ) = 0 (1) rezultă:
(m − a)(b − c) = −(m − b)(c − a) − (m − c)(a − b) . Trecând la modul în egalitatea precedentă
obţinem: m − a b − c = (m − a)(c − a ) + (m − c)(a − b) ≤ m − b c − a + m − c a − b , de

unde: m−a ≤ m−b + m−c adică MA ≤ MB + MC . Cum M nu aparţine cecului


circumscris triunghiului ABC rezultă MA < MB + MC . Din simetria relaţiei (1) rezultă
inegalităţile MB < MC + MA şi MC < MA + MB , adică segmentele MA, MB, MC
determină un triunghi.

II.18. Teorema lui Erdös – Mordell


„Nu poate exista un limbaj mai universal şi mai simplu, mai lipsit de greşeli şi de confuzii, adică mai
demn de a exprima raporturile invariabile dintre realităţile naturale. Matematica este tot atât de cuprinzătoare ca
însăşi natura. Ea defineşte toate raporturile sensibile, măsoară timpul, spaţiile, forţele şi temperaturile. Ştiinţa
aceasta dificilă se formează cu încetul, dar păstrează toate principiile odată ce şi le-a însuşit. Ea creşte şi se
consolidează fără încetare, în mijlocul atâtor erori ale spiritului uman.” - Baptiste Joseph Fourier 115

Dacă P este un punct în interiorul unui triunghi ABC , atunci


PA + PB + PC ≥ 2( PA1 + PB1 + PC1 ) , unde A1 , B1 , C1 sunt proiecţiile punctului P pe
laturile BC, AC, BA ale triunghiului ABC .
Demonstraţie. Soluţia 1. Notăm cu
A
a ', b ', c ' lungimile segmentelor PA, PB,
PC şi cu x, y, z lungimile segmentelor C2
PA1 , PB1 , PC1 . Din teorema cosinusului a' B1
B2 C1
în triunghiul PB C rezultă: z y
1 1

B1C1 = y + z + 2 yz cos A
2 2
(1). Cum b' P c'
patrulaterul AC1 PB1 este inscriptibil x
rezultă AB1C1 = APC1 (2). Deoarece B C
A1
AC1 BC
sin APC1 = , atunci PA = a ' = 1 1 Fig. 283
AP sin A
y 2 + z 2 + 2 yz cos A
de unde a' = .
sin A
x 2 + z 2 + 2 xz cos B x 2 + y 2 + 2 xy cos C
Analog se arată că b ' = , c' = , de unde rezultă:
sin B sin C

115
Baptiste Joseph Fourier (1768-1830) – matematician şi fizician francez, membru al Academiei Franceze,
contribuţii în toate domeniile matematicii
280
( y sin C + z sin B ) 2 + ( y c o s C − z co s B ) 2
P A + P B + P C = a '+ b '+ c ' = +
sin A

( z sin A + x sin C)2 + ( z cos A − x cos C)2 ( x sin B + y sin A)2 + ( x cos B − y cos A)2
+ + şi de
sin B sin C
y sin C + z sin B z sin A + x sin C x sin B + y sin A
aici avem: a '+ b '+ c ' ≥ + + ,
sin A sin B sin C
 sin C sin A   sin B sin A   sin B sin C 
adică a '+ b '+ c ' ≥ y  + + z +  + x + , deci
 sin A sin C   sin A sin B   sin C sin B 
x y
a '+ b '+ c ' ≥ 2( x + y + z ) (unde am utilizat inegalitatea + ≥ 2, ∀x, y > 0 ), cu egalitate
y x
dacă triunghiul ABC este echilateral.
Soluţia2. Fie B2 şi C2 proiecţiile punctelor B şi C pe dreapta B1C1 . Avem
BC ≥ B2 C2 = B2 C1 + C1 B1 + B1C2 (1') . Cum B2 C1 B = AC1 P = APB1 , rezultă că
triunghiurile dreptunghice BB2 C1 şi AB1 P sunt asemenea, de unde rezultă că
BC1 B1C
B2 C1 = PB1 ⋅ (2') şi analog se arată că B1C2 = PC1 ⋅ (3') . Cum patrulaterul
AP AP
AC1 PB1 este inscriptibil, din inegalitatea lui Ptolemeu rezultă:
PB1 PC
AC1 ⋅ PB1 + AB1 ⋅ PC1 = AP ⋅ B1C1 , de unde B1C1 = AC1 ⋅ + AB1 ⋅ 1 (4') . Din relaţiile
AP AP
PB1 PB1 PC1 PC
(1'), (2 '), (3') şi (4 ') rezultă: BC ≥ BC1 ⋅ + AC1 ⋅ + AB1 ⋅ + B1C ⋅ 1 , de unde :
AP AP AP AP
BC ⋅ AP ≥ PB1 (BC1 + C1 A) + PC1 ( AB1 + B1C) inegalitate echivalentă cu
AB AC
BC ⋅ AP ≥ PB1 ⋅ AB + PC1 ⋅ AC , deci AP ≥ PB1 ⋅ + PC1 ⋅ (5') . Urmărind acelaşi
BC BC
BA BC
raţionament se obţin inegalităţile: BP ≥ PA1 ⋅ + PC1 ⋅ (6') şi
AC AC
AC BC
PC ≥ PA1 ⋅ + PB1 ⋅ (7') . Sumând inegalităţile ( 5 ') , ( 6 ') , ( 7 ')
AB AB
x y
şi ţinem cont de inegalitatea + ≥ 2, ∀ x, y > 0 , rezultă:
y x
 AB AC   AB BC   AC BC 
PA + PB + PC ≥  +  ⋅ PA1 +  +  ⋅ PB2 +  +  ⋅ PC2 ≥ 2( PA1 + PB1 + PC1 )
 AC AB   BC AB   BC CA 

Consecinţe:
1) Dacă triunghiul ABC este ascuţitunghic şi H este ortocentrul triunghiului ABC ,
atunci HA + HB + HC ≥ 6r , unde r este raza cercului înscris în triunghiul ABC .
Demonstraţie. Dacă triunghiul ABC este ascuţitunghic şi P coincide cu ortocentrul H al
triunghiului ABC din inegalitatea lui Erdös obţinem:
HA + HB + HC ≥ 2( HA1 + HB1 + HC1 ) (4) . Dacă ha , hb , hc sunt lungimile înălţimilor
triunghiului ABC , atunci (4) devine: HA + HB + HC ≥ 2(ha − HA + hb − HB + hc − HC ) ,

281
2
adică HA + HB + HC ≥ (ha + hb + hc ) (5) , de unde se obţine inegalitatea
3
2  2 S 2S 2 S  4S  1 1 1 
HA + HB + HC ≥  + + =  + +  . Aplicând inegalitatea mediilor
3 a b c  3 a b c
4S 9 12 S
rezultă HA + HB + HC ≥ ⋅ = = 6r , deci: HA + HB + HC ≥ 6r (5) .
3 a +b+c 2p

3
2) Într-un triunghi ascuţitunghic ABC, cos A + cos B + cos C ≤ .
2
Demonstraţie. Dacă P coincide cu centrul cercului circumscris (O) al triunghiului
 a2 b2 c2 
ascuţitunghic ABC , avem: 3R ≥ 2(OA1 + OB1 + OC1 ) = 2  R2 − + R2 − + R2 −  ,
 4 4 4 

adică: 3R ≥ 4R2 − a2 + 4R2 − b2 + 4R2 − c2 inegalitate echivalentă cu


3
3R ≥ 2 R(cos A + cos B + cos C ) , deci cos A + cos B + cos C ≤ (6) (unde am ţinut cont de
2
a = 2 R sin A şi de relaţiile analoage).

3) Dacă triunghiul ABC este ascuţitunghic şi H este ortocentrul triunghiului atunci


HA + HB + HC ≤ 3R .
Demonstraţie. Avem: HA + HB + HC = 2 R (cos A + cos B + cos C ) ≤ 2 R ⋅ 3 / 2 = 3R (7) , unde
am utilizat relaţia (6).

4) În orice triunghi ABC este adevărătă relaţia: 2r ≤ R (Relaţia lui Euler).


Demonstraţie. Din relaţiile (5) şi (7) rezultă 6r ≤ HA + HB + HC ≤ 3R, de unde rezultă
relaţia lui Euler: 2r ≤ R (8) .

5) Dacă I este centrul cercului înscris în triunghiul ABC, atunci


6r ≤ AI + BI + CI ≤ 4 R − 2r .
Demonstraţie. Dacă P coincide cu centrul cercului înscris I al triunghiului ABC , atunci
r
relaţia lui Erdös devine: AI + BI + CI ≥ 6r (9) . Cu AI = şi analoagele rezultă
A
sin
2
   A B B C C A
 1 1 1   sin 2 ⋅ sin 2 + sin 2 ⋅ sin 2 + sin 2 ⋅ sin 2 
AI + BI + CI = r  + +  = r .
 sin A sin B sin C   A B
sin ⋅ sin ⋅ sin
C 
 2 2 2  2 2 2 
A B C r
Ţinând cont că sin ⋅ sin ⋅ sin = şi de inegalitatea
2 2 2 4R
x y + y z + zx ≤ x 2 + y 2 + z 2 , ∀ x , y , z ∈ rezultă:
4R  2 A B C 1 − cos A 1 − cos B 1 − cos C 
AI + BI + CI ≤ r ⋅  sin + sin 2 + sin 2  = 4 R  + +  de
r  2 2 2  2 2 2 
 A B C
unde: AI + BI + CI ≤ 2 R 3 − ( cos A + cos B + cos C )  = 2 R 3 − 1 − 4sin sin sin  ,
 2 2 2
282
 r
deci AI + BI + CI ≤ 2 R  2 −  = 4 R − 2r (10) . Din relaţiile (9) şi (10) rezultă:
 R 
6r ≤ AI + BI + CI ≤ 4R − 2r .

6) Dacă G este centrul de greutate al triunghiului ABC, atunci GA + GB + GC ≥ 6r.


Demonstraţie. Dacă punctul P coincide cu centrul de greutate al triunghiului ABC , relaţia
lui Erdös devine: GA + GB + GC ≥ 2(GA1 + GB1 + GC1 ) . Ţinând cont de faptul că
1
GA1 = ha şi de relaţiile analoage avem:
3

( ha + hb + hc ) =  + +  ,
2 2 2S 2S 2S
GA + GB + GC ≥ adică:
3 3 a b c 
4S  1 1 1  4S 9 12S
GA + GB + GC ≥  + + ≥ ⋅ = = 6r .
3  a b c  3 a +b+c 2p

7) Dacă P este un punct în interiorul unui triunghi ABC , atunci


PAm + PB m + PC m ≥ 2( PA1m + PB1m + PC1m ), ∀m ∈ * , unde A1 , B1 , C1 sunt proiecţiile
punctului P pe laturile BC, AC, BA ale triunghiului ABC .
sin C sin B
Demonstraţie. Din teorema lui Erdös rezultă: PA ≥ y +z şi analoagele.
sin A sin A
 sinm C sinm A   sinm C sinm B   sinm B sinm A 
Obţinem: PAm +PBm + PCm ≥ xm  m + m  + ym  m + m  + zm  m + m  ≥
 sin A sin C   sin B sin C   sin A sin B 
≥ 2 ( x m + y m + z m ) , ∀m ∈ *
unde am aplicat inegalitatea dintre media aritmetică şi
geometrică.

8) Generalizarea teoremei lui Erdös - Mordell


Fie P un punct arbitrar în planul triunghiului ABC şi a ', b ', c ' distanţele de la P la
vârfurile A,B respectiv C şi x, y, z distanţele de la P la laturile BC, CA, respectiv AB.
b c c a a b
Atunci, a '+ b '+ c ' ≥  +  x +  +  y +  +  z , cu egalitate dacă P este centrul
c b a c b a
cercului circumscris triunghiului ABC .
Demonstraţie. Fie h1 lungimea înălţimii duse din A pe latura BC. Avem:
2 A[ ABC ] = a ⋅ h1 = ax + by + cz. Evident, a '+ x ≥ h1 cu egalitate dacă şi numai dacă punctul P
aparţine înălţimii din A. Avem aa '+ ax ≥ ah1 = ax + by + cz , de unde aa ' ≥ by + cz (11) .

Px Ox B'

B C
Fig. 284
283
C'
Fie AB ' C ' simetricul triunghiului ABC faţă de bisectoarea unghiului A (Fig.284 ). Atunci,
aplicând inegalitatea (11) pentru triunghiul AB ' C ' obţinem: aa ' ≥ cy + bz adică
c b
a'≥ y + z (12) cu egalitate dacă punctul P aparţine înălţimii din A a
a a
triunghiului AB ' C ' , dreaptă ce trece prin centrul cercului circumscris triunghiului ABC .
a c b a
Analog, se obţin relaţiile: b ' ≥ z + x (13) şi c ' ≥ x + y (14) . Sumând inegalităţile
b b c c
(12), (13) şi (14) rezultă concluzia.

Observaţie: Dacă P este un punct interior triunghiului ABC , x, y, z > 0 , avem:


a b
a '+ b '+ c ' ≥ 2( x + y + z ) (unde am utilizat faptul că + ≥ 2, ∀a, b > 0 ) care este
b a
inegalitatea lui Erdös-Mordell. Egalitatea are loc dacă a = b = c , adică dacă
triunghiul ABC este echilateral şi P este centrul cercului circumscris triunghiului ABC .

Teorema lui Barrow


9) Dacă P este un punct interior triunghiului ABC , atunci
PA + PB + PC ≥ 2( PA '+ PB '+ PC ') , unde P A ', P B ', P C ' sunt bisectoarele
unghiurilor B P C , C P A , A P B ( A ' ∈ ( BC ), B ' ∈ ( AC ), C ' ∈ ( AB )) .
Demonstraţie. Notăm cu a,b,c lungimile A
segmentelor PA,PB respectiv PC şi cu
α , β , γ măsurile unghiurilor BPC , CPA,
respectiv APB . Prin A[ XYZ ] notăm aria B'
C'
triunghiului XYZ. Din
A [ P B C ] = A [ P B A '] + A [ P C A '] rezultă: P

α α
PB ⋅ PC sin α = PB ⋅ PA'sin + PC ⋅ PA'sin ,
2 2 B C
2bc α A'
deci PA ' = cos . Utilizând
b+c 2 Fig. 285
2
inegalitatea ≤ xy , x, y > 0 , rezultă
1 1
+
x y
α α β γ
PA ' ≤ bc cos şi analoagele.Atunci, 2(PA'+ PB'+ PC') ≤ 2 bc cos + 2 ca cos + 2 ab cos
2 2 2 2
α β γ
(*). Rămâne să demonstrăm că a + b + c ≥ 2 bc cos + 2 ac cos + 2 bc cos .
2 2 2
α β γ α β α β
Deoarece α + β + γ = 2π = − cos  +  = − cos cos + sin sin ,
rezultă cos
2 2 2 2 2 2 2
inegalitatea de mai sus fiind echivalentă cu:
2
 β α  β α 
 a sin − b sin  +  a cos − b cos − c  ≥ 0 , ceea ce este adevărat.
 2 2   2 2 

284
Egalitatea din (∗) se obţine pentru a = b = c , adică dacă P este centrul cercului
circumscris triunghiului.
Generalizare a teoremei lui Erdös116
Fie λ1 , λ2 , λ3 ∈ + şi t ∈ [ 0,1] , iar P un punct în interiorul triunghiului ABC. Se notează
distanţele PA, PB, PC cu x1 , x2 respectiv x3 şi cu d1 , d 2 , d3 distanţele de la laturile AB,
 dt dt dt 
BC, respectiv CA. Atunci: λ1 x1t + λ2 x2t + λ3 x3t ≥ 2t λ1λ2 λ3  1 + 2 + 3  cu egalitate
 λ λ2 λ3 
 1
λ1 λ2 λ3
dacă şi numai dacă 2t
= 2t
= şi P este centrul cercului circumscris triunghiului
a b c 2t
ABC.
c b a c b b
Demonstraţie. Se cunoaşte că x1 ≥ d1 + d3 , x2 ≥ d3 + d1 , x3 ≥ d1 + d 2 .
a a b b c a
t t
c b 
t
c t b t
 d + d 3    ⋅ d1 +   ⋅ d3
t a a
1
Pentru 0 < t < 1 rezultă x1t ≥ 2t  a a
 ≥2 ⋅ şi analoagele.
 2  2
 
1
Utilizând inegalitatea x+ ≥2 pentru x > 0 rezultă: λ1 x1t + λ2 x2t + λ3 x3t ≥
x
  c t b
t
a
t
c
t
b
t
a
t

   ⋅ λ 2 +   ⋅ λ3 ⋅ λ
  3   1 + ⋅ λ   1   ⋅ λ2
⋅ λ + 
2t    c ⋅ d 1t +   a ⋅ d 2t +   b
b c a
⋅ d 3t  ≥
 2 2 2 
 
 
2t ( λ2 λ3 ⋅ d1t + λ3 λ1 ⋅ d 2t + λ1λ2 ⋅ d3t ) .

 dt dt dt 
Observaţie: Dacă t > 1 atunci, λ1 x1t + λ2 x2t + λ3 x3t ≥ 2 λ1λ2 λ3  1 + 2 + 3 
 λ λ2 λ3 
 1

Consecinţe:
λ1 λ2 λ3  1 1 1 
1) + + ≥ 2t λ1λ2 λ3  + + 
 λ3 x3t 
 λ1 x1 λ2 x2
t t t t t
d1 d 2 d 3

λ1 λ2 λ3 2t λ1λ2 λ3  x1t x2t x3t 


2) + + ≥  + + 
( x1d1 ) ( x2 d 2 ) ( x3 d3 ) ( x1 x2 x3 )  λ1 λ2 λ3 
t t t t

t  1 1 1 
3) λ1 ( x1d1 ) + λ2 ( x2 d 2 ) + λ3 ( x3 d3 ) ≥ 2t λ1λ2 λ3 ( d1d 2 d3 ) 
t t t
+ + 
 λ dt λ2 d 2
t
λ3 d3t 
 1 1 

116
Paul Erdös (1913-1996) – matematician ungur, profesor la Universitatea Notre Dame, contribuţii importante în
teoria numerelor şi matematici discrete
285
t  1 1 1 
4) λ1d1t + λ2 d 2t + λ3 d3t ≥ 2t λ1λ2 λ3 ( d1d 2 d3 )  + + 
 λd t
λ2 d 2
t
λ3 d3t 
 1 1
i) Dacă P este centrul cercului înscris în triunghiul ABC , atunci d1 = d 2 = d3 = r şi
A B C
x1 = r cos , x2 = r cos , x3 = r cos . Din consecinţa (2) şi utilizând egalitatea
2 2 2
A B C r A B C
sin sin sin = obţinem: λ1 sin t + λ2 sin t + λ3 sin t ≥
2 2 2 4R 2 2 2
t
 2  1 t A 1 t B 1 t C
λ1λ2 λ3    cos + cos + cos  (t ∈ (0,1]).
 2 R   λ1 2 λ2 2 λ3 2
hi
ii) Dacă P este centrul de greutate al triunghiului ABC , atunci di = ( hi , i = 1,3 reprezintă
3
2 2 2
înălţimile ABC ,) şi x1 = AG = ma , x2 = mb , x3 = mc . Din consecinţa (1) pentru
3 3 3
λ1 λ2 λ3  1 1 1 
t ∈ (0,1] rezultă: + + ≥ + +  . Dacă înlocuim λi = λit
ht
1 ht
2 h  λ1 mat
t
3 λ2 mb
t
λ3 mct 

t t t
 hh   h3 h1   h1h2 
obţinem:  2 3  +  +  ≤3
 ma     
   mb   mc 

II.19. Teoremele lui Fagnano117


„În univers urla un punct
de durerea unui cerc
care-l înconjoară.”
N. Stănescu118

1) Fiind dat triunghiul ascuţitunghic ABC să se determine triunghiul înscris XYZ în


triunghiul ABC a cărui perimetru este minim.
Demonstraţie. Soluţia 1. Fie X ' şi X '' simetricele punctului X ∈ ( BC ) faţă de laturile
AC, respectiv AB (Fig. 286). Atunci, XZ = ZX '' şi XY = YX ' , deci perimetrul
triunghiului XYZ este egal cu XY + YZ + ZX = X ' Y + YZ + ZX '' . Dacă X este fixat pe
BC, atunci triunghiul cu perimetrul minim
A
înscris în triunghiul ABC se obţine atunci
X'
când punctele Y şi Z aparţin dreptei X ' X " .
Deoarece AB şi AC sunt mediatoarele Z
segmentelor XX " , respectiv XX ' rezultă
X"
YAX ' ≡ YAX şi XAZ ≡ ZAX " , de unde Y
m( X ' AX ") = 2[m(YAX ) + m( XAZ )] = 2m( BAC ).
Cum AX ' = AX = AX '' rezultă că triunghiul B X C
Fig. 286
117
Giovanni Fagnano (1715-1797) – matematician italian, contribuţii în geometrie
118
Nichita Stănescu (1933 – 1983) – eseist, poet român, ales postum membru al Academiei Române
286
AX ' X " este isoscel. Cum unghiul X ' AX " este constant pentru orice alegere a lui X
rezultă că toate triunghiurile AX ' X " sunt asemenea. Latura X ' X " are lungimea
minimă atunci când latura AX ' are lungimea minimă. Dar AX ' = AX = AX " care
are lungimea minimă atunci când X este proiecţia lui A pe BC. Analog, Y este
proiecţia lui B pe AC şi Z este proiecţia lui C pe AB. Perimetrul minim al unui
triunghi înscris este acela al triunghiului ortic.

Observaţie: Dacă m( A) = 90° , atunci punctele Z şi Y coincid cu A, iar dacă


m( BAC ) > 90° , atunci triunghiul XYZ este triunghiul degenerat XAA.

Soluţia 2. A

Z Hb

Hc Y
H

B C
Ha X

Fig. 287

Fie AH a , BH b , CH c înălţimile triunghiului ABC,H ortocentrul său şi


punctele X ∈(BC) , Y ∈ ( AC ) , Z ∈ ( AB), (Fig. 287). Avem:
YZ ⋅ H bH c XZ ⋅ H cH a XY ⋅H aH b
YZ + ZX + XY = + + ≥
H bH c H cH a H aH b
uur uuuuuur uuur uuuuuuur uuur uuuuuuur
YZ ⋅ H b H c XZ ⋅ H cH a XY ⋅ H aH b
+ + sau YZ + ZX + XY ≥
HbH c H cH a H aHb
uuuur uuuuuur uuuur uuuuuur uuuur uuuuuur uuuuur uuuuuur uuuuur uuuuuur uuuur uuuuuur
(YH b + H b H c + H c Z ) ⋅ H b H c (ZHc + Hc Ha + Ha X ) ⋅ Hc Ha ( XHa + Ha Hb + HbY ) ⋅ Ha Hb
+ + =
HbHc Hc Ha Ha Hb
uuuuuur uuuuuur uuuuuur uuuuuur uuuuuur uuuuuur
uuuuur  H a Hb H a H c  uuuur  Hb Hc Hb H a  uuuur  Hc H a Hc Hb 
Hb H c + H c H a + H a Hb + XH a  +  + YHb  +  + ZH c  + 
 H a Hb H a H c   Hb Hc Hb H a   Hc H a Hc Hb 
(1). Deoarece triunghiul ABC este ascuţitunghic, înălţimile sale sunt bisectoarele
uuuuuuur uuuuuuur uuuuuur uuuuuuur uuuuuuur uuuuuur
Ha Hb H a Hc Hb Hc Hb H a Hc H a Hc Hb
triunghiului ortic, deci vectorii + , + , + sunt
Ha Hb H a Hc Hb Hc Hb H a Hc H a Hc Hb
uuuuur uuuur uuuur
perpendiculari pe vectorii XH a , YH b , respectiv ZH c , deci din relaţia (1) rezultă
uur uur uur
XY + ZX + XY ≥ H b H c + H c H a + H a H b (2). Dacă vectorii YX , ZX , XY au aceeaşi
uuuuuur uuuuuuur uuuuuuur
direcţie cu vectorii H b H c , H c H a , respectiv H a H b , atunci există numerele pozitive
uur uuuuuur uuur uuuuuuur uuur uuuuuuur
α , β , γ astfel încât Y Z = α H b H c , Z X = β H c H a , X Y = γ H a H b . Avem
uuuuuur uuuuuuur uuuuuuur r uuuuuur uuuuuuur uuuuuuur r
α H b H c + β H c H a + γ H a H b = 0 şi H b H c + H c H a + H a H b = 0 , de unde rezultă că

287
uur uuuuuur uuur uuuuuuur uuur uuuuuuur
α = β = γ , deci YZ = α H b H c , Z X = α H c H a , X Y = α H a H b , ceea ce implică
YZ = α Hb Hc , ZX = α Hc Ha , XY = α Ha Hb . Astfel, YZ + ZX + XY = α (Hb Hc + Hc H a + Ha Hb ) (3)
uur uuuuuur uuur uuuuuuur uuur uuuuuuur
care cu relaţia (2) ne dă α = 1 . Atunci , YZ = H b H c , ZX = H c H a , XY = H a H b ,ceea ce
înseamnă că punctele X, Y, Z coincid cu punctele H a , H b , respectiv H c . În concluzie,
cel mai mic perimetru al unui triunghi înscris este cel al triunghiului ortic.

2) Fie M un punct variabil în planul triunghiului ABC. Suma MA2 + MB 2 + MC 2 este


minimă dacă şi numai dacă M coincide cu centrul de greutate al triunghiului ABC.
Demonstraţie. Fie G centrul de greutate al triunghiului ABC şi a, b, c lungimile
laturilor triunghiului. Din teorema lui Leibniz rezultă:

M
G

D C
B

Fig. 288

1 2
MA 2 + MB 2 + MC 2 = 3 MG 2 + ( a + b 2 + c 2 ) ≥ 0, minimul sumei se realizează atunci
3
când punctul M coincide cu G.

3) Fie x, y şi z distanţele de la un punct M situate în interiorul triunghiului ABC la


laturile BC, CA respectiv AB. Produsul x ⋅ y ⋅ z este minim dacă M coincide cu centrul de
greutate G al triunghiului ABC.
Demonstraţie. Produsul x⋅ y⋅z este minim când produsul
(abc) ⋅ ( xyz ) = (ax) ⋅ (by ) ⋅ (cz ) este minim. Cum ax + by + cz = 2 ⋅ A[ ABC ] rezultă că minimul
se realizează atunci când ax = by = cz , adică când M coincide cu G.

288
II.20. Dreapta lui Droz – Farny119
„Dacă cineva va găsi demonstraţia axiomei paralelelor, ar merita un diamant cât Pământul de mare. Celui care îi
va reuşi aceasta, acestuia, muritori, să-i ridicaţi un monument nepieritor.” – Farkaş Bolyai120

Teorema lui Droz - Farny


Fie d ' şi d " două drepte perpendiculare ce trec prin ortocentrul H al unui triunghi
ABC. Fie A ' şi A " , B ' şi B ", C ' şi C " punctele de intersecţie ale dreptelor d ' şi
d " cu laturile BC, AC, respectiv AB. Să se arate că mijloacele segmentelor A ' A " , B ' B "
şi C ' C " sunt trei puncte coliniare.
Demonstraţie. Soluţia 1. Dacă triunghiul ABC este dreptunghic, teorema este evidentă.
Presupunem că triunghiul ABC este oarecare. Fie C cercul circumscris triunghiului ABC ,
α , β , γ mijloacele segmentelor A ' A" , B ' B " , respectiv C ' C " . Fie Ca, Cb, Cc cercurile
circumscrise triunghiurilor HA ' A " , HB ' B " , respectiv HC ' C " şi
H a (respectiv H b , H c ) simetricele punctului H faţă de dreapta BC (respectiv CA, AB).
Cercurile Ca, Cb, Cc au centrele în punctele α , β , respectiv γ . Cum H a ∈ C, H ∈ Ca şi A ' A"
este diametrul în Ca, , rezultă că punctul H a ∈ Ca , deci punctul H a aparţine cercurilor C şi
Ca şi perpendicularei duse din H pe BC. Analog, H b aparţine cercurilor C şi Cb precum şi
dreptei BH. Fie punctul H c simetricul lui H faţă de AB. Punctul H c ∈ C. Din lemă rezultă
că dreptele H a A ', H b B ' şi H c C ' se intersectează în punctul N ∈ C. Din teorema lui Miquel

B" Cb

A
Hb

C
B'
Hc
H

A" C
B
A' N
Ha
Ca
Fig. 289

aplicată triunghiului A ' NB ' cu H a ∈ A ' N , H b ∈ NB ', H ∈ A ' B ' , rezultă că cercurile trec
printr-un punct comun M. Analog se poate demonstra că C , Cc şi Cb conţin punctele H şi M,
de unde rezultă că cercurile sunt coaxiale, deci au centrele coliniare.

119
Arnold Droz -Farny (1856-1912) – matematician elveţian, contribuţii în geometrie
120
Farkaş Bolyai (1775-1856) – matematician român de origine maghiară, contribuţii fundamentale în geometrie
289
A' B ' B 'C ' C ' A '
Soluţia 2. Deoarece transversalele sunt ortogonale rezultă: = = (1)
A" B " B "C " C " A"
(vezi „Ortocentrul unui triunghi”). Fie β E A " B " , E ∈ A " B ' , {D} = B ' C "I β E. Avem
A '' B " B "C " A' B ' B 'C '
βE = , βD = , αE = , γD= (2). Din relaţiile (1) şi (2) rezultă că
2 2 2 2
βE αE
= . Relaţia precedentă arată că triunghiurile dreptunghice βγ D şi βα E sunt
βD αD
asemenea, deci punctele α , β , γ sunt coliniare.
Observaţie: Dreapta αβγ se numeşte dreapta Droz-Farny.

Generalizarea teoremei lui Droz - Farny


Printr-un punct oarecare P din planul unui triunghi ABC , se duc două drepte
perpendiculare d ', d '' care intersectează laturile BC , CA, AB , respectiv în punctele
( A ', B ', C '), ( A '', B '', C '') . Dacă α , β , γ sunt proiecţiile punctelor A ', B ', C ' pe dreptele
PA, PB, PC , iar A1 , B1 , C1 sunt punctele de intersecţie dintre dreapta d '' cu dreptele
α A ', β B ', respectiv γ C ' , atunci mijloacele segmentelor A ' A1 , B ' B1 , C ' C1 sunt coliniare.
Demonstraţie. Triunghiurile
C1
PA1 A ', P B1 B ', P C 1 C ' sunt
dreptunghice. Fie ωa , ωb , ωc
mijloacele segmentelor
A ' A1 , B ' B1 , C ' C1 şi µa , µb , µc d"
mijloacele segmentelor ωc
PA ', PB ', PC ' . Dreapta A1 µa A
intersectează perpendiculara în B1 A2
A' pe d ' în punctul A2
şi fie B2 , C2 punctele analog ωb
γ
construite. Patrulaterul PA1 A ' A2 P β
este paralelogram deoarece C' B ' µa d'
laturile opuse sunt paralele şi
congruente. Deci PA2 A1 A ' şi B C A'
cum A1 A ' ⊥ PA rezultă
ωa
PA2 ⊥ PA . Fie A3 , B3 , C3 α
simetricele punctelor A2 , B2 , A1
respectiv C2 faţă de dreapta d ' .
Fig. 290 A3
Atunci, patrulaterul PA1 A3 A '
este dreptunghi, deoarece
PA1 A ' A3 , PA1 = A ' A2 = A ' A3 , PA1 ⊥ PA ' , deci cercul circumscris al său este punctul
ωa - mijlocul segmentului A ' A1 . Deoarece punctele A2 , B2 şi C2 sunt coliniare, rezultă şi
simetricele lor în raport cu dreapta d ' - punctele A3 , B3 , C3 - sunt coliniare, deci şi punctele
ωa , ωb , ωc sunt coliniare.
Observaţie: Dacă punctul P este ortocentrul triunghiului ABC, atunci se obţine teorema lui
Droz – Farny.
290
II.21. Teorema lui Steiner - Lehmus121
„Steiner este cel mai mare geometru de la Apollonius încoace.” – W. Ball

Un triunghi care are două bisectoare interioare egale (măsurate de la vârf la latura
opusă) este isoscel.
Demonstraţie.
Soluţia 1. Fie BE şi CF bisectoarele
A unghiurilor B, respectiv C ale ABC
G (Fig. 291). Presupunem că AB ≠ AC
şi anume fie AB < AC , atunci
m( ACB) < m( ABC) de unde rezultă
E m( ACB) m( ABC)
F că < . În
2 2
triunghiurile BEC şi BFC, rezultă
CE > BF (1) construim
paralelogramul BEGF. Astfel
B C m( ABC )
EG ≡ BF , m( FGE ) = ,
Fig. 291 2
FG = BE = FC , de unde
m( ABC ) m( ACB )
m( FGC ) = m( FCG ) . Din m( FGE ) = > m( FCE ) = rezultă că
2 2
m( EGC ) < m( ECG ) , de unde : CE < EG = BF contradicţie cu (1). Deci, presupunerea
făcută că AB < AC este falsă. Analog se tratează cazul în care AB > AC şi atunci rezultă
AB = AC , adică triunghiul ABC este isoscel.

2bc A
Soluţia 2. Utilizăm faptul că la = cos (unde la este lungimea bisectoarei interioare a
b+c 2
B
cos
unghiului BAC ). Fie că lb = lc adică 2 = b(a + c) . Presupunem prin absurd că B > C ,
C c ( a + b)
cos
2
c (a + b) B C
adică b > c şi atunci > 1 , de unde cos > cos ,deci B < C absurd. Analog,
b( a + c ) 2 2
dacă B < C se ajunge la o contradicţie. Urmează că m( ABC ) = m( ACB) , adică triunghiul
ABC este isoscel.

Observaţie: Dacă bisectoarea exterioară a unghiului B întâlneşte prelungirea laturii AC în


punctul F atunci segmentul BF se numeşte bisectoare externă a lui B. Fie CG bisectoarea
externă a lui C. Este uşor de demonstrat că dacă AB = AC atunci FB = CG.

120
Jakob Steiner (1796 – 1863) – matematician german, profesor la Universitatea din Berlin, contribuţii în
geometria proiectivă
Daniel Lehmus (1780 – 1863) – matematician german, profesor la Universitatea din Berlin
291
Reciproca (dacă doua bisectoare externe ale unui triunghi sunt egale, atunci triunghiul este
isoscel) nu este neaparat adevarată. Un exemplu elocvent în acest sens este triunghiul lui
Emmerich.Triunghiul lui Emmerich are unghiurile de masuri egale cu 132° , 36° şi
respectiv 12° şi are două bisectoare externe egale. Fie triunghiul ABC în care
m( ABC) = 132° , m(CAB) = 36°, m(BCA) = 12° . Fie BF şi CG bisectoarele externe ale
180 ° − 132 °
unghiurilor B, respectiv C. Avem: m ( FBA) = = 24 °

m ( F B C ) = 2 4 ° + 1 3 2 ° = 1 5 6 ° , m(BCF ) = 12° , m(BFC) =180°−156°−12° =12° adică
triunghiul FBC este isoscel, cu FB = BC (1) . În triunghiul BCG avem:
180° − 12°
m( BCG) = = 84° , m(GBC ) = 48° , m( BGC ) = 180° − 86° − 48° = 48° , de unde
2
rezultă că triunghiul BCG este isoscel cu CG = BC (2). Din relaţiile (1) şi (2) rezultă că
FB = CG .

C
B

Fig. 292

292
II.22. Teorema lui Barbilian122

Fie ABC şi A ' B ' C ' două triunghiuri echilaterale de acelaşi centru (O), cu vârfurile
notate în acelaşi sens de rotaţie. Să se arate că triunghiurile sunt de trei ori omologice în
ordinele: ( ABC , C ' B ' A '), ( ABC , B ' A ' C '), ( ABC , A ' C ' B ').
Demonstraţie.
B3

A
A' C2
B2
C'
C3
B1
A3
B A1 C A2

B'
C1 Fig. 293

Fie { A1 } = BC ∩ B ' C ',{ A2 } = A ' C '∩ BC ,{ A3 } = A ' B '∩ BC ,{ B1 } = B ' C '∩ AC ,


{B2 } = A ' C '∩ AC , {B3 } = A ' B '∩ AC , {C1} = AB ∩ B ' C ',{C2 } = A ' C '∩ AB,{C3} = A' B '∩ AB.
Pentru ca triunghiurile ( ABC , C ' B ' A ') să fie omologice vom arăta că dreptele AC ', BB ' şi
CA ' sunt concurente. Din congruenţa triunghiurilor OAA ', OBB ' şi OCC ' rezultă
AA ' ≡ BB ' ≡ CC '. Deoarece C2 AA ' ≡ A3 BB ' ≡ B1CC ' rezultă AC2 ≡ BA3 ≡ CB1 şi de
aici B1 A ≡ C2 B ≡ A3C (1). Deoarece AB2 C2 ≡ BA3C3 ≡ CA1 B1 rezultă AB2 ≡ BC3 ≡ CA1 ,
de unde B2 C ≡ C3 A ≡ A1 B (2) (unde am utilizat faptul că
m( AOA ') = m( BOB ') = m( COC ') = m( AB2 A ') = m( BC3 B ') = m( CB2 C ') ). Din
teorema lui Menelaus aplicată în triunghiul ABC şi transversalei B ' C ' rezultă:
A1 B B1C C1 A C1 A A1C B1 A
⋅ ⋅ = 1, de unde se obţine egalitatea = ⋅ . Atunci,
A1C B1 A C1 B C1 B A1 B B1C
C1 A A3 B B2 C A1C B1 A B2 C
⋅ ⋅ = ⋅ ⋅ = 1 (unde am utilizat relaţiile (1) şi (2)), de unde rezultă
C1 B A3C B2 A A1 B B1C B2 A
că punctele C1 , A3 şi B2 sunt coliniare, adică triunghiurile ABC şi C ' B ' A ' sunt omologice.
Analog, se arată omologia celorlalte perechi de triunghiuri.

122
Dan Barbilian (1895-1961) – matematician român, profesor la Universitatea din Bucureşti, contribuţii în algebră
şi geometrie

293
II.23. Teorema lui Bottema
„În larg azur ca Sfinxul stau mândră şi ciudată.
Mi-i inima de gheaţă şi trupul cum sunt crinii.
Urăsc tot ce e zbucium, tulburător de linii
Şi nu plâng niciodată, şi nu râd niciodată.”
Charles Baudelaire123

Pe laturile ABC se construiesc în exterior pătratele ABDE şi ACFG. Fie M mijlocul


segmentului DF. Să se arate că triunghiurile BMC şi EMG sunt dreptunghice şi isoscele.
Demonstraţie.
G

E
A F
M

D
B T C

Fig. 294

π
Notăm cu litere mici afixele punctelor corespunzătoare. Din D = RB2 ( A )
π
π
rezultă d = b + i ( a − b ) (unde prin RB2 ( A) am notat rotaţia de centru B şi unghi a
2
π
− d + f b + c + i (c − b)
punctului A), iar F = RC 2 ( A) , deci f = c − i (a − c ), de unde m = = .
2 2
2 2
Atunci BM = m − b = c − b şi CM = m − c = c − b , de unde rezultă că triunghiul
2 2
m−b
BMC este isoscel. Din = −i ∈ i ⋅ * , rezultă că BM ⊥ MC , adică triunghiul BMC
m−c
este dreptunghic isoscel. Analog, se demonstrează că şi triunghiul EMG este dreptunghic
isoscel.

123
Charles Baudelaire (1821 – 1867) – poet francez
294
II.24. Teorema lui Goormaghtigh
„Fără a o baza pe intuiţie, geometria are totuşi un caracter intuitiv. Se pare, că idea de figură, figurile elementare:
cercul, triunghiul, sfera etc., formate în practica milenară a omului, fac parte din zestrea informaţională a celulei
nervoase, sunt transmise ereditar şi dau o mare mobilitate intuiţiei.” – Radu Miron124

1) Fie TATBTC
triunghiul tangenţial al triunghiului ABC şi punctele X , Y , Z
OX OY OZ
aparţinând dreptelor OTA , OTB respectiv OTC astfel încât = = =t.
OTA OTB OTC
Dreptele AX , BY , CZ sunt concurente în izogonalul conjugat al punctului P ce
OP 1
aparţine dreptei lui Euler astfel încât = .
PH 2t
A
Demonstraţie. Izogonala dreptei AX intersectează OTA în
punctul X ' . Triunghiurile OAX şi OX ' A sunt asemenea, H P
OX OA O
deci = de unde rezultă OX ⋅ OX ' = OA , adică
2
Ma C
OA OX ' B
punctele X şi X ' sunt inverse în cercul circumscris. De X'
asemenea punctele M a (mijlocul laturii BC) şi TA sunt
inverse. Fie {P} = AX 'I OH . Avem, X

OP OX ' OX ' 1 OTA 1


= = = ⋅ = . Urmând acelaşi TA
PH AH 2OM 2 OX 2t
raţionament se arată că dreptele BTB , CTC conţin punctul P,
Fig. 295
adică izogonalele dreptelor AX , BY , CZ sunt concurente, de
unde rezultă că şi dreptele AX , BY , CZ sunt concurente.

1
Observaţie: Pentru t = , X, Y, Z sunt centrele cercurilor circumscrise triunghiurilor
2
OBC, OCA, respectiv OAB. Dreptele AX, BY, CZ sunt concurente în punctul izogonal
conjugat al mijlocului segmentului OH (centrul cercului lui Euler al triunghiului ABC)
– punctul de concurenţă se numeşte punctul lui Coşniţă .

Teorema lui Goormaghtigh


Fie O centrul cercului circumscris unui triunghi ABC, punctele A1 , B1 , C1 pe dreptele OA,
OA1 OB1 OC1
OB, respectiv OC astfel încât = = = t. i) Intersecţiile perpendicularelor
OA OB OC
din A1 pe OA, B1 pe OB şi C1 pe OC cu laturile BC, CA respectiv AB se află pe o
dreaptă d. ii) Dacă M este proiecţia lui O pe dreapta d, M ' un punct pe OM astfel
încât OM = t ⋅ OM ' ,atunci punctul invers al punctului M ' în raport cu punctul O
este izogonalul conjugat al punctului P de pe dreapta lui Euler ce are proprietatea
OP 1
= .
PH 2t
Demonstraţie. i) Triunghiul XYZ determinat de intersecţiile perpendicularelor în A1 , B1 , C1 ,
se obţine prin omotetia de centru O şi raport t a triunghiului tangenţial (Fig. 296).
Vârfurile X, Y, Z aparţin dreptelor OA ', OB ', respectiv OC '.

124
Radu Miron (1927- ) – matematician român, profesor la Universitatea din Iaşi, membru al Academiei Române
295
Din teorema 1) dreptele AX, BY, CZ sunt concurente în punctul izogonal conjugat al
OP 1
punctului P de pe dreapta OH pentru care = . Fie
PH 2t
{ X '} = BC I YZ , {Y '} = CA I ZX , {Z '} = AB I XY . Vom arăta că punctele X ', Y ', Z '
aparţin unei drepte d. Fie R raza cercului circumscris triunghiului ABC. Considerăm
inversiunea J de centru O şi raport t ⋅ R 2 , iar M ' = J(M). Inversiunea J transformă
punctele A, B, C în A1 , B1 respectiv C1 . Fie M a , M b , M c mijloacele laturilor BC, CA,
respectiv AB. Deoarece patrulaterul BB1 M a X este inscriptibil rezultă
OM a ⋅ OX = OB1 ⋅ OB = t ⋅ R 2 . Analog, OM b ⋅ OY = OM c ⋅ OZ = t ⋅ R 2 ceea ce arată că
prin inversiunea J punctele X, Y, Z
se transformă respectiv în
M a , M b , M c . Imaginea punctului
X ' prin inversiunea J este
A
punctul A2 , al doilea punct de Y
intersecţie dintre cercurile
circumscrise triunghiurilor
OM b M c şi OB1C1 . Analog, Mc A1
Z O
imaginea punctelor Y ' şi Z ' prin C1
inversiunea J sunt respectiv B1
A2
punctele B2 (al doilea punct de B X' Ma C
intersecţie dintre cercurile
circumscrise triunghiurilor X
OM a M c şi OA1C1 şi C2 (al doilea
punct de intersecţie dintre
cercurile circumscrise
triunghiurilor OM b M a şi OB1 A1 . Fig. 296
Deci, punctele X ', Y ', Z ' aparţin
unei drepte d iar punctele O, A2 , B2 , C2 aparţin unui cerc C. ii) Prin inversiunea, J
dreapta AX se transformă în cercul OA1 M a de diametru OX ' şi conţine punctul M,
proiecţia lui O pe dreapta d. Analog, imaginile dreptelor BY şi CZ sunt cercurile de
diametru OY ' şi OZ ' ce-l conţin şi pe M. Deci punctul comun dreptelor AX, BY şi
CZ este imaginea lui M prin inversiunea J, adică intersecţia dintre OM şi cercul C.
Acesta este punctul diametral opus punctului O în cercul C .

Teorema lui Musselman


Fie O centrul cercului circumscris triunghiului ABC, H ortocentrul triunghiului ABC
şi A∗ , B∗ , C ∗ simetricele vârfurilor A, B, respectiv C faţă de laturile BC, CA şi AB.
Cercurile circumscrise triunghiurilor AOA∗ , BOB ∗ şi COC ∗ se întâlnesc într-un
punct care este inversul punctului izogonal conjugat al centrului cercului lui
Euler.

296
Demonstraţia rezultă din teorema lui Goormaghtigh pentru t = 1/ 2 . Centrele cercurilor
circumscrise triunghiurilor BOB ∗ şi COC ∗ sunt coliniare, cercurile având un al
doilea punct comun, simetricul lui O faţă de linia care uneşte centrele cercurilor.
Acest punct este inversul punctului izogonal al centrului cercului lui Euler .

II.25. Teorema lui Dergiades


„Nu există ramură a matematicii – oricât de abstractă ar fi ea – care să nu
se poată aplica într-o zi fenomenelor lumii reale.” – Nikolai Lobacevski125

Fie C1 (O1 , R1 ), C2 (O2 , R2 ), C3 (O3 , R3 ) trei cercuri care trec prin vârfurile B şi C, C şi A,
respectiv A şi B ale unui triunghi ABC şi D, E, F al doilea punct de intersecţie dintre
cercurile (C2 ) şi (C3 ) , (C3 ) şi (C1 ) , respectiv (C1 ) şi (C2 ) . Perpendicularele duse în D,
E, F pe AD, BE, respectiv CF intersectează laturile BC, CA, AB în punctele X, Y,
respectiv Z. Punctele X, Y şi Z sunt coliniare.
Demonstraţie.
Z
Fie M a M b M c triunghiul median
al triunghiului ABC (Fig. 297). Y A
Dreptele O1 M a , O 2 M b , O 3 M c
fiind mediatoarele laturilor A ' O3
O2
triunghiului ABC sunt concurente Mc F
X E
în centrul cercului circumscris (O) Mb
al triunghiului ABC, deci D
triunghiurile M a M b M c şi O1O2 O3 B
sunt omologice. Fie A ', B ', C ' Ma C
mijloacele segmentelor AX, BY
respectiv CZ. Deoarece O1
O2 O3 ⊥ AD rezultă O2 O3 DX şi
cum O2 O3 este mediatoarea
segmentului AD rezultă că
A ' ∈ O2 O3 . Deoarece M b M c BC Fig. 297
rezultă că A'∈ MbMc , deci
{ A '} = O2 O3 ∩ M b M c . Analog se
arată că {B '} = O1O3 ∩ M a M c , {C '} = OO 1 2 ∩ M a Mb . Conform teoremei lui Desargues

punctele A ', B ', C ' sunt coliniare. Din teorema lui Newton – Gauss aplicată patrulaterului
BCYZ rezultă că punctele X, Y şi Z sunt coliniare.

125
Nikolai Lobacevski (1792-1856) – matematician rus, profesor la Universitatea din Kazan, contribuţii
fundamentale în geometrie
297
II.26. Teoremele lui Pappus126
„Cea mai neglijată teoremă de existenţă în matematică este existenţa oamenilor. Matematica a fost creată de
oameni şi ea poartă amprenta lor.” – Hammer Presten

Fie triunghiul ABC şi punctele M ∈ ( BC ), N ∈ ( AC ), P ∈ ( BA) care împart aceste


segmente în acelaşi raport. Să se arate că triunghiurile ABC şi MNP au acelaşi centru de
greutate.
Demonstraţie. Soluţia 1. Fie M a mijlocul laturii
A
BC şi M ' simetricul lui M faţă de M a (Fig.
298). Fie G centrul de greutate al
P
triunghiului ABC , G1 centrul de greutate al
Q
triunghiului MNP şi G2 centrul de greutate al
G G2 N triunghiului M ' NP. Cum punctele M,N,P împart
laturile triunghiului ABC în acelaşi raport rezultă
B C că patrulaterul APM ' N este paralelogram de
M M a M'
centru Q, de unde rezultă că GG2 M a M ' şi
Fig. 298 3GG2 = 2 M a M ' = MM ' . În QMM ' avem:
1
G1G2 MM ' şi G1G2 = MM ', de unde rezultă
3
că punctele G şi G1 coincid. Soluţia 2 .Fie k raportul în care sunt împărţite laturile. Atunci,
uuur uuur uuuur uuuur uuur uuur
AP = k ⋅ PB, BM = k ⋅ MC , CN = k ⋅ NA. Fie G centrul de greutate al triunghiului ABC .
uuur uuur uuur uuur uuur uuur
uuur GA + k ⋅ GB uuuur GB + k ⋅ GC uuur GC + k ⋅ GA
Atunci, GP = , GM = , GN = . Sumând relaţiile
1+ k 1+ k 1+ k
uuuur uuur uuur uuur uuur uuur r
precedente rezultă: GM + GN + GP = GA + GB + GC = 0 , deci G este şi centrul de greutate
al triunghiului MNP.

Reciproca teoremei lui Pappus


Dacă un triunghi MNP înscris într-un triunghi ABC are acelaşi centru de greutate ca şi
triunghiul ABC, atunci vârfurile triunghiului MNP împart laturile triunghiului ABC în
acelaşi raport.
Demonstraţie. Fie M a , M b , M c mijloacele A
laturilor triunghiului ABC, G centrul de greutate
comun triunghiurilor ABC şi MNP, iar P ' P
piciorul medianei PG a triunghiului MNP
(Fig. 299). Vom demonstra că
P" G N
AP BM CN PG BG
= = . Evident, = =2,
PB MC NA P ' G M bG P'
deci M b P ' BP . Cum M b M a BP rezultă că B Ma M' C
M
punctele M b , P ', M a sunt coliniare. Fie
{P "} = CP 'I AB . Deoarece MP ' ≡ NP ' şi Fig. 299
CP ' ≡ P ' P " rezultă că patrulaterul MCEP " este

126
Pappus din Alexandria (290-350) – matematician grec, contribuţii în geometrie
298
BM BP " CN
paralelogram, deci MP " CE şi EP " BC , de unde = = . Analog se arată
MC AP " NA
BM AP
că = , de unde rezultă concluzia.
MC PB
Soluţia 2. Notăm afixele punctelor cu litere mici corespunzătoare, fie
AP BM CN a + bx b + cy c + az
= x, = y, = z. Atunci, p = , m= , n= . Trebuie să
PB MC NA 1+ x 1+ y 1+ z
a+b+c m+n+ p
demonstrăm că = dacă şi numai dacă x = y = z . Dacă x = y = z ,
3 3
atunci evident că a + b + c = m + n + p . Reciproc, fie a + b + c = m + n + p . Atunci,
 1 1   1 1   1 1 
a −  + b −  + c −  = 0. Cum
 1+ x 1+ z   1+ y 1+ x   1+ z 1+ y 
 1 1   1 1   1 1 
 1 + x − 1 + z  +  1 + y − 1 + x  +  1 + z − 1 + y  = 0 şi punctele A,B,C nu sunt coliniare,
     
1 1 1 1 1 1
rezultă că : − = − = − , de unde : x = y = z .
1+ x 1+ z 1+ y 1+ x 1+ z 1+ y

2) În triunghiul ABC, fie D ∈ ( AC ), E ∈ ( AB ), astfel încât {M } = BD ∩ CE ,


NB PB
{N } = DE ∩ BC , {P} = AM ∩ BC. Atunci = .
NC PC
Demonstraţie. Din teorema lui Menelaus aplicată în triunghiul ABC şi transversalei
EA DC NB
N – E – D rezultă: ⋅ ⋅ = 1 (1) (Fig. 300). Teorema lui Ceva aplicată în
EB DA NC
EA DC PB NB PB
triunghiul ABC ne dă: ⋅ ⋅ = (2) . Din relaţiile (1) şi (2) rezultă = .
EB DA PC NC PC
A

E
M

N B P C
Fig. 300

NB PB
Observaţie: Relaţia = ne arată că punctele N, B, P, C sunt conjugate armonic.
NC PC

299
II.27. Teorema lui Salmon127
„Matematica este formată din insule de cunoaştere într-un ocean de ignoranţă.”

Fie M un punct pe cercul circumscris unui


C'
triunghi ABC. Cercurile de diametre A M
( AM ), ( BM ), (CM ) se intersectează două câte
două în trei puncte coliniare. B'
Demonstraţie. Fie A' , B ' , C ' proiecţiile
punctului M pe laturile BC , CA respectiv AB.
Punctele A' , B ' , C ' sunt punctele de intersecţie
dintre cercurile de diametre ( AM ), ( BM ) şi
(CM ) , iar conform teoremei lui Simson rezultă B
A' C
că punctele A' , B ' , C ' sunt coliniare.

Fig. 301

II.28. Teorema lui Pedoe


„Nici un om nu se întăreşte citind un tratat de gimnastică, ci făcând exerciţii; nici un om nu se învaţă a judeca
citind judecăţile scrise de alţii, ci judecând singur şi dându – şi singur seama de natura lucrurilor.” – Mihai
Eminescu128

Fie triunghiul ABC şi A ' B ' C ' situate în acelaşi plan. Dacă laturile lor au lungimile a, b,
c respectiv a ', b ', c ' , atunci: a2 (−a '2 + b '2 + c '2 ) + b2 (a '2 − b '2 + c '2 ) + c2 (a '2 + b '2 − c '2 ) ≥ 16S ⋅ S ' ,
cu egalitate dacă triunghiurile sunt asemenea ( unde cu S şi S ' am notat ariile
triunghiurilor ABC , respectiv A ' B ' C ' ).
Demonstraţie.
A"
A'
A

b c' b'
c

a' C'
B'
B a C Fig. 303
Fig. 302

127
George Salmon (1819-1904) – matematician irlandez, contribuţii în algebră şi geometrie
128
Mihai Eminescu (1850-1889) – poet, jurnalist român, considerat cel mai important scriitor romantic din
literatura română
300
Pe latura BC se construieşte triunghiul A " BC asemenea cu ABC, de unde:
BC CA " A " B a CA " A " B ab '
= = , adică = = , deci: A " C = (1) (Fig.302). Din
B 'C ' C ' A' A ' B ' a' b' c' a'
teorema cosinusului în triunghiul A " CA rezultă:
AA "2 = b 2 + A " C 2 − 2b ⋅ A " C ⋅ cos ACA " (2). Din relaţiile (1) şi (2) rezultă
2
 ab '  ab '
AA " = b + 
2 2
 − 2b ⋅ cos( A ' C ' B ' − ACB) egalitate echivalentă cu
 a '  a'
a ' 2 ⋅ A A " 2 = a ' 2 b 2 + a 2 b ' 2 − 2 a a ' b b '  c o s C ⋅ c o s C ' + sin C ⋅ sin C '  ≥ 0 (3).
 
ab sin C a ' b 'sin C ' a 2 + b2 − c2
Dar S ⋅ S ' = ⋅ 4 SS ' = aa ' bb 'sin C ⋅ sin C ' (4), iar cos C = şi
2 2 2ab
a ' 2 + b '2 − c '2
cos C ' = (5). Din relaţiile (3), (4) şi (5) rezultă concluzia. Egalitatea are loc
2a ' b '
atunci când AA "2 = 0, adică A ≡ A ", deci când triunghiurile ABC şi A ' B ' C ' sunt
asemenea.

Consecinţe:
1) Dacă triunghiul A ' B ' C ' este echilateral, atunci inegalitatea devine:
a +b +c ≥ 4 3⋅S .
2 2 2

2) Dacă triunghiul A ' B ' C ' este CBA, deci a ' = b, b ' = c, c ' = a, atunci S = S ' şi inegalitatea
devine a 4 + b 4 + c 4 ≥ 16 S 2 .
3) Dacă triunghiul A ' B ' C ' este dreptunghic în A ' , atunci: b 2 c '2 + c 2 b '2 ≥ 8SS '.

II.29. Teorema lui Simson generalizată


„Din ceas, dedus adâncul acestei calme creste,
Intrată prin oglindă în mântuit azur,
Tăind pe înecarea cirezilor agreste,
În grupurile apei, un joc secund mai pur.”
Ion Barbu129
Teorema lui Simson generalizată
Fie M un punct pe cercul circumscris triunghiului ABC , A ' ∈ BC , B ' ∈ CA, C ' ∈ AB .
Dacă m( MC ' A) ≡ m( MB ' C ) ≡ m( MA ' C ) = ϕ , ϕ ∈ (0, π ) , atunci punctele A ', B ', C '
sunt coliniare.
Demonstraţie. Patrulaterele ABMC , AB ' MC ', A ' B ' MC sunt inscriptibile
(Fig. 304). Atunci, C ' B ' M ≡ C ' AM ≡ MCB de unde rezultă:
m ( A ' B ' M ) + m ( M B ' C ') = m( A ' B ' M ) + m( MCA ') = 180° adică punctele
A ', B ', C ' sunt coliniare.

129
Ion Barbu (1895-1961) – matematician român, profesor la Universitatea din Bucureşti, contribuţii în algebră şi
geometrie
301
Observaţii:
1) Teorema de mai sus aparţine lui Lazare Carnot.
2) Dreapta ce conţine punctele A ', B ', C ' se numeşte dreapta lui Simson generalizată de
unghi ϕ a punctului M faţă de triunghiul ABC (notaţie cu dM ).
3) Pentru a determina de exemplu
poziţia punctului M pe cercul C'
circumscris triunghiului ABC
procedăm astfel: alegem punctele N şi P
arbitrar pe cerc, construim dreapta A
AA1 astfel încât măsura unghiului dintre
AA1 şi NP să fie egală cu π − ϕ
( A1 aparţine cercului circumscris M
triunghiului ABC), iar din A1 construim B'
N
dreapta A1 M astfel încât
m( A1 M , BC ) = ϕ (M aparţinând
cercului circumscris triunghiului
B A'
ABC ).

Fie M , N , P puncte pe cercul P C


circumscris al unui triunghi ABC astfel A1
încât unghiul dintre dreapta lui Simson
generalizată de unghi ϕ a punctului M dM
şi dreapta NP să aibă măsura π − ϕ . Fig. 304
Triunghiul MNP se numeşte triunghi
ϕ − S faţă de triunghiul ABC .

Dreapta lui Simson generalizată de unghi de măsură ϕ a punctului M faţă de


triunghiul ABC este paralelă cu dreapta AA1 .
Demonstraţie. Fie A ', B ', C ' proiecţiile de unghi ϕ ale punctului M pe dreptele BC , CA
respectiv AB şi A1 punctul de intersecţie dintre MA ' cu cercul circumscris triunghiului
ABC . Deoarece patrulaterul AB ' MC ' este inscriptibil rezultă:
m( AC ' B ') = m( AMB ') = ϕ − m( B ' AM ) = ϕ − m( MBC ) = m( BMA1 ) = m( BAA1 ) ,
deci AA1 d M .

302
II.30. Teorema lui Sondat
„Pe linie de cercetare, geometria cuprinde domenii abstracte foarte generale, dar geometria elementară rămâne
foarte importantă în învăţământ fie prin aplicaţiile ei derecte diverse, fie ca o verigă în înţelegerea problemelor
moderne de teoria spaţiilor generalizate.” - Nicolae Mihăileanu130

Fie triunghiurile ABC , A1 B1C1 ortologice şi omologice, Q, Q1 centrele de ortologie şi


P centrul de omologie, iar d axa de omologie a acestor două triunghiuri. Punctele P, Q
şi Q1 aparţin unei drepte perpendiculare pe dreapta d .

Q
Fig. 305
A1
B1
C1
C' B' A'
C d
B

Demonstraţie. Punctul Q aparţine perpendicularelor duse din A1 , B1 , C1 respectiv pe laturile


BC , CA, AB , iar punctul Q1 aparţine perpendicularelor duse din A, B, C pe laturile
uuuur uuuur uuuur uuuur
1 = α A1 A, B1P = β B1B,
B1C1 , A1C1 , A1 B1 .Fie {B '} = CA ∩ C1 A1 ,{C '} = AB ∩ A1B1 , B ', C ' ∈ d. Avem AP
uuuur uuuur
C 1 A = γ C 1 C , P A − Q A = l a , P B − Q B = l b , P C − Q C = l c .Arătăm
2 2 2 2 2 2

B ' C A1 P C1C α
B ' P2 − B ' Q2 = C ' P2 − C ' Q2 (1), ceea ce implică PQ ⊥ d . Avem = ⋅ = .
B ' A A1 A C1 P γ
Din teorema lui Leibniz rezultă : PC 2 ⋅ γ − PA2 ⋅ α = PB '2 ⋅ (γ − α ) + B ' C 2 ⋅ γ − B ' A2 ⋅ α şi
lc γ − laα
QC 2 ⋅ γ − QA2 ⋅ α = QB '2 ⋅ (γ − α ) + B ' C 2 ⋅ γ − B ' A2 ⋅ α , iar de aici PB '2 − QB '2 = .
γ −α
Relaţia (1) este echivalentă cu βγ (lb − lc ) + γα (lc − la ) + αβ (la − lb ) = 0 (2). Deoarece
lb − lc
BC ⊥ A1Q rezultă BA2 − CA2 + CQ 2 − BQ 2 = (3). Din relaţia lui Leibniz rezultă
α
BA2α − BP2 = BA22 (α − 1) + A1 A2α − A1 P 2 , CA2α − CP 2 = CA12 (α − 1) + A1 A2α − A1 P 2 (4).

130
N. Mihăileanu (1912-1998) – matematician român
303
lb − lc
Din relaţiile de mai sus rezultă BA2 − CA2 + CQ 2 − BQ 2 = . Analog rezultă că
α
lc − la la − lb
CB 2 − AB 2 + AQ 2 − CQ 2 = , AC 2 − BC 2 + BQ 2 − AQ 2 = . Sumând relaţiile
β γ
lb − lc lc − la la − lb
precedente rezultă + + = 0 , adică tocmai relaţia PQ ⊥ d . Analog se
α β γ
arată că PQ1 ⊥ d , deci punctele P, Q, Q1 sunt coliniare.

II.31. Teoremele lui Maxwell131


„Vis al Dreptei Simple! Poate, geometria
Săbiilor trase la Alexandria,
Libere, sub ochiul de senin oţel,
În neclătinatul idol El Gahel.”
- Ion Barbu132

1) Fie P un punct în planul triunghiului ABC şi A ' B ' C ' un triunghi care are laturile
paralele cu cevienele punctului P în raport cu triunghiul ABC . Cevienele triunghiului
A ' B ' C ' paralele cu laturile triunghiului ABC sunt concurente.
Demonstraţie. Sunt o infinitate de triunghiuri A ' B ' C ' asemenea, deci este suficient să
demonstrăm problema pentru unul din aceste triunghiuri. Fie A1 , B1 , C1 picioarele
cevienelor corespunzătoare punctului P. Fie A ' A2 AB , B ' B2 BC , C ' C2 AC ,

A'
A C2
Q
C1 B1 B' B2
A2
P
C C'
B A1
Fig. 306

( A2 ∈ B ' C ' , B2 ∈ A ' C ' , C2 ∈ A ' B ' ). Din asemănarea triunghiurilor PC1 B şi B ' A2 A ' ,
C1 B PC1 AC1 PC1
respectiv PAC1 cu C ' A ' A2 rezultă = şi = , de unde :
A ' A2 B ' A2 A ' A2 A2 C '
C1 B A2C ' B A C B' A C B A'
= . Analog, se obţin relaţiile 1 = 2 şi 1 = 2 . Deoarece dreptele
C1 A A2 B ' B1C C2 A ' A1 B B2 C '

131
James Clerk Maxwell (1831-1879) – matematician şi fizician scoţian, profesor la Cambridge
132
Ion Barbu (1895-1961) – matematician român, profesor la Universitatea din Bucureşti, contribuţii în algebră şi
geometrie
304
A1C B1 A C1 B B A' C B ' A C '
AA1 , BB1 şi CC1 sunt concurente rezultă ⋅ ⋅ = 1 adică 2 ⋅ 2 ⋅ 2 = 1 ,
A1 B B1C C1 A B2 C ' C2 A ' A2 B '
deci dreptele AA2 , BB2 şi CC2 sunt concurente.

2) Fie P un punct în planul triunghiului ABC şi A ' B ' C ' un triunghi ce are laturile
perpendiculare pe cevienele punctului P în raport cu triunghiul ABC . Cevienele
triunghiului A ' B ' C ' perpendiculare pe laturile triunghiului ABC sunt concurente.
Demonstraţia este evidentă deoarece triunghiurile ABC şi A ' B ' C ' sunt ortologice (vezi
„Triunghiuri ortologice”).

II.32. Teorema trisecţiei

În triunghiul ABC fie medianele BB’ şi CC’. Printr-un punct T ∈ ( BC ) se duc paralelele
TD şi TE la medianele BB’, respectiv CC’,( D ∈ (AB), E ∈ (AC) ). Atunci, medianele BB’ şi
CC’ împart segmentul DE în trei segmente congruente.
Demonstraţie.
A

C' B'
N E
M
D L
K

B T C
Fig. 307

Fie {M}=DE∩BB’ , {N}=DE∩CC’ , {G} = BB '∩ CC ' , {K}=DT∩BB’, {L}=TE∩CC’.


1
Din DT CC ' rezultă că triunghiurile BKT şi BGC sunt asemenea şi cum C ' G = CC '
3
1 1
rezultă DK = DT . Din asemănarea triunghiurilor DKM şi DTE rezultă DM = DE .
3 3
1 1
Analog, NE = DE , de unde rezultă DM = MN = NE = DE.
3 3

305
II.33. Teoremele lui Harcourt
„Dacă numai aş avea teoremele! Atunci aş putea destul
de uşor să găsesc demonstraţiile.” - Bernhard Riemann133

1) Dacă distanţele de la vârfurile A, B, C ale unui triunghi ABC la o tangentă dusă la


cercul înscris în triunghiul ABC au lungimile a1 , b1 respectiv c1 , atunci
aa1 + bb1 + cc1 = 2 A[ ABC ] . (a, b, c sunt lungimile laturilor BC, AC respectiv AB).

A A
C'
P
A' d1
B' P

I d3
d1'
d2

B C B A' C

Fig. 308 Fig. 309

Demonstraţie. Fie ( x, y, z ) coordonatele baricentrice ale unui punct P şi l o dreaptă ce trece


prin P, iar d1 , d 2 şi d3 sunt distanţele de la vârfurile A, B, C la dreapta l (Fig.309). Vom
arăta că d1 ⋅ x + d 2 ⋅ y + d3 ⋅ z = 0. Fie { A '} = AP ∩ BC şi considerăm d1 negativ iar d 2 şi
AP x BA ' z
d3 pozitive. Atunci, = şi = ,deci distanţa de la A ' la l este egala cu
PA ' y + z A 'C y
yd 2 + zd3 −d1 AP y + z
d1' = , de unde = = , relaţie echivalentă cu
y+z d1' PA ' x
d1 ⋅ x + d 2 ⋅ y + d3 ⋅ z = 0 . Revenind, presupunem că dreapta l trece prin I (a, b, c) şi este
paralelă cu tangenta la cercul înscris. Atunci,distanţele de la A la l sunt: d1 = a1 − r ,
d 2 = b1 − r , d3 = c1 − r . Avem: aa1 + bb1 + cc1 =
a (d1 + r ) + b(d 2 + r ) + c(d 3 + r ) = (ad1 + bd 2 + cd 3 ) + (a + b + c)r = 0 + 2 A[ ABC ] = 2 A[ ABC ] .

2) Dacă distanţele de la vârfurile A, B, C la o tangentă dusă la cercul exînscris


corespunzător laturii BC a triunghiului ABC , au lungimile a1 , b1 respectiv c1 , atunci
− aa1 + bb1 + cc1 = 2 AABC (relaţii analoge se obţin prin celelalte două cercuri exînscrise).
Demonstraţie. Prin centrul I a (− a, b, c) ducem o paralelă la tangenta considerată şi fie
d1 , d 2 , d3 distanţele de la A, B, C la această paralelă. Atunci, − ad1 + bd 2 + cd3 = 0 iar
c1 = d3 + ra , b1 = d 2 + ra , c1 = d3 + ra , unde ra este raza cercului A -exînscris. Avem:
− aa1 + bb1 + cc1 = (−ad1 + bd 2 + cd 3 ) + ra (−a + b + c) = 0 + 2ra ⋅ ( p − a ) = 2 A[ ABC ] .

133
Bernhard Riemann (1826-1866) – matematician german, profesor la Universitatea Göttingen, contribuţii
fundamentale în analiza matematică şi geometria diferenţială
306
I.34. Teorema lui Zaslavsky
„Geometria proiectivă ne - a deschis cu cea mai mare usurinţă teritorii noi în ştiinţa noastră, a fost şi numită pe
bună dreptate un drum regal, conducând în domeniul său particular de cunostiinţe.” - Poncelet134

Fie P un punct în planul triunghiului ABC şi A ' B ' C ' simetricul triunghiului ABC faţǎ
de punctul P. Prin punctele A ', B ', C ' ducem trei drepte paralele care intersecteazǎ
dreptele BC, AC şi AB în punctele X ,Y respectiv Z. Sǎ se arate cǎ punctele X,Y şi Z sunt
coliniare.

A
C' d3
Z
P B' d2
Y

B
N Z' d K
C
X A' d1

Fig. 310

BX CY AZ
Demonstraţie. Vom arata cǎ ⋅ ⋅ = 1 . Prin punctul C ducem o paralelǎ (d) la
XC AY BZ
dreptele paralele date. Fie Z ' simetricul lui Z fatǎ de P. Atunci, BZB ' Z ' este paralelogram,
deci BZ ≡ B ' Z ' şi BZ B ' Z ' B ' A ' de unde rezultǎ cǎ punctele B ', A ', Z ' sunt coliniare
AZ A' Z '
si AZ ≡ A ' Z ' . Astfel,= (1). Fie { N } = BA' ∩ d şi { K } = AB ' ∩ d . Atunci, din
BZ B ' Z '
BX BA' CY B ' K BX CY AZ
teorema lui Thales rezultǎ: = ' şi = . Avem, ⋅ ⋅ =
XC A N AY B ' A XC AY BZ
BA' B ' K AZ B ' K AZ B ' Z ' AZ BZ AZ
'
⋅ '
⋅ = '
⋅ = ' '⋅ = ⋅ = 1 , unde am utilizat faptul cǎ
A N B A BZ A N BZ A Z BZ AZ BZ
triunghiurile A ' NZ ' şi B ' KZ ' sunt coliniare. Din reciproca teoremei lui Menelaus rezultǎ
cǎ punctele X,Y şi Z sunt coliniare.

134
Jean Poncelet (1788 - 1867) – matematician francez, contribuţii importante în geometria proiectivă

307
II.35. Teorema lui Zajic
„Ne vom aminti de Arhimede când îl vom fi uitat pe Eschil fiindcă limbile mor, iar ideile matematice sunt fară
moarte. Nemurirea poate părea un cuvânt inept, dar matematicianul are, probabil, cea dintâi şansă de a se bucura
de binefacerile ei, oricare ar fi acelea.” - G.H. Hardy135

În triunghiul ABC, fie X ∈ ( BC ) şi Ca punctul de tangenţă al cercului înscris în


triunghiul ABC cu latura BC. Dacă cercurile înscrise în triunghiurile ABX şi ACX sunt
tangente laturii AX în punctele T1 , respectiv T2 , atunci Ca X = T1T2 .
Demonstraţie. Fie a, b, c lungimile laturilor
triunghiurilor ABC şi p, pb , pc semiperimetrele A
triunghiurilor ABC, ABX respectiv ACX.
În triunghiul ABC avem:
BX = BCa + Ca X = p − b + Ca X şi
CX = CCa − Ca X = p − c − Ca X . În triunghiul
ABX, XT1 = pb − c şi în triunghiul ACX, T1
I2 I1
XT2 = pc − b. Atunci, T1T2 = XT1 − XT2 =
pb − pc + b − c . Dar, 2( pb − pc ) = T2
2[( c + A X + B X ) − ( b + A X + C X )] = C X Ca B
2(c − b + Ca X ) , de unde rezultă că
Fig. 311
pb − pc = c − b + C a X , deci
T1T2 = c − b + Ca X + b − c = Ca X .

Consecinţă: Dacă Ca este punctul de tangenţă al cercului înscris în triunghiul ABC cu


latura BC, atunci cercurile înscrise în triunghiurile ABCa şi AC Ca sunt tangente laturii
ACa în acelaşi punct.
Demonstraţia rezultă din teorema lui Zajic pentru X = Ca .

II.36. Teorema lui Viviani136


Suma distanţelor de la un punct, situat în interiorul A
unui triunghi echilateral la laturile triunghiului este
egală cu înalţimea triunghiului.
Demonstraţie. Fie P un punct în interiorul ABC şi P2
P3
P1 , P2 , P3 proiecţiile lui P pe laturile BC, AC respectiv AB
(Fig. 312). Fie a lungimea laturii AB şi h lungimea P
triunghiului echilateral ABC. Avem:
A [ A B C ] = A[ P B C ] + A[ P A C ] + A [ P A B ] , adică
B P1 C
a ⋅ h PP1 ⋅ a PP2 ⋅ a PP3 ⋅ a
= + + , de unde rezultă Fig. 312
2 2 2 2
h = PP1 + PP2 + PP3 .

135
G.H. Hardy (1877-1947) – matematician englez, profesor la Universitatea Cambridge, contribuţii importante
în teoria numerelor şi analiza matematică
136
Vincenzo Viviani (1622-1703) – inginer italian, contribuţii în fizică şi geometrie
308
II.37. Teorema lui Véronèse137
„Iubirea nu se dăruie decât pe sine şi nu ia decât de la sine. Iubirea nu stăpâneşte
şi nu vrea să fie stăpânită, fiindcă iubirii îi este de ajuns iubirea.” - Gibran Kahlil138

Fie triunghiurile omologice ABC şi A ' B ' C ', { A "} = BC '∩ CB ', {B "} = CA '∩ AC ',
{C "} = AB '∩ BA '. Triunghiul A " B " C " este omologic cu fiecare din triunghiurile
ABC , A ' B ' C ' iar cele trei centre de omologie sunt coliniare.
Demonstraţie. Fie { A1} = BC ∩ B ' C ',{B1} = AC ∩ A ' C ',{C1} = AB ∩ A ' B ' (Fig.313).
Triunghiurile ABC şi A ' B ' C ' fiind omologice rezultă că punctele A1 , B1 , C1 sunt coliniare.

B"

B1
A"

C1 C'
C"
A B' A'

B C A1

Fig. 313

Omologia triunghiurilor ABC şi A ' B ' C ' implică omologia triunghiurilor ABC ' şi
A ' B ' C de unde rezultă că punctele C1 , A ", B " sunt coliniare. Analog, punctele
( A1 , B ", C ") şi ( B1 , C ", A ") sunt coliniare, deci triunghiul A " B " C " este omologic cu
fiecare din triunghiurile ABC şi A ' B ' C ', având drept axă de omologie dreapta A1C1 , iar
cum dreapta AA " nu trece prin centrul O de omologie dintre triunghiurile ABC şi A ' B ' C '
rezultă că centrele de omologie O1 şi O2 - dintre triunghiurile A " B " C " şi ABC respectiv
A " B " C " şi A ' B ' C ' - şi punctul O sunt coliniare.

137
Giuseppe Véronèse (1854-1917) – matematician italian, profesor la Universitatea din Roma, contribuţii
importante în geometria proiectivă
138
Gibran Kahlil (1883-1931) – poet, filosof, sculptor libanez
309
II.38. Teorema lui Coşniţă139
„Matematica este o ştiinţă în care nu se ştie niciodată despre ce se
vorbeşte şi nici dacă este adevărat ce se vorbeşte.” – Bertrand Russel140
Teorema lui Coşniţă
Fie O centrul cercului circumscris unui triunghi ABC şi X, Y, Z centrele cercurilor
circumsrise triunghiurilor BOC, COA, respectiv AOB. Dreptele AX, BY şi CZ sunt
concurente.
Demonstraţia1. Fie { A1 } = A X ∩ B C ,
A
{B1} = BY ∩ CA, {C1} = CZ ∩ AB (Fig. 314). Deoarece
1
m ( BOX ) = m ( OCX ) = m ( BOC ) = m ( A), Z
2 Y
rezultă m( OBC ) = 90° − m( BOX ) = 90° − m( A) şi O B1
m ( CBX ) = m ( BCX ) = m ( A ) − m ( OBC ) = C1 O∗
2m( A) − 90° = α . Analog, m( ACY) = 2m( B) − 90° = β A1 C
BA1 A[ ABX ]
şi m( AZB) = 2( C ) − 90° = γ . Avem: = = B
A1C AACX X
1 Fig. 314
⋅ A B ⋅ B X ⋅ s in ( B + α )
2 A B s in ( B + α )
=
1
⋅ A C ⋅ C X ⋅ s in (C + α ) A C s in (C + α )
2
BA1 AB ⋅ cos(C − A) CB1 CB ⋅ cos( A − B) C A CA ⋅ cos( B − C )
sau = . Analog, = şi 1 = .
A1C AC ⋅ cos( B − A) B1 A BA ⋅ cos(C − B) C1 B CB ⋅ cos( A − C )
BA1 CB1 C1 A
Atunci, ⋅ ⋅ = 1 (unde am ţinut cont că cos(− x) = cos x, ∀x ∈ ) şi din
A1C B1 A C1 B
reciproca teoremei lui Ceva rezultă că dreptele AX, BY şi CZ sunt concurente.
Demonstraţia 2: Fie { A* } = ZY ∩ BC ,{B* } = XZ ∩ AC şi {C * } = XY ∩ AB. Deoarece ZY,
XZ, XY sunt mediatoarele segmentelor AO, BO respectiv CO din teorema lui Ayme rezultă
că punctele A* , B* , C * sunt coliniare şi din reciproca teoremei lui Desergues aplicată
triunghiurilor ABC şi XYZ rezultă că dreptele AX, BY şi CZ sunt concurente.

Observaţii:
i) Punctul de concurenţă al dreptelor AX, BY şi CZ se numeşte punctul lui Coşniţă al
triunghiului ABC.
ii) Triunghiul XYZ se numeşte triunghiul lui Coşniţă al triunghiului ABC.
iii) AX, BY, CZ se numesc dreptele lui Coşniţă.

1) Într-un triunghi ABC, punctul lui Coşniţă ( O* ) şi centrul cercului lui Euler ( O9 ) sunt
izogonal conjugate.
Demonstraţie. Fie A ' şi A " punctele de intersecţie dintre bisectoarele interioară şi
exterioară a unghiului A cu cercul circumscris triunghiului ABC şi Oa centrul cercului

139
Cesar Coaşniţă (1910-1962) – matematician roman, profesor la Universitatea din Bucureşti
140
Bertrand Russell (1872 - 1970) – filosof, logician şi matematician englez, laureat al Premiului Nobel pentru
literatură
310
circumscris triunghiului BHC. Punctele A, O9 , Oa sunt coliniare (vezi „Cercurile lui
R
Carnot”) (Fig. 315). Avem: OX = , XA ' = XO − R, OOa = AH = 2 R cos A (vezi
2 cos A
A " Oa A ' Oa
„Cercurile lui Carnot”). Atunci, = = 2 R cos A, P
A" X A' X A ''
deci fasciculul ( A, NOa MO1 ) este armonic şi cum A
m( A ' AA ") = 90° rezultă că dreptele AA ' şi AA " sunt
bisectoarele unghiurilor XAOa , respectiv Oa AP, unde H
P ∈ ( BA \ [ AB]) (vezi „Centrul cercului înscris în triunghi”), O
C
deci XAA ' ≡ A ' AOa sau XAA ' ≡ A ' AO9 (1), adică B A1 O a
dreptele AX şi AO9 sunt izogonale. Analog se arată că A'

dreptele BY şi BO9 sunt izogonale, deci punctele lui Coşniţă X


*
( O ) şi centrul cercului lui Euler ( O9 ) sunt izogonal Fig. 315
conjugate.

R
2) Raza cercului circumscris triunghiului BOC are raza egală cu .
2 cos A
R⋅ R⋅a R2 ⋅ a a R
Demonstraţie. Avem: XO = = = = .
4 ⋅ A[ BOC ] R 2 ⋅ sin 2 A 2 sin 2 A 2 cos A
4⋅
2

3) Fie A∗ , B∗ , C ∗ simetricele vârfurilor triunghiului ABC faţă de laturile opuse şi O


centrul cercului circumscris triunghiului ABC. Cercurile circumscrise triunghiurilor
AOA∗ , BOB∗ şi COC ∗ se întâlnesc într-un al doilea punct care este inversul în cercul
circumscris al punctului lui Coşniţă.
Demonstraţie. Vezi „Teorema lui Goormaghtigh - Musselman”.

Teorema lui Yiu


4) Fie A∗ , B∗ , C ∗ simetricele vârfurilor triunghiului ABC faţă de laturile opuse.
Cercurile circumscrise triunghiurilor AB∗C ∗ , BC ∗ A∗ , CA∗ B∗ trec prin inversul punctului
lui Coşniţă în cercul circumscris triunghiului ABC .
Demonstraţie. Fie Q inversul punctului lui Coşniţă (O* ) în cercul circumscris
triunghiului ABC. Din teorema lui Musselman Q aparţine cercurilor circumscrise
triunghiurilor BOB ∗ şi COC ∗ (Fig. 316). Atunci, B∗QO ≡ B ∗ BO şi
C ∗QO ≡ C ∗CO, de unde rezultă că m( B∗QC ∗ ) = m( BQO) + m( C ∗QO ) =
m ( B ∗ BO ) + m ( C ∗ CO ) = [ m ( CBB ∗ ) − m ( CBO )] + [ m ( BCC ∗ ) − m ( BCO )] =
∗ ∗
[m ( C BB ) + m ( BC C )] − [ m ( C BO ) + m ( B C O )] =
∗ ∗ ∗
m( CBB ) + m( BCC ) − (180° − m( BOC )) . Dar m( CBB ) = 90° − m( C) = 90° − m( B)
m( BOC ) = 2m( BAC ). Astfel, m( B∗QC∗ ) = (90°− m( C)) + (90°− m( B)) −180° − 2m( A) =
2m( A) − m( B) − m( C ) = 3m( A) − 180°. Deci, 180° − m( B∗QC ∗ ) = 2 ⋅180° − 3m( A).
Pe de altă parte, BAC ∗ ≡ BAC şi CAB∗ ≡ BAC , de unde rezultă că

311
m( B∗ AC ∗ ) = 2 ⋅180° − [m( BAC ∗ ) + m( BAC ) + m( CAB∗ )] = 2 ⋅180° − 3m( BAC ). Ur
mează că, m( B∗ AC ∗ ) = 180° − m( B∗QC ∗ ), de unde rezultă că Q aparţine cercului

Z
Q

C∗
Y
B∗

D
O
E F
X
B C

A∗

Fig. 316

circumscris triunghiului AB ∗C ∗ . Analog, se arată că punctul Q aparţine şi cercurilor


circumscrise triunghiurilor BC ∗ A∗ şi CA∗ B ∗ .

Observaţie: În general, dat fiind un triunghi ABC şi punctele A∗ , B∗ , C ∗ astfel încât


cercurile circumscrise triunghiurilor A∗ BC , B∗CA şi C ∗ AB au un punct comun, atunci
cercurile circumscrise triunghiurilor AB∗C ∗ , BC ∗ A∗ şi CA∗ B ∗ au de asemenea un punct
comun.

5) Centrul cercului circumscris triunghiului de simetrie A∗ B ∗C ∗ al triunghiului ABC


este simetricul centrului cercului circumscris triunghiului ABC faţă de punctul lui
Coşniţă.
Demonstraţie. Centrul cercului circumscris O∗
triunghiului podar al unui punct P este mijlocul
segmentului PP* ( P* fiind izogonalul conjugat N∗
al lui P). Fie O∗ centrul cercului circumscris
T
triunghiului A∗ B∗C ∗ (Fig. 317). Cum O9 şi
N * sunt izogonal conjugate rezultă că O ∗ este H O9 G O
imaginea mijlocului segmentului O9 N * prin
Fig. 317

312
omotetie H (G , 4) (unde am folosit teorema lui Boutte – „Triunghiul celor trei imagini”).
Fie T mijlocul segmentului O9 N * . Din teorema lui Menelaus aplicată triunghiului O9TG
O ∗G OO9 N ∗T 4 3 1
avem: ⋅ ⋅ = ⋅ ⋅ = 1 rezultă punctele O, N * şi O ∗ sunt coliniare. Cum
O∗T OG O9 N ∗ 3 2 2
O∗T HO9
= = 3 rezultă O9T O∗ H , adică O9 N * O∗ H , deci N * este mijlocul segmentului
TG O9 G
OO ∗ .

II.39. Teorema lui Kiepert


„Cum se face că matematica - produs prin excelenţă al gândirii umane, independent de
experienţă – poate fi atât de admirabil adaptat obiectelor lumii reale?” – A. Einstein141

Pe laturile unui triunghi ABC, în exteriorul său, se construiesc triunghiurile isoscele


asemenea BA ' C , AB ' C , BC ' A . Dreptele AA ' , BB ' şi CC ' sunt concurente.
Demonstraţie.
A
B'
C'

A"
C
B

A'

Fig. 318

Notăm afixele punctelor cu litere mici corespunzătoare. Fie A ' CB = α şi A " mijlocul
A 'A" a "+ c sin α b + c + 2c sin α
laturii BC. Avem sin α = , deci a ' = sau a ' = şi analog
A 'C 1 + sin α 2(1 + sin α )
a + c + 2a sinα b + a + 2b sin α
se obţin egalităţile: b' = , c' = . Sumând ecuaţiile dreptelor
2(1+ sinα) 2(1 + sin α )
AA ' , BB ' ,respectiv CC ' obţinem :
 b + c + 2c sin α   b + c + 2c sin α  a(b + c + 2sin α c ) a (b + c + 2c sin α )
a − z −a − z + − = 0.
 2(1 + sin α )   2(1 + sin α )  2(1 + sin α ) 2(1 + sin α )
Obţinem identitatea a (c − b ) + c(b − a ) + b(a − c ) = 0 , de unde rezultă că dreptele
AA ' , BB ' şi CC ' sunt concurente.

141
Albert Einstein (1879-1955) – fizician german, profesor universitar la Berlin şi Princeton, laureat al Premiului
Nobel
313
II.40. Teorema lui Gergonne142
„Infinitul este numai un fel de a vorbi.” – C. Gauss143

Dacă cevienele AD,BE şi CE sunt concurente într-un punct P interior triunghiului ABC,
atunci:
PD PE PE AP BP CP
i) + + = 1 ; ii) + + = 2.
AD BE CF AD BE CF A
Demonstraţie.
PD A[ BPD ] PE A[ APC ]
i) Avem: = , = şi
AD A[ ABC ] BE A[ ABC ] F
PF A[ APB ] P E
= , de unde prin sumare rezultă concluzia.
CF A[ ABC ]
PD AP− AP AP PE BP PF CP
ii) Cum = = 1− , = 1− , = 1− ,
AD AD AD BE BE CF CF
B D C
 AP BP CP 
prin sumare rezultă 1 = 3 −  + +  , adică Fig. 319
 AD BE CF 
AP BP CP
+ + = 2.
AD BE CF

II.41. Teorema lui Heron144


„Un matematician care nu are şi fire de poet nu poate fi niciodată un matematician complet.” - K. Weierstrass145

Fie a, b, c lungimile laturilor BC, CA respectiv AB ale unui triunghi ABC, iar 2p=a+b+c.
Aria triunghiului ABC este dată de formula A[ ABC ] = p ( p − a )( p − b)( p − c).
Demonstraţie. Fie D piciorul înălţimii din A a triunghiului ABC.
Notăm AD = ha . Din teorema lui Pitagora generalizată rezultă A

a2 + b2 − c2
c2 = a2 + b2 − 2a ⋅ DC, de unde DC = . Din triunghiul
2a
2
 a 2 + b2 − c2  B D C
dreptunghic ADC rezultă: ha2 = b 2 −   =
 2a 
Fig. 320
1
(2ab − a − b + c ) ⋅ (2ab + a + b − c ),
2 2 2 2 2 2
de unde
4a2
2 h ⋅a
ha = p( p − a )( p − b)( p − c ) şi de aici obţinem: A[ ABC ] = a p( p − a)( p − b)( p − c).
a 2

142
Joseph Gegonne (1771-1859) – matematician francez, fondator al revistei Annales de Mathématiques în 1810
143
Carl Gauss (1777-1855) – matematician, fizician şi astronom german,contribuţii în teoria numerelor, geometrie
diferenţială, analiză matematică, statistică
144
Heron (10-75) – geometru egiptean
145
Karl Weierstrass (1815-1897) – matematician german, contribuţii importante în analiza matematică
314
II.42. Teorema lui Catalan146
„Geometria este cea mai bună şi mai simplă dintre toate logicile, cea mai potrivită să dea inflexibilitate
judecăţii şi raţiunii.” - Denis Diderot147

Trei antiparalele egale relative la laturile unui triunghi determină pe laturi puncte
conciclice.
Demonstraţie. Fie A ' C ", B ' A ", C ' B " trei
antiparalele egale A ', A " ∈ ( BC ) , A
B ', B " ∈ (CA) , C ', C " ∈ ( AB ). Din
BA 'C " ≡ B AC ≡ C A " B ' rezultă
C'
C " A ' C ≡ BA " B ' ; cum A ' C " ≡ A" B ' B"
rezultă că A ' A " B ' C " este trapez isoscel,
deci inscriptibil (1), Atunci B ' C " BC , deci
dreapta C ' B " este antiparalelă cu B ' C " , de
C" B'
unde rezultă că patrulaterul B ' B " C ' C " este
inscriptibil (2). Analog, patrulaterul
A ' C " C ' B " este inscriptibil (3). Din relaţiile
(2) şi (3) rezultă că punctele
A ', C ", C ', B ", B ' sunt conciclice (4). Din B A' A" C
relaţiile (4) şi (1) rezultă că punctele
A ', C ", C ', B ", B ' C ", C ' sunt conciclice. Fig. 321

Observaţie: Cercul ce conţine punctele A ', C ", C ', B ", B ' C ", C ' se numeşte cercul lui
Taylor.

II.43. Teorema lui Blanchet


„Geometria se bazează pe această sinteză succesivă a imaginaţiei productive în generarea figurilor. Este o bază a
axiomelor care formulează condiţiile intuiţiei sensibile a priori, potrivit căreia două drepte nu pot încadra un
spaţiu.”- I. Kant148

Fie M un punct oarecare pe înalţimea Q A P


AD a triunghiului ABC, D ∈ ( BC ) şi
{E} ∈ BM ∩ AC , {F } ∈ CM ∩ AB . Să
se arate că AD este bisectoarea E
unghiului FDE .
Demonstraţie. Ducem prin A o paralelă F M
d la BC şi fie {P} = DE ∩ d ,
{Q} = FD ∩ d . Din asemănarea
B D C
triunghiurilor AFQ şi BFD, respectiv
Fig. 324

146
Éugéne Catalan (1814-1894) – matematician belgian, contribuţii în geometrie, algebră şi analiză
147
Denis Diderot (1713-1734) –filosof şi scriitor francez, figură centrală a iluminismului
148
Immanuel Kant (1724-1804) – filosof german
315
AQ AF AP AE AQ BD AF EC
AEP şi CED rezultă = şi = , de unde = ⋅ ⋅ = 1 (s-a utilizat
BD FB DC EC AP DC FB AE
teorema lui Ceva), deci AQ = AP . Cum BC PQ şi AD ⊥ BC rezultă AD ⊥ PQ . Din
relaţiile (1) şi (2) rezultă că triunghiul PDQ este isoscel,deci AD este bisectoarea unghiului
PDQ.

II.44. Teorema lui Alasia149


Un cerc intersectează laturile AB, BC, CA ale unui triunghi ABC în punctele
D, D '; E , E ', respectiv F , F '. Dreptele DE ', EF ' şi FD ' determină un triunghi A ' B ' C '
omologic cu triunghiul ABC.
Demonstraţie.Fie B"
{A '} = DE '∩ EF ',{B '} = FD '∩ EF ',
{C '} = FD '∩ DE ', {A''} = B'C'∩BC,
{B"} = A' C '∩ AC, {C''} = A' B'∩ AB.
Teorema lui Menelaus aplicată
triunghiului ABC cu transversalele
A " − D ' − F , B " − D − E ', A
respectiv C "− E − F ' ne dă: F'
A " B D ' A FC D
⋅ ⋅ = 1 ,
A '' C D 'B FA
A'
B "C E 'B D A
⋅ ⋅ = 1 , F
B " A E 'C D B
C '' A F 'C EB
⋅ ⋅ = 1 , D' B' C'
C " B F ' A EC
A"
de unde rezultă: B E E' C
A " B B "C C " A
⋅ ⋅ =
A "C B " A C " B
 D' B FA   E'C DB   F ' A EC  Fig. 322
 ⋅  ⋅ ⋅  ⋅ ⋅  =1
 D' A FC   E' B DA   F 'C ED
(cf. th. lui Carnot) şi din reciproca
teoremei lui Menelaus rezultă că
punctele A ", B ", C " sunt coliniare, C"
iar din reciproca teoremei lui
Desargues rezultă că triunghiurile ABC şi A ' B ' C ' sunt omologice.

Observaţie: Din teoremă rezultă că dreptele AA ', BB ' şi CC ' sunt concurente.

149
Cristoforo Alasia (1869-1918) –matematician italian
316
II.45. Teorema lui Ayme
„Nu există pe lume un stadiu care să pună mai armonios în acţiune facultăţile spiritului decât cel al
matematicienilor. Matematicianul trăieşte mult timp şi totuşi rămâne tânăr; aripile sale nu se frâng de timpuriu şi
porii săi nu-s obturaţi de praful ce se ridică pe marile drumuri prăfuite de vieţi obişnuite.” – James Sylvester150

Fie O centrul cercului circumscris unui triunghi ABC şi X, Y, Z punctele de intersecţie


dintre mediatoarele segmentelor OA, OB, OC cu dreptele BC, CA respectiv AB.
Demonstraţie. Fie D, E, F mijloacele segmentelor OA, OB respectiv OC. Deoarece EF este
linie mijlocie în triunghiul isoscel BOC rezultă că patrulaterul BCFE este trapez isoscel,

Z
Gi

C∗
Y
B∗

D
O
E F
X
B C

A∗

Fig. 323

deci punctele B, C, F şi E sunt conciclice (Fig. 323). Analog, punctele C, A, F şi D


respectiv A, B, D şi E sunt conciclice. Conform teoremei lui Dergiades aplicată cercurilor
precedente rezultă că punctele X, Y şi Z sunt conciclice.

Teorema lui Musselman


Fie A* , B* , C * simetricele vârfurilor triunghiului ABC faţă de laturile opuse şi O centrul
cercului circumscris triunghiului ABC. Cercurile circumscrise triunghiurilor
AOA* , BOB* , COC * se întâlnesc într-un al doilea punct.
Demonstraţie. Fie X, Y, Z centrele cercurilor considerate. Mediatoarele segmentelor OA,
OB, OC trec prin punctele X, Y respectiv Z şi conform teoremei lui Ayme punctele X, Y şi Z

150
James Sylvester (1814-1897) – matematician englez , professor la Universitatea Oxford, contribuţii importante
în algebră
317
sunt coliniare. Cum punctul O aparţine cercurilor considerate şi centrele lor sunt coliniare,
rezultă că cercurile se întâlnesc într-un al doilea punct.

Observaţii:
i) Cercurile circumscrise triunghiurilor AOA* , BOB* , COC * se numesc cercurile lui
Musselman.
ii) Al doilea punct de intersecţie dintre cercurile considerate se numeşte punctul lui Gibert
(Gi ) al triunghiului ABC.

II.46. Teorema lui Bobillier151


„Ajung tot mai mult la concluzia că necesitatea geometriei noastre nu poate
fi demonstrată...Poate că în altă viaţă vom reuşi să definim spaţiul, pentru
că acum este practic imposibil.” - Carl Gauss152

Fie M un punct în planul triunghiului N


ABC. Perpendicularele ridicate din punctul
M pe dreptele MA, MB, MC intersectează
laturile BC, CA respectiv AB în trei puncte
coliniare. A
Demonstraţie. Fie N, P şi Q punctele de
intersecţie ale dreptelor AC, BC şi AB cu
perpendicularele duse din
M pe MB, MA respectiv MC. Avem:
A[ M BP ] M
PB M B ⋅ M P ⋅ sin BM P
= = =
PC A[ M PC ] M C ⋅ M P ⋅ sin PM C
MB ⋅ sin BMP P B C
PB MB ⋅ cos AMB
sau = .
MC ⋅ sin PMC PC MC ⋅ cos AMC
NC MC cos BMC
Analog se arată că = ⋅
NA MA cos AMB Fig. 325
QA MA cos AMC
şi = ⋅ . Deci,
QB MB cos BMC
PB NC QA
⋅ ⋅ = 1 şi din reciproca teoremei
PC NA QB
lui Menelaus rezultă că punctele N, P şi Q Q
sunt coliniare.

151
Étienne Bobiller (1898-1940) –geometru francez
152
Carl Gauss (1777-1855) – matematician, fizician şi astronom german,contribuţii în teoria numerelor, geometrie
diferenţială, analiză matematică, statistică
318
II.47. Teorema lui Boutin
„O demonstraţie matematică nu înseamnă o simplă alăturare de silogisme, ci silogisme aşezate într-o anumită
ordine, iar ordinea în care sunt aşezate aceste elemente este mai importantă decât elementele însăşi.” - Henri
Poincaré153

Fie M a M b M c triunghiul median corespunzător unui triunghi ABC şi O centrul


cercului circumscris triunghiului ABC . Pe dreptele OM a , OM b , OM c se consideră
OA1 OB1 OC1
punctele A1 , B1 , C1 astfel încât = = . Dreptele AA1 , BB1 , CC1 sunt
OM a OM b OM c
concurente într-un punct ce aparţine dreptei lui Euler a triunghiului ABC .
OA1 OB1 OC1
Demonstraţie. Din = = rezultă că triunghiurile M a M b M c şi A1 B1C1 sunt
OM a OM b OM c

A
C1 B1
Mc
Mb
O
PG

B Ma C

A1

Fig. 326

asemenea, iar cum {O} = A1 M a ∩ B1 M b ∩ C1 M c rezultă că triunghiul M a M b M c şi A1 B1C1


sunt omotetice, centrul de omotetie fiind punctul O (Fig. 326). Dar şi triunghiurile
M a M b M c şi ABC sunt omotetice, centrul de omotetie fiind punctul G centrul de greutate
al triunghiului ABC . Atunci, rezultă că triunghiurile ABC şi A1 B1C1 sunt omotetice,
centrul de omotetie aparţinând dreptei determinate de celelalte două centre de omotetie -
dreapta OG - adică dreptele AA1 , BB1 , CC1 sunt concurente într-un punct ce aparţine
dreptei lui Euler a triunghiului ABC .

Observaţie: Punctul de concurenţă al dreptelor AA1 , BB1 şi CC1 se numeşte punctul lui
Franke.

153
Henri Poincaré ( 1854 -1912) – matamatician şi fizician francez, contribuţii importante în toate ramurile
matematicii
319
II.48. Teorema lui Cantor154
„Infinitul? Nici o întrebare nu a mişcat atât de profund spiritul omului.”- D. Hilbert155

Perpendicularele duse din mijloacele laturilor unui triunghi pe laturile opuse ale
triunghiului tangenţial corespunzător sunt concurente.
Demonstraţie.
Alegem ca reper complex cu originea în centrul triunghiului ABC şi fie TATBTC triunghiul
său tangenţial. Notăm cu litere mici afixele punctelor corespunzătoare. Fie A ', B ', C '
mijloacele laturilor BC, AC, respectiv AB.

TB

C" A
TC B’
C’
N O

B" C Fig. 327


A’
B A"

TA

b+c a+c a+b


Avem: a ' = ,b ' = ,c' = . Afixul centrului cercului lui Euler este egal cu
2 2 2
a+b+c b+c

a+b+c n−a' 2 2 =−1∈
n= . Deoarece = rezultă că A ' N AO.
2 o−a −a 2
Cum AO ⊥ TBTC rezultă că A ' N ⊥ TBTC . Analog, se demonstrează că şi perpendicularele
din B’ şi C’ pe laturile TATC respectiv TATB trec prin punctul N (centrul cercului lui lui
Euler al triunghiului ABC).

154
Georg Cantor (1845-1918) – matematician german, contribuţii remarcabile în teoria mulţimilor; este considerat
unul din fondatorii matematicii moderne.
155
David Hilbert (1962-1943) – matematician german, profesor la Universitatea din Göttingen, contribuţii
remarcabile în geometrie şi analiza matematică
320
II.49. Teorema Carnot156
„…dacă Dumnezeu există cu adevărat şi a creat lumea, atunci, după cum ştim cu toţii, a creat-o conform
Geometriei Euclidiene şi a înzestrat mintea umană cu concepţia a numai trei dimensiuni spaţiale. Cu toate acestea
au existat şi mai există încă matematicieni, unii chiar geniali, care se îndoiesc că întregul univers a fost creat
conform geometriei euclidiene.” – Feodor Dostoievski157

Teorema lui Carnot


Fie triunghiul ABC şi punctele A ' ∈ BC , B ' ∈ AC respectiv C ' ∈ AB . Perpendicularele
duse din punctele A ', B ', C ' pe laturile BC , AC , respectiv AB sunt concurente dacă şi
numai dacă AC '2 − BC '2 + BA '2 − CA '2 + CB '2 − AB '2 = 0 (0).
Demonstraţie.
A Presupunem că perpendicularele se
întâlnesc într-un punct P. Din teorema
lui Pitagora rezultă:
C' B' AC '2 + C ' P 2 = AP 2 (1)
BC '2 + C ' P 2 = BP 2 (2)
P BA '2 + A ' P 2 = BP 2 (3)
CA '2 + A ' P 2 = CP 2 (4)
CB '2 + B ' P 2 = CP 2 (5)
AB '2 + B ' P 2 = AP 2 (6)
B A' C

Din ecuaţiile (1), (3) şi (5) respectiv


Fig. 328 (2), (4) şi (6) prin sumare rezultă:

A C '2 + C ' P 2 + B A ' 2 + A ' P 2 + C B ' 2 + B ' P 2 = A P 2 + B P 2 + C P 2 (7), respectiv


BC '2 + C ' P 2 + CA '2 + A ' P 2 + AB '2 + B ' P 2 = BP 2 + CP 2 + AP 2 (8). Scăzând membru cu
membru relaţiile (7) şi (8) rezultă concluzia. Pentru a demonstra reciproca, fie
{P} = A ' P ∩ B ' P . Fie D piciorul perpendicularei duse din P pe latura AB. Conform primei
părţi avem AD 2 − BD 2 + BA '2 − CA '2 + CB '2 − AB '2 = 0 , care cu ipoteza dă
AD 2 − BD 2 = AC '2 − BC '2 (∗) . Fie BD = x, DC ' = y şi C ' A = z. Atunci, x + y + z = c şi
din relaţia (∗) rezultă D ≡ C '.

Consecinţe:
1) Fiecare din relaţiile următoare este echivalentă cu relaţia (0):
(9) c ⋅ AC '+ a ⋅ BA '+ b ⋅ CB ' = c ⋅ C ' B + a ⋅ A ' C + b ⋅ B ' A,
(10) 2(c ⋅ AC '+ a ⋅ BA '+ b ⋅ CB ') = c 2 + a 2 + b 2 ,
(11) AC '⋅ sin C + BA 'sin A + CB 'sin B = C ' B ⋅ sin C + A ' C sin A + B ' A sin B
Demonstraţie. Relaţia (0) este echivalentă cu
( AC '− C ' B)( AC '+ C ' B) + (BA '− A' C)(BA'+ A' C) + (CB '− B ' A)(CB '+ B ' A) = 0 sau

156
Lazare Carnot (1753-1823) – matematician şi inginer francez
157
Feodor Dostoievski (1821-1881) - scriitor rus
321
AC '⋅ c − C ' B ⋅ c + BA '⋅ a − A ' C ⋅ a + CB '⋅ b − B ' A ⋅ b = 0 (adică relaţia (9)). Relaţia (10) se
obţine din relaţia (9) astfel: 2(c ⋅ AC '+ a ⋅ BA'+ b ⋅ CB ') = 2(c ⋅ C ' B + a ⋅ A' C + b ⋅ B' A) =
= c ⋅ C ' B + a ⋅ A ' C + b ⋅ B ' A + c ⋅ AC '+ a ⋅ BA '+ b ⋅ CB ' = c 2 + a 2 + b 2 . Relaţia (11) este
echivalentă cu relaţia (9) utilizând teorema sinusurilor.

2) Fie punctele diferite A şi B. Să se determine locul geometric al punctului M din plan


care diferenţa AM 2 − BM 2 este constantă.
Demonstraţie. Fie MN ⊥ AB, {N } = MN ∩ AB . Atunci, AM 2 − AN 2 = MN 2 = BM 2 − BN 2
de unde rezultă că : A M 2 − B M 2 = A N 2 − B N 2 = co n st . Dacă M aparţine locului
geometric, atunci şi N aparţine locului geometric şi reciproc. Locul geometric este o dreaptă.

3) Mediatoarele laturilor unui triunghi sunt concurente.


Demonstraţie. Fie A ', B ', C ' mijloacele laturilor BC , AC respectiv AB ale triunghiului
ABC (Fig. 329). Din reciproca teoremei lui Carnot rezultă :
AC '2 − BC '2 + BA '2 − CA '2 + CB '2 − AB '2 = 0, deoarece BA ' = A ' C , B ' A = B ' C şi
C ' A = C 'B .

A A

B'
C' C'
B' H

O B C
A'
C Fig. 330
B A'
Fig. 329

4) Înălţimile unui triunghi sunt concurente.


Demonstraţie. Fie AA ', BB ', CC ' înălţimile triunghiului ABC (Fig. 330). Avem:
2 2 2 2 2 2 2 2 2
AB − BA ' = AA ' = AC − CA ' de unde AB − AC = BA ' − CA ' . Analog se obţin
relaţiile: AC 2 − BC 2 = AC '2 − BC '2 şi BC 2 − AB 2 = CB '2 − AB '2 . Sumând relaţiile
precedente obţinem : AC '2 − BC '2 + BA '2 − CA '2 + CB '2 − AB '2 = 0 , relaţie care arată că
înălţimile AA ', BB ', CC ' sunt concurente.

5) Perpendiculare duse în punctele de tangenţă ale cercului înscris în triunghiul ABC cu


laturile acestuia pe laturile triunghiului sunt concurente .
Demonstraţia este evidentă.

322
6) Teorema lui Soons (Existenţa ortopolului unei drepte)
Vârfurile A, B, C ale triunghiului ABC se proiectează pe o dreaptă d oarecare ce nu
trece prin vârfurile triunghiului ABC în L, M respectiv N. Perpendicularele din L pe
BC , M pe AC şi N pe AB sunt concurente într-un punct numit ortopolul dreptei d a
triunghiului ABC.
Demonstraţie. Fie AL, BM şi CN perpendicularele
C'
duse din A, B şi C pe dreapta d , ( L, M , N ∈ d ).
Fie LA ' ⊥ BC , MB ' ⊥ AC , NC ' ⊥ AB , A ' ∈ BC , B' A
N d
B ' ∈ AC , C ' ∈ AB (Fig. 331). Avem:
AM 2 − ML2 = AL2 = AN 2 − LN 2 , de unde rezultă M
L
AM 2 − AN 2 = LM 2 − LN 2 . Analog ,
2 2 2 2 2 2 2 2
BN − BL = MN − LM şi CL − CM = LN − MN .
Sumând relaţiile precedente rezultă B
AM 2 − AN 2 + BN 2 − BL2 + CL2 − CM 2 = 0 , adică A' C
( MB '2 + AB '2 ) − (C ' N 2 + AC '2 ) + (C ' B 2 + C ' N 2 ) −
( BA '2 + A ' L2 ) + ( A ' C 2 + LA '2 ) − ( B ' M 2 + B ' C 2 ) = 0 Fig. 331
egalitate echivalentă cu :
2 2 2 2 2 2
AB ' − AC ' + C ' B − BA ' + A ' C − B ' C = 0 şi din reciproca teoremei lui Carnot rezultă
concluzia.

II.50. Teoremele lui Carnot158


„există matematicieni.... care îndrăznesc să viseze că două paralele, care conform teoriei lui Euclid, nu se pot
întâlni niciodată pe pământ, se întâlnesc undeva la infinit. Eu.... am ajuns la concluzia că, din moment ce nu
înţeleg nici măcar atâta lucru, cum aş putea să - l înţeleg pe Dumnezeu? ” – Feodor Dostoievski159

Teorema lui Carnot


Într-un triunghi ascuţitunghic ABC suma distanţelor de la centrul cercului circumscris
(O) la laturile triunghiului este egală cu suma lungimilor razelor cercului înscris şi
circumscris triunghiului.
Demonstraţie. Fie A1, B1, C1, proiecţiile lui O pe A
laturile BC, CA, respectiv AB. Avem de demonstrat
faptul că: OA1 + OB1 + OC1 = r + R . Notăm cu x, y, z
B1
lungimile distanţelor OA1 , OB1 , respectiv OC1 . Din C1
teorema lui Ptolomeu pentru patrulaterul inscriptibil
OB1 AC1 rezultă: OB1 ⋅ AC1 + OC1 ⋅ AB1 = OA ⋅ B1C1 , B
c b a A1
adică y ⋅ + z ⋅ = R⋅ , de unde cy + bz = Ra. C
2 2 2
Analog se demonstrează că ax + cz = Rb şi
bx + ay = Rc . Sumând relaţiile precedente rezultă:
Fig. 332
2( x + y + z ) − (ax + by + cz ) = 2 Rp ,unde

158
Lazare Carnot (1753-1823) – matematician şi inginer francez
159
Feodor Dostoievski (1821-1881) - scriitor rus
323
a+b+c
p= . Deoarece ax + by + cz = 2 A[ ABC ] = 2rp rezultă x + y + z = R + r.
2

Observaţii:
1) Egalitatea OA1 + OB1 + OC1 = r + R se numeşte relaţia lui Carnot.
2) Dacă triunghiul ABC este obtuzunghic (de exemplu fie m( A) > 90° )), atunci teorema
lui Carnot devine − x + y + z = R + r.
3) Cum m( BOA1 ) = m( BAC ) rezultă x = R cos A şi analog rezultă
y = R cos B , z = R cos C , care înlocuite în relaţia lui Carnot dau:
r
cos A + cos B + cos C = 1 + .
R
4) O formă echivalentă a teoremei lui Carnot este:
abc( cos A + cos B + cos C )
x+ y+z = .
4 ⋅ A[ ABC ]

Consecinţă:
Dacă ABCD este un patrulater inscriptibil şi r1, r2, r3, r4, sunt razele cercurilor înscrise în
triunghiurile ABC, BCD, CDA, respectiv DAB să se arate că r1 + r3 = r2 + r4 .
Demonstraţie.
A

B
D t v x
O
u
z y

Fig. 333

Notăm cu x, y, z, t, u, şi v distanţele de la centrul cercului circumscris patrulaterului (O) la


AB, BC, CD, DA, AC, respectiv DB. Fie R raza cercului circumscris patrulaterului. Teorema
lui Carnot aplicată triunghiurilor ABC, BCD, CDA, şi DAB dă: x + y + u = R + r (1),
y + z + v = R + r (2), z + t − u = R + r (3), x + t − v = R + r (4). Din relaţiile (1) şi (2)
respectiv (3) şi (4) x + y − z − v = r1 − r2 şi z − u − x + v = r3 − r4 , de unde:
r1 − r2 + r3 + r4 = 0 , adică r1 + r3 = r2 + r4 .

324
Teorema lui Carnot
Dacă un cerc taie laturile unui triunghi în punctele D, M, E, N, F, P atunci:
DB MB EC NC FA PA
⋅ ⋅ ⋅ ⋅ ⋅ = 1.
DC MC EA NA FB PB A
Demonstraţie.
F N
Din puterea punctului faţă de un cerc avem:
ρ ( A) = AF ⋅ AP = AN ⋅ AE , ρ ( B ) = BD ⋅ BM = BP ⋅ BF ,
ρ ( B ) ρ ( C ) ρ ( A)
ρ ( C ) = CM ⋅ CD = CE ⋅ CN . Cum ⋅ ⋅ =1
ρ ( C ) ρ ( A) ρ ( B ) E
P C
DB MB EC NC FA PA M
rezultă : ⋅ ⋅ ⋅ ⋅ ⋅ = 1.
DC MC EA NA FB PB B D
Fig. 334

II.51. Teorema lui Casey160


„Ca şi în geometrie, înţeleg prin poezie o anumită simbolică pentru reprezentarea formelor posibile de existenţă.
Pentru mine poezia este o prelungire a geometriei, aşa că, rămânând poet, nu am părăsit niciodată domeniul divin
al geometriei.” – Ion Barbu

Teorema lui Casey


Fie cercurile C (O, r ) , C (O1 , r1 ) , C (O2 , r2 ) , C (O3 , r3 ) , C (O4 , r4 ) . Dacă cercurile
C1 , C2 , C3 şi C4 sunt tangente interior la cercul C (orientarea fiind în ordinea
numerotării), atunci avem următoarea relaţie între distanţele tangenţiale dintre cercuri
: d12 ⋅ d34 + d 23 ⋅ d 41 = d13 ⋅ d 24 (prin distanţă tangenţială d ij dintre cercurile Ci şi C j
înţelegem lungimea tangentei comune exterioare duse la cele două cercuri, cele două
cercuri aflându-se de aceeaşi parte a tangentei).
Demonstraţie.

T4 T3

O4 O3

O1 O2

T1 T2
A B

Fig. 335
160
John Casey (1820-1891) – matematician britanic, profesor la Universitatea Dublin, contribuţii importante în
geometrie
325
Fie T1 , T2 , T3 , T4 punctele de tangenţă ale cercurilor C1 , C2 , C3 şi respectiv C4 cu cercul C .
Din teorema cosinusului în triunghiul Ti OT j şi Oi OO j obţinem :
2r 2 − Ti T j 2
cos( Ti OT j ) = şi Oi O j 2 = (r − ri )2 + (r − rj )2 − 2(r − ri )(r − rj ) cos( Ti OT j ) sau
2r 2
TiT j 2
Oi O j 2 = (ri − rj ) 2 + (r − ri )(r − rj ) . Fie tangenta comună interioară cercurilor C (Oi , ri )
r2
şi C (O j , rj ) , d ij = AB . Din trapezul dreptunghic ABO j Oi rezultă:
2
TT
i j
dij 2 = AB2 = Oi O j 2 − (Oi A − O j B)2 = (ri − rj )2 + (r − ri )(r − rj ) − (ri − rj )2 adică
r2
TiT j 2 TiT j
dij 2 = (r − ri )(r − rj ) , de unde rezultă dij = (r − ri )(r − rj ) . Egalitatea
r2 r
d 12 ⋅ d 34 + d 23 ⋅ d 41 = d 13 ⋅ d 24 este echivalentă cu
T1T2 TT TT TT
( r − r1 )( r − r2 ) ⋅ 3 4 ( r − r3 )( r − r4 ) + 2 3 ( r − r2 )( r − r3 ) ⋅ 4 1 ( r − r4 )( r − r1 ) =
r r r r
T1T3 T2T4
(r − r1 )(r − r3 ) ⋅ (r − r2 )(r − r4 ) , adică T1T2 ⋅ T3T4 + T2T3 ⋅ T4T1 = T1T3 ⋅ T2T4 care
r r
este teorema lui Ptolemeu.

A
C3

B C2

C
C1
Fig. 336
C

326
1) Fie triunghiul ABC înscris în cercul C şi cercurile C1 ,C2 ,C3 tangente interioare
cercului C şi laturilor BC, CA respectiv AB, astfel încât A şi C1 , B şi C2, C şi C3 să fie de
părţi diferite faţă de BC, CA, respectiv AB. Notăm cu l1 , l2 , l3 lungimile tangentelor din A,
B, C la cercurile C1 ,C2 respectiv C3 şi prin tij lungimea tangentei comune exterioare a
cercurilor Ci şi Cj , i, j = 1,3, i ≠ j .Atunci: t12 = t23 = t31 dacă şi numai dacă
b+c c+a a+b
l1 = ,l2 = ,l3 =
2 2 2
Demonstraţie. Cercurile C1 ,C2 ,C3 sunt tangente laturilor triunghiului ABC în mijlocul
acestora. Aplicând teorema lui Casey pentru cercurile: C şi A,C2, C1, C3 ; C şi B, C3 ,C1 ,C2;
C şi C, C1 ,C3 ,C2 se obţin egalităţile:
b c c a a b
⋅ d13 + ⋅ d12 = l1 ⋅ d 23 ; ⋅ d12 + ⋅ d 23 = l2 ⋅ d13 ; ⋅ d 23 + ⋅ d13 = l3 ⋅ d12 , (∗) de unde
2 2 2 2 2 2
b+c c+a a+b b+c c+a a+b
rezultă l1 = , l2 = , l3 = . Reciproc, dacă l1 = , l2 = , l3 = prin
2 2 2 2 2 2
înlocuirea acestora în relaţiile (∗) rezultă b(d13 − d 23 ) = c(d 23 − d12 ),
c(d12 − d13 ) = a (d13 − d 23 ) , a (d 23 − d12 ) = b(d12 − d13 ) şi de aici:
d12 − d13 d 23 − d12 d13 − d 23 0
= = = , de unde d12 = d 23 = d31 .
a b c a+b+c

2) În triunghiul ABC , fie C1 şi C2 două cercuri tangente exterior în punctul I, tangente


laturii BC a ABC şi tangente interior cercului circumscris triunghiului ABC . Să se
arate că punctul I este centrul cercului înscris în triunghiul ABC .

c b
z

Fig. 337

B X α D α Y C

327
Demonstraţie. Fie C1 I BC ={X} , C2 I BC ={Y} , {D}=AI I BC . Fie AI = z ,
BX = x , CY = y , DX = DY = DI = α . Din teorema lui Casey aplicată cercurilor
( A, C1 , B, C ) şi respectiv ( A, C2 , C , B ) ne dă : az + bx = c(2α + y ) (1) şi
x +α c
az + cy = b(2α + x ) (2), de unde bx − cy = α (c − b) adică = , relaţie echivalentă cu
y +α b
BD AB ac
= , ceea ce implică că AI este bisectoarea unghiului A şi BD = . Din (1) şi
DC AC b+c
z b+c AI AB
(2) rezultă az = α (b + c) , de unde = , adică = , egalitate care arată că BI
α a ID BD
este bisectoarea ABC , deci I este centrul cercului înscris în triunghiul ABC .

TEOREMA LUI CASEY ( T. GENERALIZATĂ A LUI PTOLEMEU)


Prin tij vom nota lungimea tangentei comune exterioare cercurilor C1 şi C2 (cele două
cercuri se afla de aceaşi parte a tangentei). Dacă cercurile C1 , C2 , C3 şi C4 toate tangente
interior (sau exterior) unui cerc C în ordinea ciclică C1 , C2 , C3 , C4 , atunci
t12 ⋅ t34 + t23 ⋅ t41 = t13 ⋅ t24 . Mai mult dacă există relaţiile ±t12 ⋅ t34 ± t14 ⋅ t23 ± t13 ⋅ t24 = 0 pentru
o anumită alegere a semnelor + sau - , există un cerc tangent (interior sau exterior) tuturor
celor 4 cercuri.

II.52. Teorema lui Clairaut161


„Matematicianul este îmblânzitorul ce a domesticit infinitul.” – Lucian Blaga162

Pe laturile AB şi AC ale unui triunghi ABC sc construiesc în exterior (sau în interior)


paralelogramele AA1 B1 B şi AA2C1C . Fie {M}= A1 B1 ∩ A2C1 , { A3 }= MA ∩ BC ; construim
punctul A4 astfel încat A3 ∈ ( AA4 ) , A3 A4 =AM şi paralelogramul BCC2 B2 cu
BB2 A3 A4 şi BB2 ≡ A3 A4 (Fig. 338).

Teorema lui Clairaut


Suma ariilor paralelogramelor AA1 B1 B şi AA2C1C este egală cu aria paralelogramului
BCC2 B2 .
Demonstraţie. Fie {B3 } = BB2 ∩ A1 B1 şi { C3 }= CC2 ∩ A2 C1 . Din
AM= BB2 = BB3 = CC2 = CC3 , MB3 = AB şi MC3 = AC rezultă că: A[ AA1B1B ] = A[ BB3 MA] ,
A[ AA2 C1C ] = A[ CC3 MA ] , A[ B B3 C 3 C ] = A[ B B 2 C 2 C ] şi A[ MB3C3 ] = A[ ABC ] . Avem:
A[ A A1 B1 B ] + A[ A A2 C 1 C ] = A[ B B 3 M A ] + A[ C C 3 M A ] = A[ B C C 3 M B 3 ] − A[ A B C ] =
A[ MB3C 3 ] + A[ BB3C 3 C ] − A[ ABC ] = A[ BB3C 3 C ] = A[ BB2 C 2 C ] .

161
Claude Clairaut (1713-1765) – matematician, fizician francez, membru al Academiei Franceze, contribuţii în
analiza matematică şi geometrie
162
Lucian Blaga (1895-1961) - filozof, umanist, jurnalist, poet, dramaturg, traducător, profesor universitar şi
diplomat român, membru titular al Academiei Române
328
Observaţie: O consecinţă a teoremei lui Clairaut este teorema lui Pitagora.

A2

A1 A

B3 C3

C1
A3

B C
B1

B2 A4 C2

Fig. 338

Astfel, dacă m( BAC ) = 90° , AA1 B1 B şi AA2C1C sunt pătrate şi MA = A3 A4 = BC


(Fig. 339). Din teorema lui Clairaut avem: A[ A A1 B1 B ] + A[ A A 2 C 1C ] = A[ B B 2 C 2 C ]
relaţie echivalentă cu: AB 2 + AC 2 = BC 2 .

C2

C1 C
A4
B2
C3
A3
A
A2
B
Fig. 339

M B1
B3 A1

329
II.53. Teorema lui Mathieu 163

„Istoria ne arată, că viaţa este doar un episod între două veşnicii ale morţii şi în acest episod gândirea conştientă
durează doar o clipă. Gândirea este doar o explozie de lumină în mijlocul unei nopţi lungi, dar această explozie
este totul.” - Henri Poincaré164

Într-un triunghi izogonalele a trei ceviene concurente sunt la rândul lor concurente.
A

B'
C'

M'
C"
M '' B"

B C
A' A"
Fig. 340

Demonstraţie. Fie triunghiul ABC şi cevienele AA ', BB ', CC ' concurente în punctul M ' .
Fie AA ", BB ", CC " ( A'' ∈ BC , B'' ∈ AC , C'' ∈ AB ) izogonalele dreptelor AA ', BB ' ,
respectiv CC ' (Fig. 340). Atunci, m( BAA ') = m( A " AC ), m( B ' BA) = m( B " BC ) şi
m( ACC ') = m(C " CB). Din forma trigonometrică a teoremei lui Ceva aplicată pentru
sin BAA' sin ABB ' sin ACC '
cevienele concurente în M rezultă: ⋅ ⋅ =1 sau
sin A'AC sin B ' BC sin C ' CB
sin A '' AC sin B '' BC sin C '' CB
⋅ ⋅ = 1 şi din reciproca teoremei lui Ceva rezultă că
sin BAA " sin ABB '' sin ACC ''
izogonalele AA'',B B '',C C '' sunt concurente într-un punct M ".

Observaţie : Punctele M ' şi M " se numesc puncte izogonale.

163
Claude Mathieu (1783-1875) – matematician francez, profesor la Ecole Polytechnique din Paris
164
Henri Poincaré ( 1854 -1912) – matematician şi fizician francez, contribuţii importante în toate ramurile
matematicii
330
II.54. Teorema lui Miquel
„Dincolo de pamânt şi infinit
Cătam să aflu cerul unde vine.
Şi-un glas solemn atunci s-a auzit
Şi cerul şi infernul sunt în tine.”
Omar Khayyam165
Teorema lui Miquel
Pe laturile triunghiului ABC se consideră punctele necoliniare D, E, F
( D ∈ BC , E ∈ CA, F ∈ AB ) . Să se arate că cercurile circumscrise triunghiurilor AEF,
BFD, CDE au un punct comun P.
Demonstraţie. Fie P punctul comun cercurilor A
circumscrise triunghiurilor BDF şi DCE. Deoarece
patrulaterele FBDP şi CEPD sunt inscriptibile E
rezultă: m( FPE) = 360° − m(FPD) − m( DPE) = F
P
360° − [180° − m( B)] − [180° − m(C )] = 180° − m( A).
deci patrulaterul FPEA este inscriptibil, adică
punctul P aparţine cercului circumscris
B C
triunghiului AFE. D

Observaţii: Fig. 341


1) Punctul P de concurenţă a celor 3 cercuri se
numeşte punctul pivot al triunghiului DEF.
2) Triunghiul DEF se numeşte triunghiul lui Miquel .
3) Cercurile circumscrise triunghiurilor AFE, BFD, CDE se numesc cercurile lui Miquel.
4) Din teorema lui Miquel rezultă m( FPE ) = 180° − m( A), m( FPD ) = 180° − m( B ),
m( DPE ) = 180° − m(C ) .
5) Fie P un punct şi Pa Pb Pc triunghiul său cevian în raport cu triunghiul ABC. Cercurile
circumscrise triunghiurilor APb Pc , BPa Pc şi CPa Pb se intersectează într - un punct MP
numit punctul pivot asociat lui P.

1) Coordonatele unghiulare ale punctului pivot P sunt:


m( EDF ) + m( A), m( DEF ) + m( B), respectiv m( EFD) + m(C ).
Demonstraţie. Avem: m (BPC ) = m (BPD ) + m (DPC ) = m (BFD ) + m (DEC ) =
[180° − m( BDF ) − m( B)] + [180° − m( EDC ) − m(C )] = 360° − [m( BDF ) + m( EDC )] − [m( B ) + m(C )] =
360° − [180° − m( EDF )] − [180° − m( A)] = m( EDF ) + m( A). Analog se arată că
m(CPA) = m( FED ) + m( B ) şi m( APB) = m( EFD) + m(C ) .

2) În triunghiul ABC fie punctele D, D ' ∈ [ BC ], E , E ' ∈ [CA], F , F ' ∈ [ AB ]. Dacă P şi P '
sunt punctele pivot ale triunghiului DEF, respectiv D ' E ' F ' atunci punctele P şi P '
coincid dacă şi numai dacă triunghiurile DEF şi D ' E ' F ' sunt asemenea.

165
Omar Khayyam (1048-1122) – matematician, poet, filosof, astronom persan, contribuţii în algebră şi geometrie
331
Demonstraţie. Avem: m( BPC ) = m( BPD ) + m( DPC ) = m ( BF D ) + m ( D E C ) =
[180° − m( DFE ) − m( EFA)] + [180° − m( DEF ) − m( FEA)] = m( EDF ) + m( BAC ) . Analog se
arată că m(CPA) = m( DEF ) + m( ABC ) , m( APB) = m( EFD ) + m( ACB ) (Fig. 342). Dacă
punctele P şi P ' coincid, atunci m( BPC ) = m( BP ' C ) , de unde m( EDF ) + m( PAC ) =
m( E ' D ' F ') + m( BAC ) , adică m( EDF ) = m( E ' D ' F ') şi analoagele de unde rezultă că
triunghiurile DEF şi D ' E ' F ' sunt asemenea. Dacă triunghiurile DEF şi D ' E ' F ' sunt
asemenea atunci, m ( BPC ) = m ( BP ' C ), m ( APB ) = m ( AP ' B ), m ( APC ) = m ( AP ' C ),
deci P coincide cu P ' .

A A

P3 E P2
E'
F F P
E
F' xP

B
D B C
D' C P1 D

Fig. 342 Fig. 343

3) Triunghiul podar al punctului pivot P al triunghiului DEF este asemenea cu


triunghiul DEF.
Demonstraţie: Fie P1 P2 P3 triunghiul podar al punctului pivot P (Fig. 343). Deoarece
patrulaterele AP2 PP3 , BP1 PP3 şi CP1 PP2 sunt inscriptibile rezultă că punctul pivot al
triunghiului P1 P2 P3 este tocmai punctul P şi conform teoremei precedente rezultă că
triunghiurile P1 P2 P3 şi DEF sunt asemenea.

4) Centrul cercului circumscris (O) al triunghiului ABC este punctul pivot asociat al
centrului de greutate (G) al triunghiului ABC .
Demonstraţie. Dacă M a M b M c este triunghiul median al triunghiului ABC , atunci
patrulaterele AM c OM b , BM a OM c , M a CM b O sunt inscriptibile, deci O este punctul pivot
asociat al lui G.

5) Ortocentrul (H) al triunghiului ABC este punctul pivot asociat tot al lui H
Demonstraţie: Dacă H a H b H c este triunghiul ortic al triunghiului ABC ,atunci
patrulaterele HH b AH c , H c HH a B , HH b CH a sunt inscriptibile, deci H este punctul pivot
asociat al lui H.

332
6) Centrul cercului înscris (I) în triunghiul ABC este punctul pivot asociat al punctului
lui Gergonne ( ) al triunghiului ABC .
Demonstraţie. Dacă Ca Cb Cc este triunghiul de contact al triunghiului ABC , atunci
patrulaterele ACc ICb , BCa ICc , CCb ICa sunt inscriptibile, deci cercurile circumscrise
triunghiurilor ACc Cb , BCa Cc şi CCb Ca se interesectează în I şi cum
{Γ} = ACa I BCb I CCc rezultă concluzia.

7) Dreptele ce unesc punctul pivot (M) asociat unui punct P cu picioarele cevienelor lui P
intersecteaza laturile triunghiului ABC sub acelaşi unghi.
Demonstraţie. Fie Pa Pb Pc triunghiul cevian al punctului P în raport cu triunghiul ABC .
Deoarece patrulaterele M Pa C Pb , M Pb A Pc , M Pc B Pa sunt inscriptibile rezultă că
m( MPa C ) = m( MPb A) = m( MPc B) .

II.55. Teorema lui Sawayama - Thebault


„Matematica e arta de a gândi prin teoreme.” - Ion Barbu166

Fie D un punct pe latura BC a triunghiului ABC, O ' centrul unui cerc (C’) tangent
dreptelor AD, DC şi cercului (C) circumscris triunghiului ABC în punctele M, N respectiv
P centrul cercului înscris (I) în triunghiul ABC aparţine dreptei MN.
Demonstraţie. Fie M ' şi N ' punctele de intersecţie dintre MP şi PN cu cercul circumscris
triunghiului ABC iar {J } = AN '∩ MN . Fie N " punctul de intersecţie dintre mediatoarea
segmentului BC şi dreapta NP, iar O centrul cercului circumscris triunghiului ABC (Fig.
344). Atunci, O ' N ON " de unde rezultă că O ' NP ≡ ON " P (1). Dar
O ' NP ≡ O ' PN (2), triunghiul O ' NP
fiind isoscel O ' P ≡ O ' N . Din relaţiile (1) şi
(2) rezultă OPN ≡ ON " P, deci triunghiul A
OPN " este isoscel. Atunci ON " ≡ OP(= R),
adică punctul N" aparţine cercului
circumscris triunghiului ABC, deci punctele
N " şi N ' coincid. Punctul N ' este astfel O O' P
mijlocul arcului BC, deci PN este bisectoarea
J M
unghiului BPC (3) şi AN ' este bisectoarea C
unghiului BAC. Deoarece cercurile (C) şi
M' D N
(C’) sunt tangente interior în punctul P, B
MN M ' N ', de unde rezultă că N'
 1 
MJA ≡ M ' N ' A ≡ M ' PA ≡ MPA = m(M ' A) 
 2  Fig. 344
deci patrulaterul MAPJ este inscriptibil (4).

166
Ion Barbu (1895-1961) – matematician român, profesor la Universitatea din Bucureşti, contribuţii în algebră şi
geometrie
333
Din teorema lui Miquel aplicată triunghiului AMJ (cu M ∈ AM , J ∈ AJ şi N ∈ MJ )
rezultă că cercul circumscris triunghiului NPJ este tangent dreptei AJ în J (5). Cercul cu
centrul în N ' şi raza BN trece prin centrul cercului înscris (I) în triunghiul ABC (vezi
„Cercuri exînscrise”). Deoarece m( BPN ) = m( N ' AC ) = m( N ' BC ) rezultă că cercul
circumscris triunghiului BNP este tangent dreptei BN ' în B, de unde rezultă că cercul
C ( N ', N ' B) este ortogonal cercului circumscris triunghiului BNP, în consecinţă şi cercului
circumscris triunghiului MNP. Cum cercul C ( N ', N ' B) este ortogonal şi cercului
circumscris triunghiului JNP rezultă că N ' J = N ' I , deci J ≡ I , adică I aparţine dreptei
MN.

Teorema lui Sawayama – Thebault


Fie punctul D pe latura BC a triunghiului ABC, I centrul cercului înscris în acest
triunghi, C 1 (O1 , r1 ) un cerc tangent interior cercului circumscris triunghiului ABC şi
segmentelor AD şi BD, iar C 2 (O2 , r2 ) un cerc tangent interior cercului circumscris
triunghiului ABC şi segmentelor AD şi CD. Punctele O1 , I şi O2 sunt coliniare.
Demonstraţie. Fie O1 N ⊥ BC , O2 N ' ⊥ BC ( N , N ' ∈ BC ) de unde O1 N O2 N '. Conform
teoremei de mai sus dreptele MN şi M ' N ' trec prin I (unde M şi M ' sunt punctele de
tangenţă ale cercurilor C 1 şi C 2 cu AD) (Fig. 345). Triunghiurile DMN şi O1 NM fiind
isoscele rezultă că DO1 este mediatoarea segmentului MN şi bisectoarea unghiului
MDN . Analog DO2 este bisectoarea unghiului N ' DM ', deci DO1 ⊥ DO2 , de unde
DO1 N ' M ' şi DO2 MN . Conform teoremei lui Pappus aplicată hexagonului
O1 NIO2 DO1 , punctele O2 , I şi O1 sunt coliniare.

Consecinţe:
O1 I
1) Dacă m( ADC ) = 2θ , atunci = tg 2θ .
IO2
Demonstraţie. Fie Ca proiecţia lui I pe
O1 I NCa A
BC. Avem: = (1). Deoarece
IO2 Ca N '
DO1 este bisectoarea unghiului ADN M'
O2
rezultă m( O1 DN ) = θ şi cum I
N ' I DO1 rezultă m( IN ' N ) = θ .
O1
ICa B
Atunci, tgθ = , deci N' M
N ' Ca Ca
N ' Ca = rctgθ (2). Iar în triunghiul
D N
r C
IC a N : tg (90 ° − θ ) = ctg θ = ,
Ca N
deci Ca N = rtgθ (3). Din relaţiile (1),
O1 I
(2) şi (3) rezultă = tg 2θ . Fig. 345
IO2

334
2) r = r1 cos 2 θ + r2 sin 2 θ .
Demonstraţie. Avem: N ' N = N ' Ca + Ca N = r (ctgθ + tgθ ) şi
r1ctgθ + r2 tgθ
N ' N = ND + DN ' = r1ctgθ + r2 tgθ , deci r = = r1 cos 2 θ + r2 sin 2 θ .
ctgθ + tgθ

3) În triunghiul ABC , fie C1 şi C2 două cercuri tangente exterior în punctul I, tangente


laturii BC a ABC şi tangente interior cercului circumscris triunghiului ABC . Să se
arate că punctul I este centrul cercului înscris în triunghiul ABC .
Demonstraţia rezultă din teorema lui Sawayama – Thebault.

4) Fie patrulaterul inscriptibil ABCD şi C1 ( I1 , r1 ), C2 ( I 2 , r2 ), C3 ( I 3 , r3 ), C4 ( I 4 , r4 ) cercurile


înscrise în triunghiurile BCD, CDA, DAB, respectiv ABC. Atunci, patrulaterul I1 I 2 I 3 I 4
este dreptunghi şi r1 + r3 = r2 + r4 .
Demonstraţie.

D C
O3

I2 I1

O4 E
O2
I3 I4

O1
A
B

Fig. 346

Fie AC I BD = {E} , C '1 (O1 , ρ1 ), C '2 (O2 , ρ 2 ), C '3 (O3 , ρ3 ), C '4 (O4 , ρ 4 ) cercurile tangente
cercului circumscris triunghiului ABC şi laturilor AE, EB; BE, CE; CE, DE; respectiv DE
şi AE. Fie m( AEB) = m( CED) = θ , de unde m( AED) = m( CEB) = π − θ (Fig.
346). Conform teoremei lui Thébault rezultă I1 ∈ O2 O3 , I 2 ∈ O3O4 , I 3 ∈ O4 O1 , I1 ∈ O1O2 şi
O4 I 3 O2 I1  π −θ  2 θ O1 I 4 O3 I 2 θ
= = tg 2   = ctg 2 , = = tg 2 . Analog se arată că
I 3O1 I1O3  2  I 4 O2 I 2 O4 2

335
O4 I 3 O2 I 4 O1 I 4 O3 I1 O2 I1 O4 I 2 O3 I 2 O1 I 3
= , = , = , = , de unde rezultă
I 3O1 I 4 O1 I 4 O2 I1O2 I1O3 I 2 O3 I 2 O4 I 3O4
I 3 I 4 O2 O4 , I1 I 4 O1O3 , I1 I 2 O2 O4 , I 2 I 3 O1O3 . Deoarece O1O3 ⊥ O2 O4 ( O1O3 şi O2 O4
sunt bisectoarele unghiurilor DEC , respectiv CEB ) rezultă că patrulaterul I1 I 2 I 3 I 4
θ θ
este paralelogram. Avem: r1 + r3 = ( ρ1 + ρ3 ) cos 2 + ( ρ 4 + ρ 2 )sin 2 = r2 + r4 .
2 2

II.56. Teorema lui Schooten167


„Atâtea claile de fire stângi!
Găsi-vor gest închis, să le rezume,
Să nege, dreapta, linia ce frângi:
Ochi în virgin triunghi taiat spre lume?”
Ion Barbu168
Dacă M este un punct situat pe arcul BC al cercului circumscris triunghiului echilateral
ABC , atunci AM = BM + CM .
Demonstraţie. A
Soluţia 1. Fie D ∈ ( AM ) astfel încât MD ≡ BD .
Deoarece m( ACB) = m( AMB) = 60° rezultă că
triunghiul MBD este echilateral, deci MD ≡ BM .
Deoarece AB ≡ BC , BD ≡ BM şi ABD ≡ CBM
rezultă că ∆ABD ≡ ∆CBM , de unde AD ≡ MC. D
Atunci, AM = AD + DM = MC + MB.
B C
Soluţia 2. Din prima teorema a lui Ptolemeu
rezultă: AM ⋅ BC = AB ⋅ MC + AC ⋅ BM , adică
AM = MC + MB ( deoarece AB ≡ BC ≡ AC ) M
Fig. 347

Generalizarea teoremei lui Schooten


Fie ABC un triunghi echilateral. Pentru orice punct M din plan are loc relatia
MB ≤ MA + MC , cu egalitatea dacă şi numai dacă punctul M aparţine cercului
circumscris triunghiului ABC.
Demonstraţie. Avem MB ≤ MA + MC ⇔ MB ⋅ AC ≤ MA ⋅ BC + MC ⋅ AB, care reprezintă
inegalitatea lui Ptolemeu .

1
Frans van Schooten (1615 – 1660) – matematician olandez, promotor al geometriei carteziene
168
Ion Barbu (1895-1961) – matematician român, profesor la Universitatea din Bucureşti, contribuţii în algebră şi
geometrie
336
II.57. Teorema lui Smarandache169
„Noi ştim că unu ori unu fac unu,
dar un inorog ori o pară
nu ştim cât face.
Ştim că cinci fără patru fac unu,
dar un nor fară o corabie
nu ştim cat face.” – N Stănescu170

Fie H a , H b , H c picioarele înălţimilor unui triunghi ascuţitunghic ABC. Dacă a ', b ', c '
sunt lungimile laturilor triunghiului podar Ha Hb Hc , atunci
4 ( a ' b '+ a ' c '+ b ' c ') ≤ a 2 + b 2 + c 2 , unde a, b, c reprezintă lungimile laturilor triunghiului
ABC.
Demonstraţie.
Lemă: Daca p şi p ' sunt semiperimetrele triunghiurilor
A
p
ABC şi H a H b H c atunci p ' ≤ .
2
Hb Demonstraţie: Avem: AH c = bcosA, AH b = ccosA, de
Hc unde Hb Hc2 = AHb2 + AHc2 − 2AHb ⋅ AHc cos A = a2 cos2 A ,
adică H b H c = a cos A . Analog, H a H c = b co s B şi
a '+ b ' + c '
B H a H b = c cos C (Fig. 348). Astfel, p ' = =
Ha 2
C a cos A + b cos B + c cos C
sau
Fig. 348 2
R ( sin 2 A + sin 2 B + sin 2C )
p' = = 2 R sin A sin B sin C
2
(unde am utilizat teorema sinusurilor, R fiind raza cercului circumscris triunghiului ABC ),
a b c A[ ABC ]
deci: p ' = 2 R ⋅ ⋅ = , (unde A[ ABC ] reprezintă aria triunghiului ABC). Cum
2R 2R 2R R
r p
A[ ABC ] = r ⋅ p , (r–raza cercului înscris în triunghiul ABC) rezultă: p ' = ⋅ p ≤ (unde am
R 2
utilizat inegalitatea lui Euler 2r ≤ R).
Demonstraţia teoremei lui Smarandache. Utilizând inegalităţile cunoscute:
3( x y + x z + y z ) ≤ ( x + y + z ) 2 ≤ 3( x 2 + y 2 + z 2 ), ∀ x , y , z ∈  rezultă
1 ( 2 p ')
2
p2 (a + b + c ) 1 2 2 2
2
a ' b '+ a ' c '+ b ' c ' ≤ ( a '+ b '+ c ') 2 = ≤ = ≤ ( a + b + c ).
3 3 3 3⋅4 4

169
Florentin Smarandache (1954 - ) - matematician român, profesor la Universitatea New-Mexico contribuţii în
teoria numerelor şi statistică matematică
170
Nichita Stănescu (1933 – 1983) – eseist, poet român, ales postum membru al Academiei Române
337
II.58. Teorema lui Snapper
„...poezia nu este lacrimă
ea este însuşi plânsul,
plânsul unui ochi neinventat,
lacrima ochiului
celui care trebuie să fie frumos,
lacrima celui care trebuie să fie fericit.”
Nichita Stănescu171

Fie un punct oarecare Q în planul triunghiului ABC, M a , M b , M c mijloacele laturilor


BC,AC respectiv AB. Fie (a) o dreaptă ce trece prin M a şi este paralelă cu AQ, (b) o
dreaptă ce trece prin M b paralelă cu BQ şi ( c) o dreaptă ce trece prin M c paralelă cu
CQ. Atunci :
i) dreptele a,b,c sunt concurente într-un punct P;
ii) centrul de greutate G al triunghiului ABC se află pe dreapta PQ astfel încât
2PG=GQ.

c Mb Q
Mc
P
b

B Ma C
a
Fig. 349

 1
Demonstraţie. Prin omotetia de centru G şi raport −  triunghiul ABC se transformă în
 2

triunghiul M a M b M c . Omotetia H  G ,− 1  transformă dreptele AQ,BQ,CQ în dreptele a,b,c


 2
respectiv paralele. Cum dreptele AQ,BQ,CQ sunt concurente în Q rezultă că dreptele a,b,c
dreptele a,b,c sunt concurente şi fie P acest punct. Cum prin omotetia de centru G şi raport
 1
uuur 1 uuur
 −  punctul Q se transformă în punctul P rezultă : GP = − GQ de unde rezultă
 2 2
concluzia.

171
Nichita Stănescu (1933 – 1983) – eseist, poet român, ales postum membru al Academiei Române
338
II.59. Teorema lui Urquhart - Pedoe
„Matematica va fi limba latină a viitorului, obligatorie pentru toţi oamenii de ştiinţă,tocmai pentru că matematica
permite accelerarea maximă a circulaţiei ideilor ştiinţifice.” – Grigore Moisil172

În triunghiul ABC fie transversala B '− D − C ' astfel încât B ∈ ( B ' A) , D ∈ ( BC ) ,


C ' ∈ ( AC ) . Să se arate că AB + BD = AC '+ C ' D dacă şi numai dacă
AB '+ B ' D = AC + CD .
Demonstraţie. Vom demonstra mai întîi teorema:

Lema lui Breusch


Fie triunghiurile A1 B1C1 şi A2 B2 C2 astfel încât m( A1 ) = 2α1 , m( A2 ) = 2α 2 , m( B1 ) = 2 β1 ,
m( B2 ) = 2β 2 , m(C1 ) = 2γ 1 , m(C2 ) = 2γ 2 şi B1C1 = B2 C2 . Atunci p( A1 B1C1 ) ≥ p( A2 B2 C2 )
dacă şi numai dacă tg β1 ⋅ tgγ 1 ≥ tg β 2 ⋅ tgγ 2 , unde am notat cu p( XYZ ) perimetrul
triunghiului XYZ .
Demonstraţie.
A
p ( A1 B 1 C 1 ) A B + A1 C 1
= 1+ 1 1 =
B1 C 1 B1C 1
2β ' 2γ
sin2γ1 +sin2β1 2sin(γ1 + β1)⋅ cos(γ1 − β1)
1+ =1+ =
sinα1 2sin(γ1 + β1)⋅ cos(γ1 + β1)
C'
2cos γ 1 ⋅ cos β1 2
= =
cos γ 1 ⋅ cos β1 − sin γ 1 ⋅ sin β1 1 − tgγ 1 ⋅ tg β1 2γ '

(Fig. 350) şi analog,
B D C
p ( A2 B 2C 2 ) 2
= .
B2C 2 1 − tg γ 2 ⋅ tg β 2 Fig. 350
Condiţia p( A1 B1C1 ) ≥ p( A2 B2 C2 )
este echivalentă cu
B'
2 2
≥ adică cu
1 − tgγ 1 ⋅ tg β1 1 − tgγ 2 ⋅ tg β 2
tgγ 1 ⋅ tg β1 ≥ tgγ 2 ⋅ tg β 2 .
Demonstraţia teoremei. Fie m( BAD) = 2 β ' , m( BDA) = 2 β , m( DAC ') = 2γ ,
m( ADC ') = 2γ ' . Din lema lui Breusch rezultă că p( AB ' D) = p( ACD ) dacă şi numai dacă
tgβ '⋅ tg(90°−γ ') = tgγ ⋅ tg(90°− β) adică tg β '⋅ ctg γ ' = tg γ ⋅ ctg β sau tg β '⋅ tg β = tgγ ⋅ tg γ ' ,
condiţie echivalentă cu p( ABD) = p( AC ' D) , ceea ce trebuia demonstrat.

172
Grigore Moisil (1906-1973) – matematician român, profesor la Universitatea din Iaşi, membru al Academiei
Române
339
II.60. Relaţii metrice în triunghiul dreptunghic
„Lumea este o imensă problemă matematică. Dumnezeu este geometrul atotputernic care pune problema şi o
rezolvă.” – Gottfried Leibniz173

Teorema înălţimii: Într-un triunghi dreptunghic lungimea înălţimii duse din vârful
unghiului drept este medie proporţională între lungimile catetelor pe
ipotenuză.
Demonstraţie.
A
Fie triunghiul ABC cu m( BAC ) = 90° şi D piciorul înălţimii
duse din A pe latura BC. Deoarece ADB ≡ ADC şi
BAD ≡ ACD rezultă că triunghiurile ABD şi CAD să fie
AD BD
B D C asemenea, de unde = , adică AD 2 = BD ⋅ DC.
DC AD
Fig. 351

Reciproca teoremei înălţimii: Fie D ∈ ( BC ) proiecţia vârfului A al triunghiului ABC pe


latura BC. Dacă AD 2 = BD ⋅ DC , atunci triunghiul ABC
este dreptunghic.
AD BD
Demonstraţie. Deoarece ADB ≡ CDA şi = rezultă că triunghiurile ABD şi CAD
DC AD
sunt asemenea, de unde avem ABD ≡ DAC şi BAD ≡ ACD . Dar
m( ABC ) + m( BAC ) + m( ACB) = 180°
Adică 2  m( ABC ) + m( ACB )  = 180° , de unde rezultă
A  
m( ABC ) + m( ACB) = 90° şi de aici
m( BAC ) = 180° − 90° = 90°, adică triunghiul ABC este
C dreptunghic.
D B
Observaţie: Dacă D ∈ ( BC ) \ BC , atunci teorema reciprocă nu
Fig. 352 mai este adevărată. Se observă că triunghirile DAB şi DCB sunt
asemenea, de unde rezultă că AD 2 = BD ⋅ DC , dar triunghiul
ABC nu este dreptunghic. Deci, condiţia ca D ∈ ( BC ) este esenţială pentru ca triunghiul
ABC să fie dreptunghic.
A
Teorema catetei: Într-un triunghi dreptunghic lungimea
unei catete este medie proporţională
între lungimile ipotenuzei şi a proiecţiei
acestei catete pe ipotenuză.
B D C

Fig. 353

173
Gottfried Leibniz (1646-1716) – matematician şi filosof german, contribuţii importante în analiza matematică şi
algebră
340
Demonstraţie.Fie D proiecţia vârfului A al triughiului ABC ( m( BAC ) = 90° ) pe ipotenuza
BC. Deoarece BAD ≡ ACB şi ADB ≡ BAC rezultă că triunghiurile ABD şi CBA sunt
BD AB
asemenea, de unde = , adică Analog se arată că AC 2 = BC ⋅ DC.
AB BC

Reciproca teoremei catetei: Fie D ∈ ( BC ) proiecţia vârfului A al triunghiului ABC pe


latura BC. Dacă AB 2 = BC ⋅ BD (sau AC 2 = BC ⋅ DC )
atunci triunghiul ABC este dreptunghic.
AB BD
Demonstraţie. Deoarece ABD ≡ ABC şi = rezultă că triunghiurile ABD şi CAB
BC AB
sunt asemenea, de unde BDA ≡ BAC (1). Analog, din asemănarea triunghiurilor CDA şi
CAB rezultă CDA ≡ CAB (2). Din relaţiile (1) şi (2) rezultă BDA ≡ CDA , iar
m( BDA) + m(CDA) = 180° , de unde : m( BDA) = 90° şi deci m( BAC ) = 90° .

Teorema lui Pitagora: Într-un triunghi dreptunghic pătratul lungimii ipotenuzei este
egal cu suma pătratelor lungimilor catetelor.
Demonstraţie. Fie D proiecţia vârfului A pe ipotenuza BC a triunghiului ABC. Din teorema
catetei obţinem AB 2 = BD ⋅ BC şi AC 2 = CD ⋅ BC , de unde
AB + AC = BC ( BD + CD ) = BC .
2 2 2

Reciproca teoremei lui Pitagora: Dacă într-un triunghi pătratul lungimii unei laturi este
egal cu suma pătratelor lungimilor celorlalte două
laturi, atunci triunghiul este dreptunghic.
Demonstraţie. Fie triunghiul ABC în care BC 2 = AB 2 + AC 2 şi triunghiul dreptunghic
A ' B ' C ' astfel încât A ' B ' = AB şi A ' C ' = AC. Din teorema lui Pitagora aplicată în
triunghiul A ' B ' C ' rezultă B ' C '2 = A ' B '2 + A ' C '2 = AB 2 + AC 2 = BC 2 , de unde
BC = B ' C '. Din congruenţa triunghiurilor ABC şi A ' B ' C ' (conform cazului de congruenţă
LLL) rezultă m( A) = m( A ') = 90°.

II.61. Aria unui triunghi


„Ar fi trebuit sa fii un cerc subţire,
dar n-ai fost, n-ai fost asa.
Ar fi trebuit să fiu un romb subţire,
dar n-am fost, n-am fost aşa..."
Nichita Stănescu
În cele ce urmează notăm cu A[ ABC ] aria triunghiului ABC, cu a,b,c lungimile laturilor
BC,CA, respectiv AB, iar cu ha , hb , hc lungimile înălţimilor triunghiului duse din A,B,
a ⋅ ha b ⋅ hb c ⋅ hc
respectiv C. Din definiţia ariei unui triunghi avem: A[ ABC ] = = = .
2 2 2

341
A
A
ha
c Hb
b
Hc H b c

B a C
Ha
B a C
Ha Fig. 355
Fig. 354

1) Aria unui triunghi este egală cu jumătatea produsului a două laturi înmulţit cu
sinusul unghiului dintre ele.
h h
Demonstraţie. Din triunghiul ABH a avem sin B = a (Fig. 354), sau sin(π − B) = a
c c
a ⋅ c ⋅ sin B
(Fig. 355), de unde ha = c sin B , deci A[ ABC ] = .
2

c ⋅ b ⋅ sin A b ⋅ a ⋅ sin C
Observaţie: Prin permutări circulare obţinem A[ ABC ] = = .
2 2

a+b+c
2) Aria unui triunghi este egală cu A[ ABC ] = p ( p − a )( p − b)( p − c ) , unde p =
2
(formula lui Heron).
Demonstraţie. Avem
a ⋅ b ⋅ sin C ab C C ( p − a )( p − b ) p( p − c)
A[ ABC ] = = ⋅ 2 ⋅ sin ⋅ cos = ab ⋅ sau
2 2 2 2 ab ab
A[ ABC ] = p( p − a)( p − b)( p − c) .

abc
3) Aria unui triunghi este egală cu A[ ABC ] = , unde R este raza cercului circumscris
4R
triunghiului ABC.
a ⋅ c ⋅ sin B ac ac b abc
Demonstraţie. A[ ABC ] = = ⋅ sin B = ⋅ = .
2 2 2 2R 4R

4) Aria unui triunghi ABC este egală cu


A[ ABC ] = pr = ( p − a)ra = ( p − b)rb = ( p − c)rc = rra rb rc , unde r este raza cercului înscris,
iar ra , rb , rc sunt razele cercurilor exînscrise.
Demonstraţie: Aceste formule rezultă imediat din descompunerea triunghiului ABC.
Dacă I este centrul cercului înscris în triunghiul ABC atunci:
1 1 1
A[ ABC ] = A[ AIB ] + A[ BIC ] + A[CIA] = ar + br + cr = pr şi A[ ABC ] = A[ ABI a ] + A[ ACI a ] − A[ BCI a ] =
2 2 2

342
c ⋅ ra b ⋅ ra a ⋅ ra (b + c − a) ⋅ ra
+ − = = ( p − a) ⋅ ra . Ultima formulă se obţine înmulţind
2 2 2 2
primele patru expresii ale ariei triunghiului ABC şi ţinând seama de formula lui Heron.

A B C A B C
5) Aria unui triunghi ABC este egală cu A[ ABC ] = r 2 ctg ctg ctg = ra2 ctg tg tg .
2 2 2 2 2 2
A B C
Demonstraţie. Din p = r ⋅ ctg ⋅ ctg ⋅ ctg . (vezi „Cercul înscris”) şi A[ ABC ] = pr rezultă
2 2 2
A B C
A[ ABC ] = r 2 ctg
ctg ctg . A doua egalitate se obţine utilizând teorema 27) – „Cercuri
2 2 2
exînscrise”, efectuând produsul p( p − a)( p − b)( p − c) .

6) Dacă vârfurile A,B,C ale triunghiului ABC au coordonatele carteziene


( x A , y A ), ( xB , yB ), respectiv ( xC , yC ) , atunci aria triunghiului ABC este egală cu:
xA yA 1
1
A[ ABC ] = ⋅ ∆ , unde ∆ = xB yB 1 .
2
xC yC 1
x y 1
Demonstraţie. Ecuaţia dreptei BC este: xB yB 1 = 0 , iar distanţa de la punctul A la
xC yC 1

dreapta BC este egală cu: d ( A, BC ) = , deci aria triunghiului ABC
( xC − xB ) 2 + ( yC − yB ) 2
1
este egală cu: A[ ABC ] = ⋅∆ .
2

7) Consecinţă: Dacă vârfurile A,B,C ale triunghiului ABC au afixele z A , z B , respectiv zC


zA zA 1
i
atunci aria triunghiului ABC este egală cu: A[ ABC ] = ∆ , unde ∆ = ⋅ z B zB 1 . .
4
zC zC 1

zA + zA
Demonstraţia rezultă imediat utilizând proprietatea 5) şi faptul că x A = şi
2
zA − zA
yA = .
2i

8) Pe laturile triunghiului ABC se consideră punctele A1 , B1 , C1 astfel încât


BA1 CB AC
= p, 1 = q, 1 = r . Dacă {P} = AA1 ∩ CC1 ,{R} = BB1 ∩ CC1 ,{Q} = BB1 ∩ AA1 ,
A1C B1 A C1 B
(1 − pqr ) 2
atunci A[ PQR ] = ⋅ A[ ABC ] .
(1 + p + pq)(1 + q + qr )(1 + r + pr )

343
Lemă: Pe laturile BC şi AC ale unui triunghi ABC se consideră punctele A1 , respectiv B1
BA1 CB
astfel încât = p, 1 = q . Dacă {Q} = AA1 ∩ BB1 să se arate
A1C B1 A
AQ p + 1 BQ
că = , = p(q + 1).
QA1 pq QB1
Demonstraţie. Fie B1 B2 AA1 , B2 ∈ ( BC ) . Avem : A
BQ BA1 AC AC
= = p⋅ 1 = p⋅ = p(q + 1) . Analog,
B1Q A1 B2 A1 B2 AB1
B1
fie A1 A2 BB1 , A2 ∈ ( AC ) , Q A2
AQ AB1 CB1 p +1
avem: = = = (Fig. 356).
QA1 B1 A2 qB1 A2 pq
B A1 B2 C
BQ Fig. 356
Observaţie: Din = p(q + 1) rezultă
B1Q
BQ p + pq AQ p + 1 AQ p +1
= şi din = rezultă = .
BB1 1 + p + pq QA1 pq AA1 1 + p + pq
A[ PQR ] QR PR A[ RB1C ] B1C ⋅ d ( R, B1C ) B1C RB1
Demonstraţie teoremă. Avem = ⋅ , = = ⋅ ,
A[ RB1C ] RB1 RC A[ ABC ] AC ⋅ d ( B, AC ) AC BB1
de unde rezultă că
A[ PQR ] QR PR B1C QR PR CC1 B1C A
= ⋅ ⋅ = ⋅ ⋅ ⋅ (1) .
A[ ABC ] BB1 RC AC BB1 CC1 RC AC
B1
Utilizând lema de mai sus obţinem:
CR pr CR q + qr CB1 Q
= , = (2). Din =q C1 P
CC1 1 + r + pr CC1 1 + q + qr B1 A R
B1C q B A1 C
rezultă = (3) . Mai mult,
AC q + 1
Fig. 357
Q R B Q B R B 1Q
= 1 − = 1 − − =
B B1 B B1 B B1 B B1
p + pq qr (1 + q)(1 − pqr ) PR RC PC
1− − = (4) şi = 1− 1 − =
1 + q + pq 1 + q + qr (1 + q + pq )(1 + q + qr ) CC1 CC1 CC1
(1 + r )(1 − pqr )
(5). Din relaţiile (1), (2), (3), (4) şi (5) rezultă concluzia.
(1 + q + qr )(1 + r + pr )

344

S-ar putea să vă placă și